Sunteți pe pagina 1din 189

CURSO DE PRÁCTICA EN DERECHO LABORAL.

MODULO I

EL INTERCAMBIO EPISTOLAR

A) Contratación y registro

Impugnación de modalidad de contratación

Abuso del “tiempo parcial”'

Intimación del trabajador


Dejo impugnada la modalidad de contratación registrada como “a tiempo
parcial”, dado que la jornada resultante del horario que efectivamente vengo
cumpliendo, los días ... a ... en el horario de ... a ... hs. 1 1 2, excede el máximo
previsto por el art. 92 ter de la LCT y, por tanto, la remuneración devengada
debe calcularse en función de la jornada completa. En consecuencia, los intimo
para que dentro del plazo de 48 hs. 3 paguen las diferencias salariales
resultantes de computar la fijada para la jornada completa, deduciendo los
pagos percibidos a cuenta (art. 260, LCT), por el período ... 4 bajo
apercibimiento de ...5.

1
Referencia normativa. Art. 92 ter, LCT, texto según ley 26.474.

2
El horario indicado debe reflejar una jornada de tiempo superior
a las 2/3 partes de la jornada habitual de la actividad o la jornada
legal de trabajo, según las pautas sobre limitación de jornada que, en
cada caso, resulten del CCT (convenio colectivo de trabajo) o de la ley
11.544 y su reglamentación. Por ejemplo, cuando rige la limitación
general de la citada ley en jornada diurna no insalubre, el máximo de
las 2/3 partes son 32 horas semanales. A partir de allí, se adeuda la
remuneración de jornada completa, puesto que el contrato a tiempo
parcial se habilita para jornadas “inferiores a las 2/3 partes”.
3
El plazo de 48 hs. o dos días hábiles dispuesto por el art. 57 de la
LCT. Para el caso de remuneraciones pendientes, los plazos son los
previstos por el art. 128.

4
Delimitación temporal del reclamo en función del modo de
determinación del salario que rija en el caso concreto. Al ser fijado por
tiempo (art. 104, LCT), la intimación debe incluir la referencia
temporal del o los períodos comprendidos en ella, agregando aquí: por
todo el tiempo no prescripto, o desde mi ingreso, o por los meses de ...
del año ... al mes ... del año ...

5
La advertencia de la conducta a seguir surge de la siguiente
referencia jurisprudencial: “Es requisito necesario para una válida
ruptura del vínculo la existencia de una intimación previa conteniendo
la afirmación de hechos (u omisiones) que configuran la injuria
alegada y el apercibimiento bajo el cual se efectúa el emplazamiento
ya fuera con la finalidad de obtener una revisión de la supuestamente
viciosa conducta de que se trata o la simple posibilidad de que la otra
parte pueda ejercer el derecho a replicar y esta obligación incumbe
tanto al trabajador como al empleador, pues deben conocer cuál será
la determinación que adoptará el uno o el otro para garantizar la
posibilidad de esgrimir oportunamente sus defensas" (CNTrab, Sala
VII, 2/2/07, “Payer, José L. c/Cadenas García, Carlos, y otros”,
LLonline, AR/JUR/362/2007).

Respuestas
a) Rechazamos su TCL n° CD ... por falso e improcedente. Negamos que
del horario de trabajo por Ud. cumplido resulte una jornada superior a la
admitida y que de ello se derive la obligación de pagarle salarios por tiempo no
trabajado. El horario de trabajo que Ud. tiene asignado y debe cumplir, es
de ... de ... a ... hs., no resultando de éste cualquier tipo de jornada en exceso
de la admitida.

b) Contestando su TCL n" CD ... debemos rechazar la intimación allí


contenida por improcedente. Negamos que su jornada de trabajo sea superior
a la admitida y coincida con el horario que Ud. indica. Si bien es cierto que en
el período comprendido entre los días ... y ... Ud. cumplió un horario superior,
ello se dio en el contexto del art. 89 de la LCT, tal como le fuera notificado en
su momento, habiéndosele pagado las horas suplementarias así cumplidas
conforme lo habilita el art. 92 ter de la LCT.

Falsa categoría profesional

Intimación del trabajador


Impugno la categoría profesional registrada de "administrativo B” y la
liquidación de salarios realizada en función de ella, por no corresponderse a las
tareas que real y habitualmente 6 vengo cumpliendo de “vendedor B”
(vendiendo “en forma telefónica productos de ...”), según el nomenclador y la
descripción de tareas del CCT 130/75, lo cual no puede ser desconocido por
Uds., dado que “me liquidan comisiones por ventas y mal pueden pretender
que sea correcta la categoría de administrativo”, razón por la cual los intimo
para que dentro del plazo de 48 hs. 7 paguen las diferencias salariales
derivadas de la incorrecta categoría registrada, por el período comprendido
entre ... y ..., subsanando dicho incumplimiento de registro (falsa categoría
profesional) bajo apercibimiento de "considerarme despedido por su exclusiva
culpa"8.

6
La referencia al carácter habitual de las tareas correspondientes
a una categoría con mayor remuneración, tiene por función descartar
la posibilidad de que se trate de asignación transitoria de tareas
distintas, en el contexto previsto por el art. 78 de la LCT y que sólo
habilitaría al reclamo de diferencias por el tiempo durante el cual esas
tareas distintas se llevaron a cabo.

7
El plazo mínimo de dos días hábiles está previsto en el art. 57 de
la LCT.

8
Por necesidad de advertir la conducta a seguir.

Respuesta
Contestando su TCL n° CD ... debemos rechazar la intimación allí contenida
por improcedente. Negamos que cumpla en forma habitual tareas distintas a
las inherentes a su categoría de “administrativo”, aun cuando su jornada de
trabajo sea superior a la admitida y coincida con el horario que Ud. indica. Si
bien es cierto que, en alguna ocasión, percibió comisiones, ello se dio en el
contexto del art. 78 de la LCT y no implicó una modificación permanente de
tareas que habilite las diferencias salariales que reclama.

Abuso de contratación “eventual” o por agencia

Intimación directa genérica9


En su carácter de empleador real del suscripto, dado que desde mi ingreso fui
afectado a la realización de tareas de ... que se corresponden con su actividad
permanente de ..., en el horario de ..., en el establecimiento habilitado a su
nombre sito en haciendo Uds. intermediar de manera fraudulenta en nuestro
vínculo a dejo impugnadas las condiciones falsas del registro impuestas, por
no coincidir con las reales de la relación laboral que mantengo con Uds. y, por
lo tanto, los intimo para que dentro del plazo de 48 hs. indiquen si procederán
a registrar en legal forma el contrato, con los datos que a continuación indico,
bajo apercibimiento de considerarme despedido por su culpa 10. Asimismo,
conforme lo dispuesto por los arts. 8o, 11, 15 y concs. de la ley 24.013, los
intimo…11”
9
Referencias normativas: arts. 29, 29 bis, 99 y 100, LCT; arts. 69 a
80, ley 24.013, y decr. 1694/06.
Referencia jurisprudencial. “Cuando de acuerdo con el primer párrafo
del art. 29, LCT, se establece que el trabajador ha sido empleado
directo de la empresa usuaria de sus servicios, procede la
indemnización prevista en el art. 8° de la ley 24.013 aunque el
contrato de trabajo haya sido inscripto solamente por la empresa
intermedia” (CNTrab, en pleno, n° 323, 30/6/10, “Vásquez, María L.
C/Telefónica de Argentina SA, y otro s/despido”, DT, 2010-B-2079).

10
Por necesidad de advertir la conducta a seguir.

11
Se sigue el texto de la intimación prevista por el art. 11 de la ley
24.013, que debe ser cursada con copia a la AFIP.

Intimación con impugnación del carácter “eventual” de la modalidad


de contratación
En su carácter de empleador real del suscripto, dado que desde mi ingreso fui
afectado a la realización de tareas de ... que se corresponden con su actividad
permanente de ..., en el horario de ..., en el establecimiento habilitado a su
nombre sito en ..., haciendo Uds. intermediar de manera fraudulenta en
nuestro vínculo a ..., dejo impugnada la modalidad registrada por no
corresponderse con el carácter permanente de mis tareas y los intimo para
que, dentro del plazo de 48 hs., indiquen si procederán a subsanar dicha
irregularidad de registro, bajo apercibimiento de considerarme despedido por
su culpa12. Asimismo, conforme lo dispuesto por los arts. 8°, 11, 15 y concs.
de la ley 24.013, los intimo ...13.

12
Por necesidad de advertir la conducta a seguir.

13
Se sigue el texto de la intimación prevista por el art. 11 de la ley
24.013, que debe ser cursada con copia a la AFIP.
Intimación por vencimiento de plazos máximos admitidos para la
contratación “eventual”14
En su carácter de empleador real del suscripto, dado que desde mi ingreso fui
afectado a la realización de tareas de ... que se corresponden con su actividad
permanente de ..., en el horario de ..., en el establecimiento habilitado a su
nombre sito en ..., haciendo Uds. intermediar de manera fraudulenta en
nuestro vínculo a dejo impugnada la modalidad registrada por no
corresponderse con el carácter permanente de mis tareas, lo cual se ve
ratificado por la extensión del contrato en plazos que exceden los máximos
admitidos para la modalidad eventual y los intimo para que dentro del plazo de
48 hs. indiquen si procederán a subsanar dicha irregularidad de registro, bajo
apercibimiento de considerarme despedido por su culpa 15. Asimismo, conforme
lo dispuesto por los arts. 8°, 11, 15 y concs. de la ley 24.013, los intimo ...16.

14
Las empresas de servicios eventuales sólo podrán mediar en la
contratación de trabajadores bajo la modalidad de trabajo eventual
(art. 77, ley 24.013). La duración de la causa que diera origen a estos
contratos no podrá exceder de seis meses por año y hasta un máximo
de un año en un período de tres años (art. 72, inc. b, ley 24.013).

15
Por necesidad de advertir la conducta a seguir.

16
Se sigue el texto de la intimación prevista por el art. 11 de la ley
24.013, que debe ser cursada con copia a la AFIP.

Motivado en requerimiento a suspensión de la prestación de servicios,


impugnando el carácter “eventual” de éstos
Ante negativa de trabajo, materializada por medio del requerimiento que me
fuera comunicado en el día ... para que concurra a la agencia ... donde me
serían asignadas nuevas funciones, invocando Uds. razones vinculadas al
supuesto carácter “eventual” de mis servicios y la posibilidad de suspender su
prestación por los plazos fijados en el art. 5° del decr. 1694/06, rechazo dicha
pretensión y dejo impugnada la modalidad registrada por ... 17. En
consecuencia, los intimo para que dentro del plazo de 48 hs. otorguen tareas
habituales, aclaren mi situación laboral y se expidan acerca de si procederán a
registrarla en legal forma con los datos que a continuación indico, bajo
apercibimiento de considerarme despedido por su culpa 18. Asimismo, conforme
lo dispuesto por los arts. 8o, 11, 15 y concs. de la ley 24.013, los intimo ...19.

17
La impugnación puede estar dada por no haberse seguido los
requerimientos formales para la contratación especial (art. 90 y ss.,
LCT), exceso de los plazos máximos admitidos (art. 72, ley 24.013),
carácter permanente de las tareas asignadas con abuso de la figura
(art. 99, LCT).

18
Por necesidad de advertir la conducta a seguir.

19
Se sigue el texto de la intimación prevista por el art. 11 de la ley
24.013, que debe ser cursada con copia a la AFIP.

Impugnación de modalidad de encuadramiento convencional

Intimación del trabajador


Dejo impugnada la categoría profesional registrada de “vendedor” y el
encuadramiento convencional que aplican del CCT 130/75 que se derivan en la
liquidación y pago de salarios inferiores a los que deben pagar, puesto que
cumplo funciones inherentes a la categoría profesional de "cocinero”,
comprendidas en el CCT 389/04, aplicables en su establecimiento cuya
actividad principal consiste en “la elaboración de comidas para su venta al
público” y no la “intermediación entre la compra y venta de productos ya
elaborados”1. Por tal motivo, los intimo para que dentro del plazo de 48 hs.
paguen las diferencias salariales resultantes de computar los salarios y
adicionales fijados en el CCT anteriormente citado, para un establecimiento de
su categoría2, deduciendo los pagos percibidos a cuenta (art. 260, LCT), por el
período ...3 bajo apercibimiento de ...4.

1
Los campos incluidos entre comillas se corresponden a las bases
lácticas y convencionales que, en cada caso, puedan tornar viable la
intimación y, por lo tanto, deben ser ajustadas. Debe tenerse en
cuenta que el CCT, aun cuando no requiere de prueba en juicio, debe
ser correctamente individualizado (art. 8o, LCT).

2
Cuando ello incide en la base de fijación salarial debe hallarse
explicado en la intimación, para que constituya un requerimiento claro
de la conducta que debe seguir el destinatario.

3
Delimitación temporal del reclamo.

4
Por necesidad de advertir la conducta a seguir.

Respuestas
a) Rechazamos su TCL n° CD ... por falso e improcedente. Negamos que
nuestra principal actividad sea la que Ud. indica, puesto que nos dedicamos
a .., tal como incluso se desprende de nuestra inscripción ante la autoridad de
fiscalización. Por lo tanto, resulta improcedente el encuadramiento requerido y
no existe incumplimiento de ningún tipo por nuestra parte que habilite y
legitime el apercibimiento contenido en su misiva, el cual, en su caso, correrá
por su exclusiva cuenta y riesgo.

b) Contestando su TCL n° CD ..., negamos que nuestra actividad principal


sea la que Ud. indica. De todos modos, aclaramos que los niveles
remuneratorios que percibe con el encuadre actual, son superiores a los
salarios y beneficios del convenio colectivo que pretende, razón por la cual, no
existe incumplimiento salarial de ningún tipo en nuestro proceder, que
justifique su intimación y legitime el apercibimiento consignado en la parte
final de su misiva.

Registración del CONTRATO (LEY 24.013) 1


Texto común al inicio de las intimaciones por irregularidad
de registro cursadas por el trabajador
Intimo plazo 48 hs. se expida acerca de si procederá a subsanar las
irregularidades de registro laboral que a continuación indico, bajo
apercibimiento de ...2.

a) Omisión total de registro


Términos y apercibimientos previstos por los arts. 8°, 11, 15 y concs. de la ley
24.013, intimóles para que dentro del plazo de 30 días corridos exhiban
comprobantes que acrediten subsanación del incumplimiento de registro
consistente en omisión total de registro, siendo mi fecha de ingreso el ...,
jornada cumplida..., categoría y tarea realizada de..., lugar de trabajo ... 3,
remuneración devengada de $ ...4.

1
Referencia normativa. Arts. 8" a 11 y 15, ley 24.013, y decr.
2725/91.

2
Por necesidad de advertir la conducta a seguir.

3
El requisito previsto por el art. 11 de la ley 24.013 es que con la
intimación el trabajador deberá indicar la real fecha de ingreso y las
circunstancias verídicas que permitan calificar a la inscripción como
defectuosa (en el caso, omisión total de registro).

4
Referencia jurisprudencial. "Para el caso de ausencia total de
registro de la relación laboral no es fundamental indicar las
remuneraciones percibidas, exigidas para el caso de registro
defectuoso, en cuyo caso se debe consignar "la real fecha de ingreso y
las circunstancias verídicas que permitan calificar la inscripción como
defectuosa" (CNTrab, Sala VI, 9/2/05, “Kleki, María c/Udeca SRL
s/despido”, expte. 33.226/02, sent. 57.754).

b) Falsa fecha de ingreso


Términos y apercibimientos previstos por los arts. 9o, 11, 15 y concs. de la ley
24.013, intimóles para que dentro del plazo de 30 días corridos exhiban
comprobantes que acrediten subsanación de incumplimiento de registro
consistente en falsa fecha de ingreso, siendo que cuando mi real fecha de
ingreso fue del ... Uds. falsamente registran como tal, la del ...

c) Falso monto de remuneraciones


Términos y apercibimientos previstos por el art. 10 de la LCT y los arts. 11, 15
y concs. de la ley 24.013, intimóles para que dentro del plazo de 30 días
corridos exhiban comprobantes que acrediten subsanación de incumplimiento
de registro consistente en falso monto de remuneraciones, siendo que desde
mi ingreso ocurrido el he venido percibiendo un complemento remuneratorio
por encima del salario registrado, que en los últimos ... fue de $ ... netos por
períodos ... [“mensuales", "quincenales”, “semanales", “por día”, etcétera].

d) Intimación cursada por la asociación sindical


Tenemos el agrado de dirigirnos a Uds., en nuestro carácter de asociación
sindical de primer grado, con personería gremial, habilitada para representar
los intereses de los trabajadores ... que se desempeñan en el establecimiento
habilitado a su nombre, sito en ..., a los fines previstos por los arts. 9°, 11, 15
y concs. de la ley 24.013 y con motivo de haber detectado la existencia de las
irregularidades de registro que a continuación se detallan. En tal carácter, los
intimamos para que dentro del plazo de 30 días corridos exhiban
comprobantes que acrediten la subsanación de la irregularidad de registro
consistente en ... o, que afecta a los compañeros que más adelante son
individualizados, con sus respectivas fechas de ingreso y datos reales con los
cuales deben ser registrados: a) (DNI), fecha de ingreso el ..., tarea cumplida
de ..., remuneración percibida por $ ...; b) ... (DNI), fecha de ingreso el ....
tarea cumplida de ..., remuneración percibida por $ ...; c) ... (DNI), fecha de
ingreso el ..., tarea cumplida de ..., remuneración percibida por $ ....

e) Copia a la AFIP
En cumplimiento de lo normado por el art. 11 de la ley 24.013, remito copia
del texto de telegrama enviado a mi empleador con domicilio en ... 5.

5
Referencia normativa. Art. 11, ley 24.013. La remisión de copia de la
intimación a la AFIP es requisito para la procedencia de las
indemnizaciones especiales previstas en los arts. 8o a 10 de dicha ley,
no así para la duplicación indemnizatoria prevista por su art. 15. La
remisión de copia a la AFIP puede hacerse presentando la copia
certificada del TCL, junto con un formulario por mesa de entradas, o
bien despachando un telegrama que incluya la transcripción de dicha
intimación, tal como se propone aquí.
Referencia jurisprudencial. "La remisión dispuesta en el art. 11, inc. b,
de la LNE -copia a la AFIP-, solamente resultaría exigible para la
indemnización reclamada en el marco del art. 8" de dicha normativa,
pero no hace a la procedencia de la multa establecida en el art. 15 del
mencionado cuerpo legal, ya que ésta no se encuentra comprendida
en la enumeración introducida por el art. 47 de la ley 25.345, que
solamente alcanza a las multas previstas en los arts. 8o a 10 de la ley
24.013, pero en modo alguno obsta a la duplicación a que alude el
mencionado art. 15, siempre y cuando se hubiere cursado intimación
dirigida al empleador, de manera plenamente justificada” (del
dictamen del procurador fiscal ante la CSJN, Felipe Obarrio, al que se
adhieren los ministros Petracchi, Belluscio, Fayt, Boggiano, Maqueda y
Zaffaroni; CSJN, 31/5/05, “Di Mauro, José c/Ferrocarriles
Metropolitanos SA y otro s/despido”; Fallos, 25:4192). El texto, en
cuanto copia de una intimación cursada dentro de las 24 hs. que exige
la ley, debe incluir los datos del empleador intimado, como
destinatario de aquélla. Aquí debe insertarse el texto de la intimación
de que se trate, en el todo o en la parte pertinente referida a las
irregularidades de registro.

Extensión de Requerimientos – Obligaciones solidarias

a) A la “empresa usuaria" por incumplimiento de agencia


De conformidad con lo dispuesto por los arts. 29 y 29 bis de la LCT, dado el
incumplimiento que más adelante indico por parte de la agencia ... que me ha
contratado para prestar servicios en el establecimiento habilitado a su nombre,
hago extensiva a Uds. la intimación cursada el día de la fecha al intermediario,
en los siguientes términos: ... 1.

1
Se sigue el texto de la intimación cursada a la empresa de
servicios eventuales, por los incumplimientos de que se trate,
respecto de los cuales la empresa usuaria es solidariamente
responsable.

b) Al titular de la explotación (art. 30, LCT)2


Dado su carácter de “concedente” [“contratista principal”, “titular de la
explotación”] en la explotación donde presto servicios de ... que se
corresponden con su actividad normal y específica, de conformidad con lo
dispuesto por el art. 30 de la LCT, hago extensiva a Uds. la intimación cursada
en el día de la fecha a mi empleador registral ... 3, que reviste la condición a su
respecto de “concesionario” [“contratista” "subcontratista”], en los siguientes
términos: ...4.
2
Referencia jurisprudencial. “Es aplicable el art. 705 del Cód. Civil
a la responsabilidad del art. 30 de la LCT” (CNTrab, en pleno, n° 309,
3/2/06, “Ramírez, María I. c/Russo Comunicaciones e Insumos y otro
s/despido”, DT, 2006-A-218).
3
Se siguen los datos del empleador directo o registral o deudor
principal de la relación.

4
Se sigue el texto de la intimación al empleador directo que se
pretende hacer extensiva.

c) Al adquirente del establecimiento (arts. 225, 227, 228 y 230, LCT)


En su carácter de titular actual del establecimiento donde prestara servicios
hasta el ... y hallándose incumplidas al día de la fecha, las obligaciones del
transmitente que a continuación indico, de conformidad con lo dispuesto por el
art. 225 y ss. de la LCT, los intimo para que dentro del plazo de 48 hs. ...

d) Al cesionario del contrato de trabajo (art. 229, LCT)


En su carácter de cesionario y actual empleador del suscripto, hallándose
incumplidas al día de la fecha las obligaciones del cedente que a continuación
indico, de conformidad con lo dispuesto por el art. 229 de la LCT, los intimo
para que dentro del plazo de 48 hs. ...

e) A los socios o controladores en casos de abuso de personalidad


jurídica societaria (arts. 54, 157 y 274, LSC; arts. 512, 1074 y 1109,
Cód. Civil)
En su carácter de “socio” ["controlador”, “director"] y siendo Ud.
personalmente responsable por intervenir en los incumplimientos laborales que
más adelante indico, conforme lo dispuesto por los arts. 54, 157 y 2745 de la
ley 19.550 y el art. 1109 del Cód. Civil, hago extensiva a Ud. la intimación
cursada en el día de la fecha a mi empleador "registrar ... y en los siguientes
términos: ...6.

5
El art. 157 corresponde en los casos de sociedades de
responsabilidad limitada y el art. 274 cuando se trata de sociedades
anónimas. Según el régimen de responsabilidades personales derivado
de la ley 19.550, su art. 157 declara que los gerentes tienen las
mismas obligaciones que los directores de la sociedad anónima. De
ello se sigue que también tienen las mismas responsabilidades, a las
que se refiere el art. 274 y que debe ser citado cuando se pretende
extender la responsabilidad a los directores de sociedad anónima. De
acuerdo con esta última norma, los directores responden a terceros
“por cualquier otro daño producido por dolo, abuso de facultades o
culpa grave”. La norma está redactada con la amplitud suficiente para
cubrir los daños provocados a los trabajadores de la sociedad, con
motivo de irregularidades en el registro de su contrato (arts. 1109,
1081 y concs., Civil).

6
Se sigue el texto de la intimación que se pretende hacer
extensiva, sobre la base normativa de la solidaridad prevista en el art.
30 de la LCT.

Derechos y deberes de las partes

Prestación del servicio

a) Incumplimiento del empleador al deber de dar ocupación (casos


genéricos)

Intimación cursada por el trabajador


Ante negativa de trabajo 1 desde el día cuando ... 2, intimo plazo 48 hs.3, aclare
mi situación laboral4, bajo apercibimiento de considerarme despedido por su
culpa5.
1
Se utiliza la expresión “negativa de trabajo”, atendiendo a que la
intimación tiene por causa el incumplimiento a la obligación del
empleador de dar ocupación (art. 78, LCT) y no un "despido”, como
forma de extinción del contrato de trabajo que requiere una
comunicación por escrito (art. 243), y prueba por escrito (art. 235).
En caso de utilizarse la expresión “despido verbal” u otras similares
que denoten la previa extinción del contrato, el reconocimiento del
empleador neutraliza la procedencia de las indemnizaciones
especiales de la ley 24.013, dado que con éste habría quedado
operada la extinción del contrato con carácter previo a la intimación,
no cumpliéndose de ese modo con el requerimiento contenido en el
art. 3°, inc. 1, del decr. 2725/91, reglamentario del art. 11 de la ley
24.013. La intimación, para que produzca los efectos previstos en este
artículo, deberá efectuarse estando vigente la relación laboral.

2
Referencia jurisprudencial. “Es requisito necesario para una
válida ruptura del vínculo la existencia de una intimación previa
conteniendo la afirmación de hechos (u omisiones) que configuran la
injuria alegada y el apercibimiento bajo el cual se efectúa el
emplazamiento ya fuera con la finalidad de obtener una revisión de la
supuestamente viciosa conducta de que se trata o la simple
posibilidad de que la otra parte pueda ejercer el derecho a replicar y
esta obligación incumbe tanto al trabajador como al empleador, pues
deben conocer cuál será la determinación que adoptará el uno o el
otro para garantizar la posibilidad de esgrimir oportunamente sus
defensas” (CNTrab, Sala VII, 2/2/07, "Payer, José L. c/Cadenas
García, Carlos, y otros”, LLonline, AR/JUR/362/2007).

3
El plazo de 48 hs. o dos días hábiles, está impuesto por el art. 57
de la
LCT.

4
La intimación debe posibilitar la conservación del contrato (art.
10, LCT) y tiene por objeto que el empleador conteste (art. 57, LCT),
sea limitándose a negar los hechos, o a explicar sus causas, motivos o
circunstancias.

5
La intimación debe anunciar claramente la conducta posterior
que adoptará el trabajador (ver referencia jurisprudencial), sea
interpretando la persistencia del incumplimiento como injuria (art.
242, LCT). Hay supuestos especiales, referidos a contextos distintos y
que requieren la reformulación del apercibimiento [ins variandi, art.
66, LCT; al régimen de tutela a la libertad sindical en la ley de
asociaciones sindicales (LAS), situaciones de violencia en el empleo],
todos los cuales son objeto de tratamiento.

Respuestas

a) Rechazamos su TCL n° CD ... por falso e improcedente. Negamos los


hechos por Ud. invocados, siendo falso que le hayan sido negadas tareas en
las circunstancias que indica. En cuanto a su situación laboral, la realidad es
que se halla ausente sin aviso ni justificación desde el día ..., razón por la cual
lo intimamos para que dentro del plazo legal retome tareas bajo
apercibimiento de considerar extinguido el contrato por abandono de trabajo a
Ud. imputable (art. 244, LCT).

b) Contestando su TCL n° CD ..., tal como le fue informado en la


oportunidad que Ud. indica, la empresa decidió prescindir de sus servicios lo
cual le fuera informado por carta documento n° CD ... de fecha ..., cursada al
último domicilio real por Ud. informado (sito en ...) y en los siguientes
términos:
Queda de ese modo aclarada su situación laboral.

Incumplimiento del trabajador (intimación fundada POR AUSENCIAS


AL EMPLEO)

Intimación cursada por el empleador


Ausente sin aviso ni justificación desde el ... 6, lo intimamos para que dentro
del plazo legal7 retome tareas, caso contrario consideraremos abandono de
trabajo y configurada la extinción del vínculo conforme art. 244 de la LCT.
6
Referencia jurisprudencial. "A los fines de acreditar la
configuración de un abandono de trabajo en los términos del art. 244
de la LCT, resulta menester, además de la previa intimación al
trabajador, probar que el ánimo de éste fue el de no reintegrarse a sus
tareas, ya que no toda ausencia permite inferir la existencia de ese
elemento subjetivo” (CNTrab, Sala VII, 26/8/09, “Doval, María C.
c/Angeducar SA s/despido”, LLonline, AR/JUR/32964/2009). "La
figura del abandono de trabajo requiere para su configuración: a) la
inejecución por el trabajador, sin aviso, de la prestación laboral; b) la
intimación de reintegro, dentro de un plazo razonable según las
circunstancias, y c) la persistencia del trabajador en su conducta
omisiva, durante el plazo fijado” (CNTrab, Sala VIII, 28/3/08,
“Morales, Marcelo D. c/Cafhelar SA”, LLonline, AR/JUR/3106/2008).

7
Referencia normativa. Art. 57, LCT, plazo no inferior a dos días
hábiles. Referencia jurisprudencial. “El despido por la causal de
abandono de trabajo es incausado si la empleadora intimó a la
trabajadora a reintegrarse a sus tareas en el término de veinticuatro
horas -en el caso, también se consideró incausado porque la
empleadora se encontraba en mora antes del distracto-, pues el plazo
del art. 244 de la LCT, aunque allí no se establezca uno expreso, nunca
puede ser inferior al de dos días hábiles, que resulta de aplicar el que
contempla el art. 57 de ese mismo texto legal” (CNTrab, Sala VII,
31/3/08, "Diez, Claudia M. c/Valentini, Lorena R.”, LLonline,
AR/JUR/1655/2008).

Respuestas

1) Rechazo su carta documento n° CD ... y la intimación allí cursada por


improcedente. Tal como informé en su momento, me encuentro imposibilitado
de concurrir a trabajar por hallarme enfermo en mi domicilio, a disposición del
control médico de su facultativo.
2) Rechazo su carta documento n° CD ... y la imputación de ausencias al
empleo por ser falsos los hechos que indican Uds. Desde el día ... me vengo
presentando a trabajar en mi horario habitual y no se me permite hacerlo,
alegando que ... Sin perjuicio de ello, el día ... me presentaré a retomar tareas
en cumplimiento de su intimación, acompañado de testigos. Quedan intimados
a otorgarlas en esa oportunidad, bajo apercibimiento de considerarme
despedido por su culpa.

3) Rechazo su carta documento n° CD ... y la intimación allí cursada por


improcedente. Hallándose Uds. en mora con relación al pago de mis salarios
correspondientes a los meses de ..., como asimismo con relación a los demás
incumplimientos que fueron objeto de la intimación que les cursara mediante
TCL n” CD ..., sólo retomaré tareas en oportunidad que ofrezcan regularizar
dichas obligaciones a su cargo (art. 1201, Cód. Civil). A todo evento,
transcribo el tenor del TCL antes referido y cuyas intimaciones mantengo:

Modificación de condiciones de trabajo (Ius variandi)

Empleador notifica modificación de condiciones 1

1) Lugar de trabajo
Como medida de alcance general2, derivada del traslado del establecimiento de
su lugar actual al domicilio de ..., notificámosle que a partir del día ... deberá
presentarse a cumplir tareas en este último domicilio, manteniéndose las
demás condiciones habituales de trabajo.
2) Sector de trabajo
Por razones de reestructuración del sector ... donde Ud. se desempeña
actualmente, notificárnosle que a partir del día ... pasará a cumplir tareas de
similares características e inherentes a su calificación profesional, en el
sector ..., manteniéndose las demás condiciones habituales de trabajo.
3) Horarios de trabajo
Por razones de modificación de nuestro proceso productivo, originadas en el
sustancial incremento que hemos tenido en la demanda de nuestros productos
y la necesidad de atenderlos con la mayor actividad posible de nuestros
recursos, le notificamos que hemos resuelto modificar el sistema actual de
turnos fijos, por el sistema de turnos rotativos, medida que comenzará a regir
a partir del día ... En los próximos días se le informarán las características del
sistema de rotación y la compensación que se ha previsto, por las molestias o
inconvenientes que el cambio pudiera ocasionarle.

1
Referencia normativa. Art. 66, LCT. El llamado ius variandi
constituye una facultad del empleador a modificar de manera
unilateral “la forma y modalidades de la prestación del trabajo, en
tanto esos cambios no importen un ejercicio irrazonable de esa
facultad, ni alteren modalidades esenciales del contrato, ni causen
perjuicio material ni moral al trabajador”.
Se han señalado tres condiciones para el ejercicio legítimo del ius
variandi. Ellas están dadas por el propio contrato individual de trabajo
(limitación contractual), por los fines de la empresa (limitación
funcional) y por la consideración de los legítimos intereses del
trabajador (limitación por el "deber de previsión").

2
La referencia al carácter general de la medida, cuando así
corresponde, acota el margen de la opción que pueda ejercer el
trabajador, puesto que tal carácter obsta la promoción de la acción
sumarísima tendiente al restablecimiento de las condiciones alteradas
(ley 26.088).

Rechazo e intimación por ejercicio abusivo del ius variandi


1) Con opción de restablecimiento de condiciones alteradas3
Rechazo su pretensión tendiente a modificar mi sector habitual de trabajo,
pasando de ... a ... por importar dicha medida un ejercicio abusivo del ius
variandi. En efecto, el trabajo en el sector que Uds. pretenden asignarme trae
aparejado ... que representa un perjuicio ... consistente en ... 4. Además, la
medida no es razonable, dado que ...5. Por lo expuesto, los intimo para que
dentro del plazo de 48 hs. dejen sin efecto la medida y mantengan mis
condiciones habituales de trabajo, caso contrario ejercitaré la opción prevista
por el art. 66 de la LCT accionando judicialmente por el restablecimiento de las
condiciones que pretenden alterar.

3
Sólo frente a medidas de alcance individual.

4
Por razones de colaboración y buena fe (arts. 62 y 63, LCT), la
intimación debe contener una justificación de los motivos por los
cuales el cambio de tareas le ocasiona perjuicio material y/o moral al
trabajador. La explicación del perjuicio posibilita que el empleador
ofrezca compensarlos, para evitar la resistencia a la medida o la
extinción del contrato.

5
La comunicación puede, al mismo tiempo, contener la
impugnación referida a la razonabilidad del cambio, sobre todo en los
casos donde éste encubre una sanción disciplinaria (acción no
permitida por el art. 69, LCT) o móviles del tipo discriminatorios, lo
cual debe ser explicado.

2) Con opción de despido indirecto

a) Cambio de lugar de trabajo


Rechazo su pretensión de modificar mi lugar habitual de trabajo sito en... al
nuevo en..., por causarme perjuicio la medida, puesto que representa a mi
respecto un mayor tiempo de viaje (en el caso ...) y un mayor costo de
transporte (dado que ...). No habiendo propuesto como compensar el mayor
tiempo de viaje con reducción equivalente de jornada, ni el mayor costo en el
transporte con su provisión en especie o compensación del gasto, los intimo
para que dentro del plazo de 48 hs. dejen sin efecto la medida o propongan
alternativas tendientes a evitar los perjuicios indicados, caso contrario ejerceré
la opción prevista por el art. 66 de la LCT considerándome despedido por su
culpa.

b) Cambio de honorarios de trabajo


Rechazo su pretensión de modificar los horarios habituales de trabajo en turno
fijo, por uno nuevo en turnos rotativos, puesto que la medida exige invadir mi
tiempo libre y que tengo afectado a otras actividades, particularmente ... 6. Por
lo expuesto, los intimo para que dentro del plazo de 48 hs. dejen sin efecto la
medida, manteniendo en mi caso los horarios actuales de trabajo, caso
contrario ejerceré la opción prevista por el art. 66 de la LCT considerándome
despedido por su culpa.

6
La diagramación del tiempo libre que hace el trabajador
pertenece a las estructuras de la relación. Sólo un criterio muy
restrictivo de interpretación, puede llevar a la exigencia que el
trabajador explique cómo tiene empleado el espacio de tiempo que
pasaría a ser invadido por el cambio.
Debe tenerse en cuenta que si bien el empleador “tiene facultades
suficientes para organizar económica y técnicamente la empresa,
explotación o establecimiento" (art. 64, LCT), éstas, como poder que
la sociedad habilita para el sector privado, deben ser ejercidas "con
carácter funcional, atendiendo a los fines de la empresa, a las
exigencias de la producción, sin perjuicio de la preservación y mejora
de los derechos personales y patrimoniales del trabajador” (art. 65).
De manera tal que siempre que el empleador pretenda modificar
condiciones estructurales del contrato, como ser el horario de trabajo,
sus razones deben ser justificadas y analizadas bajo el prisma
funcional del citado art. 65.

a) Intimaciones cursadas por el trabajador con motivo DE


INCUMPLIMIENTOS SALARIALES

1) Al pago de salarios vencidos


Habiendo vencido los plazos legales sin que cumplieran con el pago de “la
remuneración correspondiente al mes de "a la ... quincena del mes de ...”, “al
SAC ... cuota del año ...”, los intimo para que dentro del plazo de 48 hs.
cumplan con dicha obligación, bajo apercibimiento de “accionar judicialmente”,
"retener tareas a su costa hasta tanto regularice el incumplimiento",
"considerarme despedido por su culpa”.

Respuesta
Rechazamos su TCL n° CD ... por falso e improcedente. Las remuneraciones
que indica estuvieron a su disposición en el lugar de trabajo y la mora es sólo
a Ud. imputable, razón por la cual lo intimamos para que dentro del plazo de
48 hs. proceda a percibirlas, caso contrario nos veremos obligados a consignar
judicialmente el crédito a su costa.

2) Al pago de diferencias salariales (derivadas del pago


insuficiente)
Intimo plazo 48 hs. pague diferencias salariales por ... 1 2, derivadas del pago
de remuneración por valores inferiores a los devengados 3 de acuerdo a mi
calificación profesional de ... 4 fijados en escalas salariales complementarias al
CCT ...* * 5, bajo apercibimiento de ...6.

1
Referencias normativas. Prohibición de pagar salarios inferiores
(art. 119, LCT); plazo para el cumplimiento de la obligación por el
empleador (arts. 127 y 128); instrumentación del pago (arts. 138 a
143).
2
Delimitación del reclamo en función del modo de determinación
del salario que rija en el caso concreto. Al ser fijado por tiempo (art.
104, LCT), la intimación debe incluir la referencia temporal del o los
períodos comprendidos en ella, agregando aquí: por todo el tiempo no
prescripto, o desde mi ingreso, o por los meses de ... del año ... al
mes ... del año ...

3
Referencia normativa. Art. 119, LCT.

4
Tratándose de intimación al pago de salarios fijados en CCT,
deben incluirse los datos necesarios, en referencia del convenio de
que se trate, para su corrección por el destinatario. En el caso, la
categoría profesional debe coincidir con alguna de las definidas en las
escalas salariales complementarias al convenio invocado. Del mismo
modo, si la intimación comprende el pago de adicionales
puntualmente resultantes de un CCT y que no fueron pagados, debe
así decirlo, por ejemplo por haberse omitido el pago del adicional
por ... También la intimación, en el contexto de remuneraciones
fijadas a jornal horario, puede ser resultante del cómputo de una
cantidad de horas inferior a la efectivamente cumplida, debiendo en
este caso especificarse el horario efectivamente cumplido (del cual
resultará la cantidad de horas que atañen a la base del reclamo) e
impugnación de la cantidad de horas incluida en la liquidación de
haberes (por lo general consignadas en los duplicados de recibos
extendidos por el empleador).

5
Referencia normativa. Arts. 7o a 9o y 16, LCT.

6
Debe advertirse al destinatario la conducta que será seguida en
caso de incumplimiento (ver § 1, a). Concretamente, si el
requerimiento es bajo apercibimiento de considerarme despedido por
su culpa o de accionar judicialmente.
Respuestas
a) Rechazamos su TCL n° CD ... por falso e improcedente. Negamos
adeudarle las diferencias salariales que reclama, siendo que los salarios
pagados y que constan en los recibos emitidos en legal forma, integrados con
su firma y cuyos duplicados obran en su poder, se ajustan a los salarios fijados
en el convenio colectivo aplicable, su categoría profesional, tarea y horario
cumplidos. En ese contexto, no existe incumplimiento de ningún tipo por
nuestra parte que habilite y legitime el apercibimiento contenido en su misiva,
el cual, en su caso, correrá por su exclusiva cuenta y riesgo.

b) Contestando su TCL n" CD ..., le informamos haber realizado la compulsa


de las liquidaciones de sus salarios, en los períodos que Ud. indica, detectando
que existen diferencias a su favor, razón por la cual procederemos a depositar
el importe de éstas en su cuenta sueldo y por el importe de $ ... Según
nuestros cálculos, dicho importe se corresponde a ... y no fue cancelado en
tiempo propio por un error que subsanamos en este acto.

3) Al pago de diferencias salariales (motivadas en reducción del


salario)
Habiendo Uds. procedido de manera ilegítima a reducir mi remuneración
devengada desde el mes de ... del año ..., los intimo para que dentro del plazo
de 48 hs. paguen las diferencias salariales así adeudadas, bajo apercibimiento
de ...7.

7
Sobre advertencia de conducta a seguir, ver referencia anterior.

Respuesta
Rechazamos su TCL n° CD ... por falso e improcedente. Negamos haber
reducido el importe de sus remuneraciones por los períodos que indica, siendo
que en los mismos el menor importe resultante obedece a ... (por ejemplo:
"haber disminuido la cantidad de horas extraordinarias que le eran requeridas
con anterioridad, dado que Ud. no está obligado a realizarlas, tampoco puede
pretender que estemos obligados a mantenerlas” o “que han cesado las tareas
superiores que transitoriamente le fueron requeridas por el período anterior a
su reclamo, retornando desde entonces a las tareas propias de su calificación
profesional, cumpliéndose de ese modo la condición resolutoria a la cual se
hallaba subordinado el adicional, conforme lo dispuesto por el art. 78, LCT").
En ese contexto, no existe incumplimiento de ningún tipo por nuestra parte
que habilite y legitime el apercibimiento contenido en su misiva, el cual, en su
caso, correrá por su exclusiva cuenta y riesgo.

4) Al reintegro de deducciones indebidas


Habiendo Uds. procedido de manera ilegítima a descontar de mi salario
correspondiente “a la ... quincena” [“al mes de ...”] la cantidad de $ ... con la
mención de “reparación de rotura”, siendo dicha deducción improcedente de
acuerdo a derecho8, los intimo para que dentro del plazo de 48 hs. completen
el pago del salario correspondiente al período indicado, bajo apercibimiento
de ...9.

b) Intimación al pago de comisiones10


Habiendo omitido el pago en tiempo propio de las comisiones del ... %
devengadas a mi favor de acuerdo a11, por mi intervención en las operaciones
“nos ... del registro de la empresa" ya concertadas 12 y facturadas por Uds., los
intimo para que dentro del plazo de 48 hs. procedan a su pago, bajo
apercibimiento de ...

8
Referencia normativa. Arts. 131 y 132, LCT.
9
Sobre advertencia de conducta a seguir, ver § 7, a, 2.
10
Fuente normativa. Arts. 108 a 111, LCT.
Referencia jurisprudencial. "Si bien el silencio del empleador frente a
la intimación del trabajador accionante fue injustificado y activó la
presunción establecida en el art. 57 de la LCT, en el caso resulta difícil
la aplicación misma de la presunción contenida en la norma, ya que el
reclamante no precisó en ninguna de sus comunicaciones a qué
comisiones adeudadas aludía, cuál habría sido el hecho generador del
concepto, cuáles sus pautas de liquidación, o al menos qué
operaciones no le habrían sido abonadas con comisiones, máxime si se
tiene en cuenta que no se probó la percepción de comisiones ni su
falta de pago” (CNTrab, Sala III, 18/11/09, “Portillo, Élida E.
c/Actionline de Argentina SA y otro”, LLonline, AR/JUR/48270/2009).

11
Referencia a la fuente obligacional de que se trate, como ser el
contrato individual o CCT.

Referencia jurisprudencial (viajantes de comercio). “A efectos de


calcular las comisiones sobre las ventas y la indemnización por
clientela correspondiente a un viajante de comercio, debe partirse de
los montos reclamados por el actor, quien efectúa el juramento
previsto en el art. 11 de la ley 14.546, mientras que la demandada
omitió acompañar la documentación respaldatoria de las operaciones
concertadas” (CNTrab, Sala VI, 9/5/05, "Casais, Zulma S. c/Coca Cola
FEMSA de Buenos Aires SA”, LL, 2005-D-918, y LLonline,
AR/JUR/1558/2005).

12
Debe tratarse de operaciones concertadas con intervención del
trabajador, según lo dispuesto por el art. 108 de la LCT. Advertencia
de la conducta a seguir, ver § 7, a, 2.

a) Rechazamos la intimación contenida en su TCL n° CD ... por


improcedente. Negamos que la modalidad de su salario incluya comisiones y
que Ud. haya concertado en forma personal las operaciones que indica, razón
por la cual, no existe incumplimiento salarial de ningún tipo en nuestro
proceder, que justifique su intimación y legitime el apercibimiento consignado
en la parte final de su misiva.
b) Contestando su TCL n° CD ... le informamos que las comisiones que
indica serán pagadas juntamente con su remuneración principal del período en
curso, tal como lo determina el art. 127 de la LCT. No habiendo vencido los
plazos legales, rechazamos su intimación por improcedente.

Jornada

a) Requerimiento del trabajador a diagramar su jornada 1


En atención a que el horario de trabajo que tengo asignado, de ... días en el
horario de ...2 no cumple con el régimen legal vigente de limitación de jornada
y descansos3, lo intimo plazo 48 hs. notifique diagramación de jornada de
trabajo acorde a límites legales y que permita descanso semanal conforme art.
204, LCT o compensatorio en términos del decr. 16.117/33 bajo
apercibimiento de ...4.

1
Debe tenerse en cuenta que el trabajador no está obligado a
trabajar horas extraordinarias (art. 203, LCT) y que las horas
trabajadas en infracción al descanso, no son compensables en dinero.

2
Las bases fácticas de la intimación deben contener esta
descripción y de ésta debe resultar el trabajo en tiempo superior al
debido, sea en los límites generales que surgen de la ley 11.544 y su
reglamentación, o de los especiales que pudieran derivarse de otra
fuente normativa (EP o CCT).

3
Cuando la jornada cumplida supere la especial fijada en EP o CCT
aplicable, se debe individualizar la fuente obligacional para delimitar
el objeto de la conducta requerida por el destinatario y cumplir con lo
dispuesto, en su caso, por el art. 8", LCT.

4
Advertencia de la conducta a seguir, ver § 1, a.

Respuesta
Rechazamos su TCL n° CD ... por falso e improcedente. Negamos que el
horario asignado sea el indicado por Ud. Su horario de trabajo es de ... a ... y
de ... a ... hs., con descanso semanal desde las 13 hs. del ... hasta las 24 hs.
del ajustándose éste a las disposiciones legales que indica. Por lo tanto,
ninguna medida debemos adoptar en el sentido que lo exige Ud.

b) Intimación del trabajador al paco de horas extras 5.


Habiendo omitido el pago, intimo plazo 48 hs. pague las horas extras
resultantes del horario de trabajo que efectivamente vengo cumpliendo, los
días ... a ... en el horario de ... a ... hs. 6 por el período comprendido entre ... 7,
bajo apercibimiento de ...8.

5
Referencias normativas. Ley 11.544; decr. 15.115/33; arts. 200 a
203, LCT.

6
El horario indicado debe reflejar una jornada de tiempo superior
a la jornada convencional o legal de trabajo. En el primer caso,
indicarse la fuente normativa que así la tenga prevista (ver § 7, b) y
tener en cuenta la doctrina resultante del plenario 226 (CNTrab,
25/6/81, “D’Aloi, Salvador c/Selsa SA”, DT, 1981-1207, y ED, 95-
169). En el segundo caso, del horario debe resultar una cantidad de
horas trabajadas superior a la limitación fijada en la ley 11.544 y su
reglamentación. La jornada legal diurna (de 6 a 21 hs.) fijada por la
ley 11.544 es de ocho horas diarias o cuarenta y ocho semanales. En
caso de distribución desigual de la jornada se admite recargar la diaria
en no más de una hora (hasta nueve horas por día). En trabajo
nocturno, la jornada es de 7 horas diarias o cuarenta y dos semanales,
con igual previsión para el caso de distribución desigual de la jornada.
En trabajo mixto (diurno y nocturno), cada hora de trabajo nocturno
equivale a una hora con ocho minutos de trabajo diurno). En trabajo
por turnos o por equipos se admite la jornada de ocho horas en
jornada nocturna, con descanso compensatorio de un día, cada siete
de trabajo (art. 9°, decr. 16.115/33). En trabajo donde media
declaración administrativa de insalubridad, la jornada es de seis horas
diarias o treinta y seis semanales. Sólo son horas extras las
trabajadas en exceso de estas jornadas y deben ser pagadas con
recargo del 50%, salvo las trabajadas en días sábados después de las
13, domingos o feriados, donde rige el recargo del 100% (art. 201,
LCT).
Las horas trabajadas en estos últimos días no generan recargo alguno,
cuando no superan los límites indicados de la jornada, sólo generan el
derecho a un descanso compensatorio, que debe ser otorgado
conforme el art. 204 de la LCT y art. 18 del decr. 16.117/33.

7
Delimitación temporal del reclamo, ver cap. primero, § 1, nota 4.

8
Advertencia de la conducta a seguir, ver cap. primero, § 1, a,
nota 5.

Descansos

a) Descanso semanal y compensatorio


Trabajador notifica a su empleador el goce de descanso compensatorio no
otorgado'
Habiéndose requerido la prestación de mis servicios en tiempo de descanso
semanal, los días ... y ..., sin que en la semana siguiente me fuera otorgado el
descanso compensatorio, notifico a Uds. que de conformidad con lo dispuesto
por el art. 207 de la LCT y el decr. 16.117/33, haré uso de aquél el día ... 2.
Quedan intimados al pago del recargo legal, juntamente con los salarios del
período, bajo apercibimiento de ...3.

b) Licencia anual
1) Empleador notifica a trabajador el otorgamiento de la licencia
anual4
En cumplimiento de la legislación vigente, notificamos a Ud. que la licencia
ordinaria que le corresponde por el año en curso, es de ... días corridos y le
fue asignado para su otorgamiento el período comprendido entre los días ...
y ... del mes de ... del año ..., ambas fechas inclusive, debiendo reincorporarse
a sus tareas el día ... en su horario habitual.

2) Trabajador notifica a su empleador el goce del descanso anual no


otorgado5
Habiendo vencido los plazos legales sin que me fuera notificado el período de
otorgamiento de los ... días de licencia ordinaría que me corresponden por el
año ..., conforme lo dispuesto por el art. 157 de la LCT, notifico a Uds. que
haré uso de ellas desde el día ... hasta el día ... inclusive, reintegrándome a
mis tareas el día ... en horario habitual.

c) Licencias especiales. Trabajador notifica al empleador


EL GOCE DE LICENCIA ESPECIAL6
Notifico que con motivo de ..., haré uso de la licencia especial correspondiente
de ... días, los días ... a ... del mes de ..., reintegrándome a mis tareas
habituales, con el correspondiente certificado que justifica el hecho, el día ...
en mi horario habitual.

1
Referencia normativa. Art. 207, LCT.

2
Se requiere la notificación con 24 hs. de anticipación (art. 207,
LCT).

3
Advertencia de la conducta a seguir, ver § 1, a, nota 5.
4
Referencia normativa. Arts. 150 a 157, LCT.

5
Salvo disposición legal o convencional expresa en contrario, esta
notificación debe ser cursada por escrito al trabajador, con cuarenta y
cinco días de anticipación al inicio de sus vacaciones. Deben ser
otorgadas entre el 1o de octubre y el 30 de abril del año siguiente, por
la cantidad de días corridos que fija el art. 150 de la LCT, según su
antigüedad al 31 de diciembre del año al que corresponden. Para
gozar este derecho, el trabajador debe haber prestado servicios
durante la mitad, al menos, de los días hábiles del año (art. 151, LCT),
aunque se consideran trabajados aquellos donde no prestó servicios
por gozar de licencia legal o convencional o por enfermedad inculpable
o infortunio del trabajo o causas no imputables al mismo (art. 152).
En tal caso, le corresponderá gozar una licencia proporcional de un día
de descanso por cada veinte de trabajo efectivo (art. 153). La licencia
debe comenzar en día lunes, el siguiente si fuese hábil o el primer día
laborable luego de su descanso semanal compensatorio.

6
Referencia normativa. Art. 157, LCT.

Violencia en el empleo

a) Cese de trato discriminatorio1


Intimo plazo 48 hs. disponga medidas efectivas tendientes al cese del trato
discriminatorio que vengo sufriendo con motivo de ... 2, consistentes en ...3,
haciéndome conocer dichas medidas mediante respuesta fehaciente a la
presente, bajo apercibimiento de ...4.

1
Referencia normativa. Arts. 158 a 161, LCT.

2
Referencias normativas. Convenios OIT, 100 y 111; arts. 14 bis y
16, Const, nacional; arts. 17, 17 bis y 81, LCT, y ley 23.592 (ver
Mansueti, Discriminación, elDial, DC9BB).
Este espacio debe ser completado explicando la causal de exclusión
que se deriva en la conducta discriminatoria sufrida, como ser el
género, la raza, opinión política, etcétera.

3
Debe incluirse una descripción de los hechos configurativos del
incumplimiento a la obligación de trato igualitario. Como ser el pago
de remuneraciones inferiores por trabajo igual y de igual valor;
privación de beneficios otorgados a otros en igualdad de
circunstancias; modificación de condiciones de trabajo vinculadas a
haber tomado conocimiento el empleador de alguna de las causales de
exclusión (sobre los alcances de la igualdad en materia salarial, ver
Mansueti, Igualdad salarial y discriminación, ED, 236-326).
4
Advertencia de la conducta a seguir, ver § 1, a, nota 5.

Respuestas
a) Rechazamos su TCL n° CD ... por falso e improcedente. Negamos la
veracidad de los hechos que Ud. indica y que ellos tengan la entidad
discriminatoria que les adjudica. No es cierto que ... Por lo tanto, ninguna
medida debemos adoptar en el sentido que Ud. lo exige.

b) Contestando su TCL n" CD ..., le informamos que no nos consta la


veracidad de los hechos que Ud. invoca. De todos modos, en lo inmediato
procederemos a investigar lo ocurrido. Asimismo, en forma preventiva hemos
resuelto que Ud. pase a desempeñar sus tareas habituales en el sector de ...,
manteniendo las modalidades de contratación y por el tiempo que demande
nuestra investigación, que será de ... días.

Cese de hostigamiento o acoso moral5


Intimo plazo 48 hs. ajuste su comportamiento a lo dispuesto por los arts. 62 y
63 de la LCT, disponiendo medidas efectivas tendientes al cese del
hostigamiento en mis condiciones de trabajo 6, consistentes en ...7 llevadas a
cabo por ...8 y que tienden a mi separación de la organización, haciéndome
conocer dichas medidas mediante respuesta fehaciente a la presente, bajo
apercibimiento de ...9.

5
Mansueti, El "mobbing” y el derecho, elDial.com, DC886.

6
Referencia jurisprudencial. “Corresponde considerar justificado
el despido indirecto en que se colocó un trabajador ante la falta de
respuesta por parte de la patronal respecto a la solicitud que formuló
en el sentido de que se garantizara su seguridad física dentro del
establecimiento luego de la agresión sufrida por parte de un
compañero de trabajo, si tanto los términos de la respuesta al
requerimiento telegráfico como los de la contestación de la demanda
ponen en evidencia que la demandada desoyó de plano el pedido sin
ningún argumento que justifique ese proceder, lo que debe reputarse
reñido con los deberes de buena fe y de asegurar la integridad
psicofísica del trabajador, que se infiere de los arts. 62, 63, 75 y 77 de
la LCT" (CNTrab, Sala I, 7/7/10, "González Gómez, Sebastián M.
c/Jumbo Retail Argentina SA", LLonline, AR/JUR/42178/2010).

7
Debe incluirse una descripción de los hechos configurativos del
incumplimiento a la obligación de trato igualitario. Como ser el pago
de remuneraciones inferiores por trabajo igual y de igual valor;
privación de beneficios otorgados a otros en igualdad de
circunstancias; modificación de condiciones de trabajo vinculadas a
haber tomado conocimiento el empleador de alguna de las causales de
exclusión (sobre los alcances de la igualdad en materia salarial,
Mansueti, Igualdad salarial y discriminación, ED, 236-326).

8
A efectos que el empleador adopte medidas o, en caso negativo,
opere a su respecto la responsabilidad subjetiva del art. 1109 del Cód.
Civil (la objetiva del art. 1113, párr. 2°, opera cuando el acto es
ejecutado por el dependiente), debe individualizarse al autor de los
hechos configurativos de acoso moral (si es del tipo vertical, en la
persona del superior jerárquico que ejerce la voluntad del empleador;
si es del tipo descendente o bossing la habilitación de medidas contra
los subordinados con portación de apellido o prerrogativas especiales;
si es del tipo horizontal, la individualización pasará por el sector de la
organización -empleados o proveedores- sobre el cual deben recaer
las medidas).

9
Advertencia de la conducta a seguir, ver § 1, a.

c) Cese de hostigamiento o acoso sexual10


Intimo plazo 48 hs. disponga medidas efectivas tendientes al cese del
hostigamiento sexual que vengo sufriendo en mis condiciones de trabajo,
11
consistente en .. llevadas a cabo por ...12 y que lesionan mi derecho a la
intimidad por la hostilidad e indignidad que ello trae a mi lugar y condiciones
de trabajo, haciéndome conocer dichas medidas mediante respuesta
fehaciente a la presente, bajo apercibimiento de ...13.

10
Ver Mansueti, Acoso sexual en las relaciones laborales, TSS,
2003-8.

11
Debe incluirse una descripción de los hechos configurativos del
acoso u hostigamiento, con connotación del tipo sexual y no aceptadas
por el destinatario (insultos, observaciones, requerimientos,
solicitudes o invitaciones reiteradas del tipo quid pro quo, exhibición
no tolerada de pornografía, entre otros).

12
A efectos que el empleador adopte medidas, siendo aplicables las
explicaciones incluidas (§ 10, b).
13
Advertencia de la conducta a seguir, ver § 1, a.

d) Requerimiento a adoptar medidas de seguridad, con suspensión


de tareas14
Ante vuestro incumplimiento de la adopción de medidas de seguridad
necesarias para prevenir los daños en mi salud, derivados de ... 15, los intimo
para que dentro del plazo de ... 16 adopten medidas mínimas de seguridad,
suministrando elementos de protección adecuados, bajo apercibimiento
de ...17. Hasta tanto provean condiciones seguras de labor, suspendo la
prestación de mis servicios a su costa18.

14
Referencias normativas. Art. 75, LCT, y ley 19.587.

15
Descripción de condiciones puntuales de trabajo que requieran
medidas concretas, tales como protectores por la exposición al ruido,
polvillo, agentes contaminantes, etcétera.

16
El plazo dependerá de las medidas que se requieran y será el
mínimo razonable en función de cada caso concreto.
17
Advertencia de la conducta a seguir, ver § 1, a. En caso de
incumplimiento grave y resistencia del empleador a modificar su
actitud, la situación puede ser configurativa de injuria y legitimar la
disolución del vínculo por despido indirecto (arts. 242 y 246, LCT).

18
Referencia normativa. Art. 1201 del Cód. Civil (exceptio non
adimpleti contractus).

Enfermedades y accidentes
a) Trabajador da aviso de enfermedad inculpable1
Notifico imposibilidad de prestar servicios por enfermedad inculpable,
consistente en ...2, hallándome en ...3 a disposición del control médico de su
facultativo4.

1
Referencia normativa. Art. 209, LCT; decr. 1028/10 (agrega al
Anexo I del decr. 150/96, el telefonograma en reemplazo optativo del
telegrama obrero, para casos de enfermedad o fallecimiento de
familiar directo).

2
El art. 209 de la LCT impone el deber al trabajador de dar aviso al
empleador “de la enfermedad o accidente”. Los deberes de
colaboración y buena fe imponen la carga de indicar cuál es la
enfermedad que ocasiona el impedimento.

3
El art. 209 de la LCT impone al trabajador el deber de indicar el
lugar donde se encuentra, para posibilitar el control médico del
empleador, que es facultativo y no obligatorio.

4
El trabajador no está obligado a justificar su enfermedad con
certificado médico, salvo en caso de falta de aviso o discrepancia con
el resultado del control médico del empleador (ver Mansueti, El
certificado médico con prescripción de reposo. Obligaciones a cargo
del trabajador, del empleador y de los médicos, LL, Suplemento
Actualidad, 27/7/10).

b) Notificación de accidente o enfermedad derivados del trabajo 5


Con motivo del accidente de trabajo sufrido el día ... en oportunidad de
hallarme en ejercicio de mis tareas habituales que me ocasionó ... 6, los intimo
para que dentro del plazo de 48 hs. Lo denuncien ante la ART por Uds.
contratada, haciéndome saber los datos necesarios para recabar el
otorgamiento de las prestaciones de ley, bajo apercibimiento de ... 7.

5
Referencia normativa. Art. 31, ley 24.557.

6
Descripción de las circunstancias del accidente o enfermedad
(art. 6o, ley 24.557) que hubiera provocado incapacidad para el
trabajo y requiriera las prestaciones del sistema (arts. 11 y 20, ley
24.557).

7
Advertencia de la conducta a seguir, ver § 1, a.

Sanciones disciplinarias y suspensiones

a) Empleador notifica sanción disciplinaria al trabajador


1) Apercibimiento
No habiendo justificado que con fecha ... en el horario de ... cuando ... Siendo
que Ud. está obligado a prestar sus servicios con puntualidad, asistencia
regular y dedicación adecuada, deberes que ha incumplido en el caso, nos
vemos obligados a aplicarle una sanción disciplinaria de apercibimiento, con
constancia en su legajo personal. Asimismo le hacemos saber que en caso de
producirse nuevos incumplimientos de su parte, nos veremos obligados a
aplicar sanciones disciplinarias de mayor severidad.

2) Suspensión1
Habiéndose advertido que Ud. con fecha ..., en el horario de ..., desobedeció
expresas instrucciones recibidas en el sentido que debía ... 2 y no lo hizo, lo
cual constituye un incumplimiento grave de su parte a los deberes que le
impone el art. 84 de la LCT, el que es reiteratorio de otros similares por los
cuales fue apercibido, nos vemos obligados a aplicarle una sanción disciplinaria
de suspensión por ... días 3, apercibimiento, con constancia en su legajo
personal. Asimismo le hacemos saber que en caso de producirse nuevos
incumplimientos de su parte, nos veremos obligados a aplicar sanciones
disciplinarias de mayor severidad.

1
La validez de toda suspensión queda condicionada al
cumplimiento de tres requisitos llamados comunes, que establece el
art. 218 de la LCT y son la justa causa (es el incumplimiento concreto
imputado, que debe estar explicado con claridad en la comunicación,
para posibilitar su conocimiento y defensa por parte del trabajador), el
plazo fijo (días ciertos y definidos en la comunicación para su
cumplimiento) y la notificación por escrito. Esta última no
necesariamente debe cumplirse con el envío de una carta documento o
telegrama. En caso que dicha diligencia se practique por nota, con
constancia de notificación por parte del trabajador, corresponde
reciba el doble ejemplar de ella (art. 1021, Cód. Civil).

2
La sanción disciplinaria debe contener una explicación clara de
los hechos configurativos de la justa causa analizada por el
empleador. También debe ser notificada con anterioridad al inicio de
la medida. Son requisitos vinculados con el ejercicio del derecho de
defensa del trabajador, los cuales se verían frustrados de no contar
con un plazo mínimo y razonable de notificación de la medida, antes
de su ejecución.

3
Por causas disciplinarias, el plazo no debe exceder los treinta
días dentro del año aniversario (art. 220, LCT). Los plazos deben
contarse en días corridos y continuados (arts. 27 a 29, Cód. Civil) por
cada suspensión, no correspondiendo su aplicación alternando días
hábiles con inhábiles, cuando ello tenga por objeto la prolongación de
los plazos máximos legalmente admitidos.

b) Trabajador rechaza sanción disciplinaria4


Rechazo "suspensión” ["apercibimiento”] notificado mediante su ... [“carta
documento”, “telegrama", “nota interna”] de fecha ... por ser falsa la causal
alegada y desproporcionada la medida. Niego que ... 5. A todo evento, aun si
hubieran sido ciertos los hechos imputados, la medida se muestra como
francamente desproporcionada en atención a ... Por lo expuesto, los intimo
para que dentro del plazo de 48 hs. dejen sin efecto la medida “o en su caso la
ajusten a un llamado de atención y provean tareas habituales, caso contrario
accionaré por salarios caídos por su culpa”.

4
El art. 67 de la LCT impone un plazo de caducidad de treinta días,
contados desde la notificación, para que el trabajador proceda a
cuestionar su procedencia y su tipo o extensión. Transcurrido dicho
plazo, el efecto previsto por la norma es que la sanción se tenga por
consentida. A su vez, el art. 223 de la LCT condiciona el derecho del
trabajador a percibir los salarios de suspensión, a que hubiera
impugnado la medida.

5
La negativa de los hechos invocados por el empleador tiene que
ver con la impugnación de la medida disciplinaria aplicada, la cual no
tendría justa causa cuando se apoya en hechos inexistentes. También
puede tratarse de hechos que en verdad hubieran sucedido, pero con
eximentes de responsabilidad a favor del trabajador o que hubieran
merecido algún tipo de sanción menos grave. El cues- tionamiento
pasa aquí por carecer la medida aplicada de la proporcionalidad que
exige el art. 67 de la LCT.

c) Empleador notifica suspensión por falta o disminución DE


TRABAJO6
En atención a la falta de trabajo que afecta a la empresa, debido a los hechos
por completo ajenos a nuestra voluntad que Ud. conoce y están dados por ... 7,
nos vemos obligados a suspenderlo en sus tareas por ... días, desde el ...
hasta el ...8, ambas fechas inclusive. Ponemos en su conocimiento que la firma
ha procedido al cumplimiento de los requisitos previstos por ... 9, dando
cumplimiento también al orden fijado en el art. 221 de la LCT.

6
Referencias normativas. Arts. 221, 223 y 223 bis, LCT; arts. 98 a
110, ley 24.013, y decr. 328/88.

7
La falta o disminución de trabajo que aquí se trata no debe
necesariamente revestir el carácter de imposibilidad de otorgar tareas
(lo que sí es el caso de la fuerza mayor), sino que el cumplimiento de
dicho deber a cargo del empleador se ha convertido en excesivamente
oneroso, por circunstancias no imputables a él o al riesgo empresario
a su cargo y que deben ser explicadas en la comunicación.

8
Por falta o disminución de trabajo no imputable al empleador, el
plazo no debe exceder los treinta días dentro del año aniversario (art.
220, LCT). Para el cómputo de los plazos, ver nota 106.

9
Según el caso, el procedimiento preventivo de crisis (art. 98 y
ss.; ley 24.013, cuando la medida afecte a un número mayor del
porcentaje de trabajadores allí requerido) o la reglamentación que
determina el decr. 328/88, que rige como norma general, cuando no
resultan aplicables las disposiciones del mecanismo anterior. Ambos
procesos son llevados a cabo ante la autoridad administrativa del
trabajo e intervienen el empleador y la o las asociaciones sindicales
que ejercen representación sobre los trabajadores involucrados con la
medida. El trabajador afectado no es parte necesaria. Podrá
cuestionar judicialmente la procedencia de la medida aplicada y, en
dicho contexto, las actuaciones llevadas a cabo en sede administrativa
(principalmente destinadas a la búsqueda de remedios negociados,
como ser el previsto en el art. 223 bis de la LCT), no atañen cosa
juzgada acerca del cumplimiento o no por parte del empleador de los
requisitos de procedencia de la suspensión.
d) Empleador notifica suspensión por fuerza mayor 10
Hallándonos imposibilitados para darle ocupación, debido a los hechos por
completo ajenos a nuestra voluntad que Ud. conoce y están dados por ...", nos
vemos obligados a suspenderlo en sus tareas por ... días, desde el ... hasta
el ...12, ambas fechas inclusive. Ponemos en su conocimiento que la firma ha
procedido al cumplimiento de los requisitos previstos por ... 13, dando
cumplimiento también al orden fijado en el art. 221 de la LCT.

10
Referencias normativas. Arts. 221, 223 y 223 bis, LCT; arts. 98 a
110, ley 24.013, y decr. 328/88.

11
La fuerza mayor que aquí se trata, que debe ser un hecho ajeno
tanto a la voluntad del empleador como al riesgo empresario a su
cargo, debe provocar la imposibilidad de otorgar tareas al trabajador.
El o los hechos que revisten tal carácter, deben ser explicados en la
comunicación.

12
Por fuerza mayor, el plazo no debe exceder los setenta y cinco
días dentro del año aniversario (art. 221, LCT). Para el cómputo de los
plazos, ver § 12, a, 1.

13
El procedimiento preventivo de crisis (art. 98 y ss., ley 24.013) o
la reglamentación que determina el decr. 328/88, según el caso.

e) Trabajador rechaza suspensión (por falta o disminución DE


TRABAJO O POR FUERZA MAYOR)
Rechazo suspensión notificada mediante su ... [“carta documento”,
“telegrama”, "nota interna”] de fecha ... por ser falsa la causal alegada y, a
todo evento, improcedente la medida. Niego que ... 14. A todo evento, aun si
hubieran sido ciertos los hechos invocados, éstos se corresponden con el
riesgo empresario a su cargo y no justifican la medida que pretenden aplicar.
Por lo expuesto, los intimo para que dentro del plazo de 48 hs. dejen sin
efecto la suspensión aplicada y provean tareas habituales, caso contrario
accionaré por salarios caídos por su culpa.

14
La negativa de los hechos invocados por el empleador o el
cuestionamiento de su entidad para justificar la medida, tiene que ver
con el cumplimiento del recaudo que exige el art. 223 de la LCT, para
que el trabajador luego se encuentre habilitado a reclamar los salarios
de suspensión. Aun cuando no se trata de una suspensión
disciplinaria, la falta de impugnación obsta el reclamo posterior de
salarios caídos. Es que el art. 223 de la LCT, cuando reglamenta el
derecho del trabajador a la percepción de los "salarios de
suspensión”, expresamente condiciona su viabilidad a los casos donde
“hubiere impugnado” la medida. Esta impugnación debe ser expresa y
constituye una carga que debe cumplir el trabajador si desea reclamar
los salarios por tiempo no trabajado y que estuvo a disposición del
empleador para hacerlo (arts. 103 y 197, LCT). Se trata de un
supuesto de "caducidad” de este derecho que, por hallarse
expresamente contemplado como recaudo para su ejercicio, es válida
(art. 259, LCT) como excepción al principio de irrenunciabilidad que
consagra su art. 12.

f) Trabajador notifica imposibilidad de prestar servicios POR


FUERZA MAYOR
Notifico que por razones de fuerza mayor, derivadas de ... 15, me encuentro
transitoriamente imposibilitado de prestar servicios a partir de la fecha. Una
vez cesado el impedimento, retomo actividad presentando los justificativos
pertinentes.

15
Por aplicación de las reglas generales (arts. 513 y 514, Cód.
Civil), la fuerza mayor o el caso fortuito que provocan impedimento
temporario a la prestación personal de tareas a cargo del trabajador,
son válidas para justificar su incumplimiento, cuando se trata de
hechos que la legislación o la negociación colectiva no han previsto
como causales de suspensión del contrato (licencias) y cuando, con
dicha justificación, se preserva la continuidad del contrato (art. 10,
LCT). Aun así, la justificación tendría entidad para impedir el ejercicio
del poder disciplinario del empleador, pero al no hallarse prevista
como supuesto de suspensión del contrato con derecho a percibir
haberes, éste no tendría la consecuente obligación de pagarlos. La
comunicación debiera explicar los hechos configurativos de la fuerza
mayor, tales como la necesidad de atender familiar cuando no existe
otro que lo haga, epidemia, incendio o calamidad climática, etcétera.

a) Trabajador/a notifica su matrimonio1


Notifico que con fecha ... contraeré matrimonio en ..., gozando de la licencia
especial por diez días corridos, a partir de ese día y hasta el inclusive,
retomando tareas el día ... en horario habitual, con el correspondiente
certificado que justifica el hecho.

1
Referencia normativa. Arts. 158, inc. b, y 181, LCT. A partir de
esta notificación, comienza a regir la presunción derivada del art. 181
y todo despido sin expresión de causa o con causa no acreditada,
producido dentro de los tres meses anteriores o dieciocho posteriores
al matrimonio, se considera ocasionado por éste y rige la
indemnización especial (que se acumula a las comunes) del art. 182
de la LCT que es un monto equivalente a un año de remuneraciones
(doce meses más el SAC). Esta indemnización también procede en los
casos de despido indirecto (CNTrab, Sala X, 29/9/00, “Cedeira, Nancy
c/Edenor SA y otro”, LL, 2001-D-841). La jurisprudencia de la Capital
Federal sigue los lineamientos del plenario 272 (CNTrab, en pleno,
23/3/90, “Drewes, Luis A. c/Coselec SSCS s/cobro de pesos”, DT,
1990-A-893, donde aprobó la siguiente doctrina legal: “En caso de
acreditarse que el despido del trabajador varón obedece a causas de
matrimonio, es procedente la indemnización prevista en el art. 182,
LCT”) y se inclina por asignarle al art. 181 de la LCT un alcance
restringido a la trabajadora mujer, dado que se encuentra inserto en
el título VII de la LCT referido al trabajo de dicho género. Así, se ha
resuelto: “La presunción de despido por causa de matrimonio a la que
alude el art. 181 de la ley de contrato de trabajo, sólo rige respecto
del personal femenino, sin perjuicio de la posibilidad de que el
trabajador varón acredite que su despido obedeció a esa causa"
(CNTrab, Sala II, 26/3/09, "Pezzelatto, Ricardo O. c/Gemmo América
SA y otro", DT, 2009-657). No es el criterio seguido en la Provincia de
Buenos Aires, habiendo resuelto la SCBA que “la condición de varón es
irrelevante para la procedencia de la indemnización del art. 182 por
despido por causa de matrimonio” (20/12/00, “Pizzonia, Miguel A.
c/Cruppi SA”, DT, 2001-B- 1939). De todos modos se trata de una
presunción del tipo inris tantum y admite la prueba en contrario, que
estaría dada por la acreditación del incumplimiento grave o injuria
invocado por el empleador en la comunicación rescisoria (SCBA,
3/11/92, “Gallardo, Hilda E. c/Frigorífico Minguillón SA”, LL, 1993-A-
345).

b) Trabajador/a notifica ejercicio de derecho de acumular LA


LICENCIA ANUAL CON LICENCIA ESPECIAL POR MATRIMONIO2
Dado que con fecha ... contraeré matrimonio, gozando a partir de allí la
licencia especial. Notifico que opto por gozar la licencia anual ordinaria del
período ...3 por ... días, a partir del vencimiento de la licencia especial.

2
Referencia normativa. Art. 164, LCT.

3
Período de goce o año al que corresponden las vacaciones (art.
151, LCT).

c) Trabajadora notifica al empleador su estado de embarazo4


A todos los efectos legales, comunico mi estado de embarazo con fecha
probable de parto el día ... de ... de ... Certificado médico y control facultativo
a su disposición.

4
Referencia normativa. Arts. 177 a 179, LCT. A partir de esta
notificación, comienza a regir la presunción derivada del art. 181 de la
LCT y todo despido sin expresión de causa o con causa no acreditada,
producido dentro de los siete meses y medio anteriores o posteriores
al parto, se considera ocasionado por la maternidad y rige la
indemnización especial (que se acumula a las comunes) del art. 182
de la LCT que es un monto equivalente a un año de remuneraciones
(doce meses más el SAC). Esta indemnización también procede en los
casos de despido indirecto (CNTrab, Sala II, 8/6/10, “Guerra, María
del V. c/Universidad del Salvador Asoc. Civil”, LLonline,
AR/JUR/31674/2010). La presunción es del tipo iuris tantum y cede
ante la acreditación del empleador a que el despido se produjo por
otros motivos, como ser el abandono de trabajo (CNTrab, Sala IV,
16/7/10, "Rositto, Silvina C. c/Koltan, Silvio E.”, LLonline,
AR/JUR/42655/2010) o como consecuencia de una medida de alcance
general consistente en el traslado del establecimiento (CNTrab, Sala
VI, 28/9/06, "Fernández Prats, Carolina c/UPS de Argentina SA”,
LLonline, AR/JUR/6545/2006).

d) Trabajadora notifica a su empleador, el ejercicio de la opción


POR RESCINDIR EL CONTRATO, CON DERECHO A COMPENSACIÓN ESPECIAL 5
Hallándome comprendida en las disposiciones del art. 183 de la LCT por haber
dado a luz el día ..., notifico a Uds. mi opción por rescindir el contrato a partir
del día ...6, solicitando el pago de la compensación especial allí prevista,
juntamente con la liquidación final de haberes pendientes.

5
Referencia normativa. Art. 183, inc. b, LCT.

6
Fecha de reinicio de actividades, luego de finalizada la
suspensión del contrato por los noventa días de maternidad (art. 177,
LCT).

e) Trabajadora notifica a su empleador, el ejercicio de la opción


POR EL ESTADO DE EXCEDENCIA7
Hallándome comprendida en las disposiciones del art. 183 de la LCT por haber
dado a luz el día ..., notifico a Uds. que haré uso de la situación de excedencia
prevista por el inc. c, de dicha norma, por el período de ... meses, contados
desde el día ..., reintegrándome a mis tareas el día ... en mi horario habitual.

7
Referencia normativa. Arts. 183, inc. c, LCT.

Tutela sindical1

a) Notificación al empleador de convocatoria a elecciones (por


asociación sindical)
Tenemos el agrado de dirigirnos a Uds., con motivo de haber convocado esta
asociación sindical elecciones para la renovación de los cargos de con un
mandato de ... años, a partir del día ... El comicio se llevará a cabo el día ... en
horario de ... a ... hs. Sin otro particular y contando desde ya con vuestra
colaboración, saludamos a Uds. muy atentamente.

1
Referencias normativas. Arts. 47 a 52, ley 23.551; arts. 29 y 30,
decr. 467/88, y convenios OIT 87, 98 y 135.
b) Notificación al empleador de postulación como candidato
Notifico mi postulación como candidato, para el cargo de … 2 en las elecciones
convocadas por mi asociación sindical ...3 para el día ...4.

2
El régimen de tutela sindical regulado por los arts. 47 a 52 de la
LAS, comprende una tutela genérica en su art. 47 y los siguientes se
ocupan del régimen de tutela específica para los trabajadores que
ocupen cargos representativos, esto es, que surjan de procesos
electorales, en asociaciones sindicales con personería gremial. Estos
cargos pueden ser de representación en el establecimiento (delegado,
miembro de comisión interna) o en el órgano directivo o deliberativo
(delegado congresal) de la misma asociación. El art. 50 de la LAS
permite al trabajador que se postula a uno de estos cargos y todavía
no ha sido oficializado como candidato por la autoridad electoral
sindical, ingresar al régimen de tutela especial, notificando su
postulación al empleador. Debe indicar aquí el cargo para el cual se
postula.

3 El régimen de tutela especial diseñado por la LAS exige que se


trate de asociación sindical con personería gremial. La Corte Suprema
ha interpretado que los representantes de asociaciones menos
representativas o con simple inscripción gozan de igual protección
(CSJN, 9/12/09, “Rossi, Adriana M. c/Estado Nacional, Armada
Argentina”, LL, 2010-A-208, y DT, 2010-61).

4
Los procesos electorales destinados a la renovación de cargos
representativos en asociaciones sindicales son convocados por los
órganos a los cuales el estatuto respectivo le atribuye esa
competencia y fijan la fecha del comicio. La comunicación cursada por
el postulante requiere que haya comicio convocado.
c) Notificación al empleador de oficialización como candidato
1) Cursada por la asociación sindical al empleador
Tenemos el agrado de dirigirnos a Uds. con motivo de llevarse a cabo el día ...
de ... próximo las elecciones para la renovación de cargos de ... en el
establecimiento habilitado a su nombre. Tal como se les notificara
oportunamente, el mandato a renovar es por el período de ... años 5 a partir
del ... y hasta el ... Se han oficializado candidatos de dos listas y en
condiciones estatutarias, integrada la lista n° ... por los Cros. ... (DNI ...),
candidato a ..., y ... (DNI ...) como candidato a ..., y la lista n° ... se integra
con los ... (DNI ...), candidato a ..., y ... (DNI ...) como candidato a ... Para el
mejor desenvolvimiento del comicio solicitamos a Uds. tengan a bien facilitar a
esta asociación un lugar en su establecimiento para que funcione la urna
respectiva, procurando en todo momento no entorpecer la labor productiva del
establecimiento y llevar a cabo tan trascendente acto en el menor tiempo
posible. Nos permitimos recordarles que conforme lo dispone el art. 44 de la
ley 23.551, la empresa debe facilitar un lugar para el desarrollo de las tareas
de delegados de personal y tal deber para nosotros es de suma importancia el
día del comicio, puesto que, de lo contrario, nos veremos obligados a colocar
la urna y llevar a cabo el acto en la puerta de acceso a la planta. Sin otro
particular y contando desde ya con vuestra colaboración, saludamos a Uds.
muy atentamente.

5
Como máximo dos años para los representantes de los
trabajadores en el establecimiento (art. 42, LAS) y cuatro para el
órgano de dirección y administración (art. 17, LAS).

2) Cursada por el trabajador al empleador


Notifico mi oficialización en condiciones estatutarias como candidato, para el
cargo de ...6 en las elecciones convocadas por mi asociación sindical ... 7 para el
día ...8.
6
El régimen de tutela sindical regulado por los art. 47 a 52 de la
LAS, comprende una protección genérica en su art. 47 y los siguientes
se ocupan del régimen de tutela específica para los trabajadores que
ocupen cargos representativos, esto es, que surjan de procesos
electorales, en asociaciones sindicales con personería gremial. Estos
cargos pueden ser de representación en el establecimiento (delegado,
miembro de comisión interna) o en el órgano directivo o deliberativo
(delegado congresal) de la misma asociación. El art. 50 de la LAS
permite al trabajador que se postula a uno de estos cargos y todavía
no ha sido oficializado como candidato por la autoridad electoral
sindical, ingresar al régimen de tutela especial, notificando su
postulación al empleador. Debe indicar aquí el cargo para el cual se
postula.

7
El régimen de tutela especial diseñado por la LAS exige que se
trate de asociación sindical con personería gremial. La Corte Suprema
ha interpretado que los representantes de asociaciones menos
representativas o con simple inscripción, gozan de igual protección
(ver § 14, b).

8
Los procesos electorales, destinados a la renovación de cargos
representativos en asociaciones sindicales, son convocados por los
órganos a los cuales el estatuto respectivo le atribuye esa
competencia y fijan la fecha del comicio. La comunicación cursada por
el postulante requiere que haya comicio convocado.

d) Notificación de designación como representante sindical

1) Cursada por la asociación sindical al empleador


Tenemos el agrado de dirigirnos a Uds. con motivo de haberse llevado a cabo
el día ... de ... las elecciones para la renovación de cargos de ... en el
establecimiento habilitado a su nombre. El proceso electoral fue llevado a cabo
en condiciones estatutarias y con normalidad. Como resultado de éste, fueron
designados los Cros. ... (DNI ...) y ... (DNI ...) en los cargos de ... y ...,
respectivamente, con mandato por ... años, desde el ... y hasta el ... Sin otro
particular y contando desde ya con vuestra colaboración, saludamos a Uds.
muy atentamente.

2) Cursada por el trabajador al empleador


Notifico que en el acto electoral llevado a cabo el día ... fui designado en el
cargo de ... con mandato por ... años, desde el ... y hasta el ...

e) Motivadas por infracción al régimen especial de protección 9

1) Despido directo o suspensión, intimando a la reincorporación


bajo apercibimiento de acción sumarísima
Rechazo "despido” “suspensión” notificado en su carta documento ... por
importar dicha medida una violación al régimen de tutela sindical que me
ampara, dado que “ejerzo el cargo electivo de ...” “revisto la condición de"
“postulante” “cantidato”10 con mandato “vigente hasta el ...” (o “cumplido el ...
y por lo tanto dentro del plazo de estabilidad posterior”), siendo por lo tanto
nula. En consecuencia, los intimo para que dentro del plazo de 48 hs. la dejen
sin efecto y me repongan en mis tareas habituales, bajo apercibimiento de
promover para ello la respectiva acción judicial a su costa.

9
Referencia normativa. Art. 52, LAS. Se plantean los tres
supuestos previstos por la norma, a saber: el despido, la suspensión o
la modificación de las condiciones de trabajo del representante
sindical, sin haberse seguido el proceso previo de desafuero. Se
incluyen apercibimientos vinculados a las versiones vinculadas a la
doble opción del representante habilitadas por la norma: considerarse
despedido o promover la acción sumarísima.
10
Debe tratarse de alguno de los cargos de origen electivo,
previstos por la LAS en sus arts. 40 (delegados de personal,
comisiones internas y organismos similares), 48 (representantes en
órganos directivos o deliberativos de asociaciones sindicales con
personería gremial) y 50 (postulante o candidato oficializado, por los
seis meses posteriores a la notificación al empleador).
2) Despido directo o suspensión, intimando al cese bajo
apercibimiento de despido
Rechazo “despido” “suspensión” [se sigue la primera parte del anterior]. En
consecuencia, los intimo para que dentro del plazo de 48 hs. dejen sin efecto y
me repongan en mis tareas habituales, bajo apercibimiento de considerarme
despedido por su exclusiva culpa, reclamando las indemnizaciones comunes
más las especiales por infracción a la tutela sindical.

3) Justificada con modificación de condiciones de trabajo, con


apercibimiento de acción sumarísima
Rechazo su pretensión a modificar mis condiciones habituales de trabajo en ...
[lugar, sector u horarios], requiriendo que pase a desempeñarme en ...
[detalle de nuevas condiciones], por importar dicha medida una violación al
régimen de tutela sindical que me ampara, dado que “ejerzo el cargo electivo
de ..." ["revisto la condición de" “postulante", “candidato”] con mandato
"vigente hasta el ..." [o “cumplido el ... y por lo tanto dentro del plazo de
estabilidad posterior”], siendo por lo tanto nula. En consecuencia, los intimo
para que dentro del plazo de 48 hs. la dejen sin efecto y me repongan en mis
tareas habituales, bajo apercibimiento de promover para ello la respectiva
acción judicial a su costa.

4) Justificada con modificación de condiciones de trabajo, con


apercibimiento de despido
Rechazo su pretensión de modificar [se sigue la primera parte del anterior]. En
consecuencia, los intimo para que dentro del plazo de 48 hs. la dejen sin
efecto y me repongan en mis tareas habituales, bajo apercibimiento de
considerarme despedido por su exclusiva culpa, reclamando las
indemnizaciones comunes más las especiales por infracción a la tutela sindical.
C) Extinción del contrato

Causas que atañen al trabajador

a) Renuncia al empleo1 con preaviso


Preaviso renuncia al empleo el día Durante dicho lapso, gozaré la licencia
diaria del siguiente modo: ...3.

b) Renuncia al empleo4 sin preaviso


Renuncio al empleo a partir del día de la fecha5.

1
Referencia normativa. Art. 240, LCT.

2
El preaviso para el trabajador es siempre de quince días (art.
231, inc. a, LCT).

3
Se trata de la licencia diaria especial de dos horas por día, al
inicio o a la terminación de la jornada o bien que pueden ser
acumuladas en uno o más días en la semana (art. 237, LCT).

4
Referencia normativa. Art. 240, LCT.
La obligación de preavisar la terminación voluntaria del contrato de
trabajo está impuesta también para el trabajador por el art. 231 de la
LCT. Del incumplimiento de esta obligación debiera seguirse como
consecuencia, el deber de indemnizar a la otra parte, pagando un
importe equivalente a la remuneración por el plazo del preaviso, esto
es, de quince días (art. 232). Sin embargo, cuando es el trabajador
quien omite el preaviso, de acuerdo al régimen de responsabilidades
previsto por la LCT, la indemnización sustitutiva debiera estar a su
cargo sólo si el daño provocado con ello al empleador fue del tipo
intencional (art. 135). En ningún caso el empleador está habilitado
para pretender compensar este rubro con los debidos al trabajador
por liquidación final o salarios (art. 131), contando con un plazo de
caducidad de noventa días para la acción por daños y perjuicios
derivados de la omisión del preaviso.

5
Referencia jurisprudencial. "La ley de contrato de trabajo
dispone en el art. 240 que el trabajador puede disolver el vínculo
mediante despacho telegráfico colacionado cursado personalmente a
su empleador o ante autoridad administrativa. Como todo acto
jurídico, su validez está supeditada a que la voluntad del trabajador
no esté viciada por error, dolo, violencia, intimidación, simulación o
fraude. Por ello si se pretende impugnar la validez de la renuncia, es
preciso demostrar la existencia de alguno de los vicios de la voluntad
que tornan anulable el acto jurídico” (CNTrab, Sala III, 30/6/10,
“Cabrera, Rito C. c/Ramón Chozas SA", DTonline,
AR/JUR/45906/2010). "Corresponde declarar la nulidad del acto de
renuncia al empleo que formalizó un trabajador, toda vez que se
encuentra acreditado el grave y delicado estado que atravesaba su
salud mental al momento en que remitió su comunicación extintiva y
que dicha gravedad era suficiente como para afectar su
discernimiento” (id., Sala V, 31/8/09, "M., F. L. c/Soda Álvarez Hnos.
e Hijos SRL s/despido", LLonline, AR/JUR/29434/2009).

Causas que atañen al empleador

a) Despido sin justa causa o sin expresión de causa en período


DE PRUEBA1

1) Con preaviso2
Preavisamos la terminación del contrato de trabajo a prueba, a partir del
día ...3. Haberes, liquidación final y certificados estarán a su disposición en los
plazos legales4.
1
Referencia normativa. Art. 92 bis, LCT.

2
Referencia normativa. Arts. 231 a 239, LCT. El preaviso debe ser
notificado al trabajador por medio que sólo puede probarse por escrito
(art. 235, LCT), con una anticipación que depende de su antigüedad en
el empleo y está determinada por el art. 231 de la LCT. En período de
prueba será de quince días. Cumplido el período de prueba, este plazo
de preaviso omitido comienzan a correr a partir del primer día del mes
siguiente al de la notificación y será de un mes si su antigüedad no
excede los cinco años y de dos meses cuando fuere superior. En el
régimen especial para la pequeña empresa, el preaviso será siempre
de un mes, plazo que comienza a correr a partir del día siguiente al de
la notificación (ley 24.467, art. 95).

3
La notificación debe cursarse con quince días de anticipación
(art. 231, inc. b, LCT), que comienzan a correr a partir del día
siguiente.

4
Los plazos legales para el pago de remuneraciones por los días
trabajados (incluidos los del preaviso), SAC e indemnización por
vacaciones no gozadas, son los determinados en los arts. 128, 149 y
255 bis de la LCT. Los certificados y constancias deben ser entregados
cuando el trabajador “lo requiriese a la época de la extinción de la
relación” (art. 80, LCT). La norma fija un plazo de cumplimiento para
la obligación del empleador, en dos días hábiles contados desde la
recepción del requerimiento que a tal efecto formulare el trabajador.
La reglamentación (decr. 146/01, art. 3°), amplía dicho plazo, al
exigir que el trabajador aguarde treinta días corridos, desde la
extinción del vínculo, para cursar la intimación prevista en la norma
que se pretende reglamentar.
2) Sin preaviso
Comunicamos la terminación del contrato de trabajo a prueba que nos vincula,
a partir del día de la fecha. Haberes, liquidación final, indemnización
sustitutiva del preaviso5 y certificados estarán a su disposición en los plazos
legales6.

5
En este supuesto de omisión de preaviso, el empleador debe
pagar una indemnización sustitutiva (art. 232) cuyo valor es
equivalente a la remuneración que correspondería al trabajador
durante los plazos del art. 231 de la LCT. En el caso, la indemnización
estaría dada por el valor de quince días de salario, con más la
incidencia del SAC. Como se trata de una indemnización, no genera los
aportes y contribuciones que tienen por causa el salario o
remuneración.

6
Por plazos legales, ver nota 4. En este caso, habiéndose omitido
el otorgamiento del preaviso, que en período de prueba es por quince
días contados desde el día siguiente al de la notificación (art. 231,
LCT), corresponde incluir la indemnización sustitutiva del art. 232. El
plazo legal para el pago de esta indemnización será también el
determinado por los arts. 128, 149 y 255 bis de la LCT.

b) Despido finalizado el período de prueba (directo y sin justa


CAUSA, EN CONTRATO A PLAZO INDETERMINADO ESTABLE) 7

7
Referencia normativa. Arts. 243 y 245, LCT.
En nuestra interpretación, el despido sin expresión de causa
constituye un acto ilícito, porque de éste se sigue como mínimo la
obligación de indemnizar (art. 1066, Cód. Civil). Frente a la libertad de
contratación del empleador, que habitualmente se esgrime como
fundamento de una supuesta libertad para despedir, se encuentra el
derecho humano fundamental de toda persona “a ganarse la vida
mediante un trabajo libremente escogido o aceptado” (art. 6o. 1,
PIDESyC). Aun cuando nuestro país no ratificó el convenio OIT 158
sobre la terminación de la relación de trabajo por iniciativa del
empleador (Conferencia 68, Ginebra, 22/6/82; entrada en vigor el
23/11/85), entendemos que la compatibilización de ambas libertades
exige la preservación del derecho del trabajador a conocer los motivos
concretos por los cuales el empleador decide privarlo de su derecho a
trabajar. Ello con independencia del mecanismo de reparación
patrimonial que se adopte, como protección contra el despido
arbitrario. La expresión de causa, aun seguida del pago de una
indemnización, permite al trabajador conocer los motivos por los
cuales pierde su empleo. Aun cuando pueda no aceptarlos,
prevalecerán los poderes de dirección del empleador y su libertad de
contratación. Ello así, salvo en los casos donde tales motivos reciben
otras limitaciones del ordenamiento jurídico, sea por la protección
especial a la estabilidad del empleo (público, representantes
sindicales, maternidad, entre otros) o por el reproche al móvil del
despido (discriminatorio, antisindical, etc.) con la consecuente nulidad
del acto.

1) Con preaviso8
Preavisamos su despido a partir del día ... 9. Haberes, indemnización por
antigüedad10, liquidación final y certificados estarán a su disposición en los
plazos legales11.

8
Ver § 16, a, 1, notas 3 y 4.

9
Transcurrido el período de prueba, los plazos del preaviso,
cuando es otorgado conforme al texto sugerido, comienzan a correr a
partir del día siguiente al de su notificación. En caso de preaviso no
otorgado, esto es, cuando procede la indemnización sustitutiva, los
plazos comienzan a correr a partir del primer día del mes siguiente a
la notificación (art. 233, LCT).

10
La indemnización por antigüedad será la prevista en el art. 245
de la LCT.

11
Por plazos legales, ver § 16, 1. Para la indemnización por
antigüedad o despido rigen también los plazos previstos en los arts.
128, 149 y 255 bis de la LCT y se computan a partir del momento de la
notificación.

2) Sin preaviso
Comunicamos su despido a partir del día de la fecha 12. Haberes, liquidación
final, indemnización por antigüedad, sustitutiva del preaviso 13 y certificados
estarán a su disposición en los plazos legales14.

12
Sólo por razones de estilo, ya que no están exigidas de manera
expresa por nuestro ordenamiento, a veces se antecede este párrafo
con "Por razones de reestructuración ”.

13
Ver § 16, a, 1. En la práctica, la mayoría de los textos prescinde
de la referencia puntual a cada una de las indemnizaciones que serán
liquidadas con motivo del despido. Su agregado en el texto sugerido
sólo se justifica para facilitar su interpretación. En este supuesto, el
valor de la indemnización sustitutiva del preaviso (transcurrido el
período de prueba) será el equivalente a uno o dos meses de salario
(con incidencia del SAC), de acuerdo a la antigüedad del trabajador
(hasta o más de cinco años, respectivamente) y lo determinado por el
art. 231 de la LCT. Cuando la notificación del preaviso no tuviere lugar
el último día de un mes, corresponderá adicionar la indemnización por
integración del mes del despido, que estará dada por el valor de los
salarios devengados desde el día siguiente al de la notificación y hasta
la finalización del mes (art. 233, LCT). Tanto el preaviso omitido como
la integración del mes del despido tienen carácter indemnizatorio; por
lo tanto, son rubros que no constituyen base de cálculo para
contribuciones y aportes al sistema de seguridad social.

14
Por plazos legales, ver § 14, e, nota 8.

3) Con expresión de causa y para ser indemnizado15


Por razones de ... “cierre del establecimiento” “no haber alcanzado Ud. el
cumplimiento de los objetivos de producción esperados por nosotros” “no
haber aceptado Ud. el cambio implementado en nuestro proceso productivo"
[“el ostensible bajo rendimiento que viene evidenciando para el cumplimiento
de sus labores en los últimos tiempos”] es que habiendo agotado nuestras
posibilidades para continuar la relación laboral, nos vemos obligados a
comunicarle su despido a partir del día de la fecha. Haberes, liquidación final,
indemnización por antigüedad, sustitutiva del preaviso y certificados estarán a
su disposición en los plazos legales16.

15
Por las razones expuestas en § 16, b, el texto sugerido permite al
trabajador conocer los motivos del despido. Puede ser de utilidad en
los casos donde hubiera sospecha de conducta discriminatoria en la
decisión del empleador (ley 23.592), la cual quedaría desvirtuada con
la expresión de las razones objetivas que impulsaron su determinación
y que pudieran llegar a acreditarse en juicio.

16
Ver § 16, 1, a y b.

4) Por hallarse el trabajador en condiciones de jubilarse17


Hallándose Ud. en condiciones de obtener las prestaciones previsionales
determinadas por la ley 24.241, cumplimos en intimarlo para que inicie los
trámites pertinentes, colocando a su disposición los certificados de servicios y
demás documentación necesaria. Conforme a la legislación vigente, su relación
laboral con nosotros quedará concluida con la obtención del beneficio, dentro
del plazo máximo de un año a partir de la fecha, sirviendo la presente de
suficiente preaviso.

17
Referencias normativas. Art. 252, LCT, y art. 19 y ss., ley 24.241.
La eficacia de esta intimación quedó virtualmente desarticulada en
jurisdicción de la Ciudad de Buenos Aires con la doctrina legal
aprobada por la CNTrab (5/6/09, en pleno, “Couto de Capa”), donde
resolvió: "Es aplicable lo dispuesto por el art. 253, último párrafo, LCT
al caso de un trabajador que sigue prestando servicios sin
interrupción a las órdenes del mismo empleador, luego del goce del
beneficio de la jubilación". Consideramos que dicha doctrina es
equivocada, ya que pulveriza el alcance de la intimación prevista en el
art. 252 de la LCT. Según ella, el empleador queda eximido de cumplir
con la carga de cursar esta intimación. Aun sin ella, la antigüedad del
trabajador concluye con la obtención de un beneficio previsional y se
agrega, de ese modo anómalo, una caducidad de derechos que no fue
expresamente prevista por el legislador, contraria a lo dispuesto por el
art. 259 de la LCT. Si la estabilidad en el empleo, regulada con el
contrato a plazo indeterminado, concluyera de manera automática con
la obtención del beneficio previsional (interpretación del art. 91 de la
LCT en el sentido más perjudicial para el trabajador), no tendría
sentido el art. 252 de la LCT, en cuanto otorga al empleador la facultad
(“podrá” dice su texto) de extinguir el contrato por esta vía,
eximiéndose del pago de las indemnizaciones pertinentes. A mayor
abundamiento, la LCT, en su art. 18, define el cómputo de la
antigüedad en el empleo y no ha previsto la caducidad o finalización
que interpreta la Cámara en el plenario citado.

c) Rechazo del trabajador, por no reunir requisitos


Contestando la intimación que me cursara por carta documento n° ..., debo
rechazarla, puesto que no es cierto que me encuentre en condiciones de
obtener las prestaciones previsionales que Uds. indican. En efecto, ... "aun
contando con la edad necesaria, los años de aportes al sistema no cubren la
cantidad legal requerida"18.

18
Corresponde aquí indicar las causas por las cuales el trabajador
no está aún en condiciones de acceder al beneficio previsional. Para el
texto se ha propuesto la más común de ellas.

d) Rechazo de la trabajadora, por ejercer opción de continuar


EN LA ACTIVIDAD HASTA LOS 65 AÑOS DE EDAD
Contestando la intimación que me cursara por "carta documento n° ...", debo
rechazarla, puesto que ejerzo la opción conferida a mi favor por el art. 19 de la
ley 24.241, en el sentido de continuar con mi actividad laboral hasta los 65
años de edad.

e) Por falta o disminución de trabajo19


En atención a la falta de trabajo que afecta a la empresa, debido a los hechos
por completo ajenos a nuestra voluntad que Ud. conoce y están dados
por ...20, nos vemos obligados a prescindir de sus servicios a partir del día de
la fecha. Haberes pendientes por liquidación final, indemnización sustitutiva
del preaviso, la determinada por el art. 247 de la LCT y certificados de trabajo
a su disposición en los plazos legales.

19
Referencias normativas. Art. 247, LCT; arts. 98 a 110, ley 24.013,
y decr. 328/88.

20
La falta o disminución de trabajo no debe necesariamente
revestir el carácter de imposibilidad de otorgar tareas (lo que sí es el
caso de la fuerza mayor), sino que el cumplimiento de dicho deber a
cargo del empleador se ha convertido en excesivamente oneroso, por
circunstancias no imputables a él o al riesgo empresario a su cargo y
que deben ser explicadas en la comunicación.
f) Por fuerza mayor21
Hallándonos imposibilitados para darle ocupación, debido a los hechos por
completo ajenos a nuestra voluntad que Ud. conoce y están dados por ... 22,
nos vemos obligados a prescindir de sus servicios a partir del día de la fecha.
Haberes pendientes por liquidación final, indemnización prevista por el art. 247
de la LCT y certificados de trabajo a su disposición en los plazos legales.

21
Referencias normativas. Art. 247, LCT; arts. 98 a 110, ley 24.013,
y decr. 328/88.

22
La fuerza mayor que aquí se trata, que debe ser un hecho ajeno
tanto a la voluntad del empleador como al riesgo empresario a su
cargo, debe provocar la imposibilidad de otorgar tareas al trabajador.
El o los hechos que revisten tal carácter deben ser explicados en la
comunicación.

g) Por incapacidad física o mental del trabajador 23


Habiéndose verificado por parte de “la Comisión Médica .... en expte. ...",
["nuestro centro de medicina laboral ...”] que Ud. se encuentra incapacitado
en forma absoluta para trabajar y de manera permanente 24, notificamos la
extinción del contrato de trabajo que nos vincula por tal motivo a partir del día
de la fecha. Haberes pendientes por liquidación final, indemnización prevista
por los arts. 212 y 254 de la LCT y certificados de trabajo a su disposición en
los plazos legales.

23
Referencia normativa. Arts. 212 y 254, LCT.

24
Referencia jurisprudencial. "Procede hacer lugar a la demanda
promovida por un trabajador que por encontrarse incapacitado
parcialmente, fue despedido en los términos del art. 212, 2o párrafo,
ley de contrato de trabajo, pues, su empleador no cumplió con la carga
de probar la inexistencia de tareas livianas que pudieran serle
otorgadas y por el contrario se probó que tenía la posibilidad táctica
de reincorporarlo pero careció de voluntad de otorgarle aquellas
tareas, circunstancia que torna asimilable la situación a la del despido
arbitrario o incausado" (CNTrab, Sala H, 12/6/09, "Semino, Gladys N.
c/Toso SRL”, DT, 2009-B-91).

h) Por pérdida de habilitación especial (inhabilidad)


PARA EL EJERCICIO DE LA ACTIVIDAD CONTRATADA25

25
Referencia normativa. Arts. 242 y 254, LCT.

1) Pérdida de habilitación por motivos involuntarios del trabajador 26


Habiendo Ud. perdido la habilitación especial para ... que hace al objeto de
nuestra vinculación y no contando en nuestra firma con otro tipo de tareas
para asignarle27, notificamos la extinción del contrato de trabajo que nos
vincula por tal motivo a partir del día de la fecha. Haberes pendientes por
liquidación final, indemnización prevista por los arts. 247 y 254 de la LCT y
certificados de trabajo a su disposición en los plazos legales.

26
Referencia jurisprudencial. "Es aplicable lo dispuesto en la
primera parte del art. 254 de la LCT a los casos de pérdida de
habilitación especial contemplado en el segundo supuesto del mismo
artículo, cuando tal inhabilitación se origina en enfermedad o
disminución psicofísica contraída sin dolo o culpa grave del mismo
trabajador” (CNTrab, en pleno, n° 303, 3/5/02, "Palloni, Mariela H.
c/De- pormed SA s/despido”, TSS, 2001-1063).

27
Referencia jurisprudencial. "Es improcedente el despido por
inhabilidad sobreviniente de un trabajador cuando se funda en el
mismo impeditivo de prestar servicios que originara la licencia por
enfermedad inculpable del art. 208 de la ley 20.744 -en el caso,
adicción a las drogas-, pues el hecho de que haya sido considerado
inepto para el desempeño de sus tareas habituales -chofer de
colectivo- no implica necesariamente que deba ser despedido, sino
simplemente la imposibilidad de otorgarle dichas tareas durante cierto
período” (CNTrab, Sala II, 8/2/00, “T, C. D. c/Empresarios Transporte
Automotor de Pasajeros SA”, LL, 2001-C-755).

2) Pérdida de habilitación por dolo o culpa grave e inexcusable del


trabajador
Dado que con motivo de ...28, Ud. ha perdido la habilitación especial para ...
que hace al objeto de nuestra vinculación, quedando demostrado con ello que
la pérdida de habilitación fue provocada "de manera intencional por Ud.” ["por
su culpa grave e inexcusable”] consistente en nos vemos obligados a
despedirlo a partir del día de la fecha, del modo como nos habilita a hacerlo el
art. 254 de la LCT y en atención a que los motivos por los cuales ha perdido la
habilitación son sólo a Ud. imputables. Haberes pendientes por liquidación final
y certificados de trabajo a su disposición en los plazos legales.

28
Como toda comunicación rescisoria, el texto debe cumplir con las
exigencias del art. 243 de la LCT y explicar con claridad y autonomía
conceptual los motivos en los cuales se funda la ruptura del vínculo.
Luego de cursada esta notificación no podrán alterarse sus términos
(modificar la causal originaria por otra o ampliarla con nuevos
hechos), ya que ello infringiría el derecho de defensa del trabajador.
Es lo que se conoce como invariabilidad de la causal de despido,
recibida en este mismo art. 243 de la LCT. En el caso, tanto los hechos
provocados por el trabajador para perder su habilitación, como la
referencia a la resolución, dictamen o acto administrativo de la
entidad habilitante que así lo dispuso.

i) Rechazo del trabajador (a despidos fundados en causas DE


EXCEPCIÓN O ART. 247, LCT)
Rechazo despido notificado su "carta documento n° CD ...” ["telegrama
del ...”] por ser falsa la causal alegada. Niego haya existido cualquier tipo de
“falta o disminución de trabajo" “fuerza mayor" no imputable a Uds. que
justifique la medida, mucho menos cuando de su comunicación no surgen los
presupuestos que viabilizan tal modo de extinción ni las medidas que pudieron
haber adoptado para evitar mi despido. "A todo evento, confirma ello el hecho
que no hubieran iniciado ningún procedimiento preventivo de crisis o el exigido
por el decr. 328/88”. Por lo expuesto, siendo arbitrario y carente de justa
causa el despido, los intimo para que dentro del plazo legal paguen las
indemnizaciones derivadas del despido (integración del mes del despido -art.
233, LCT-, indemnización sustitutiva del preaviso -art. 232, LCT- e
indemnización por antigüedad -art. 245-), caso contrario accionaré sin más
trámite ni intimación, tornándose operativo el recargo indemnizatorio previsto
por el art. 2" de la ley 25.323.

j) Rechazo del trabajador por considerar el despido sin expresión


DE CAUSA, COMO MEDIDA DISCRIMINATORIA29
Rechazo despido notificado su "carta documento n° CD ...", [“telegrama
del ...”] por tratarse de una medida discriminatoria, contraria a lo dispuesto
por el art. 1o de la ley 23.592 y por lo tanto nula. En efecto, Uds. adoptan
dicha medida en razón de ... 30 y como consecuencia de ello pretenden dejarme
sin empleo. Por lo expuesto, siendo nulo el despido por importar un hecho
discriminatorio, los intimo para que dentro del plazo de 48 hs. dejen la medida
sin efecto y me reintegren mis tareas habituales, caso contrario promoveré el
pertinente amparo judicial a su costa.

29
Referencia normativa. Art. 1o, ley 23.592. Su aplicación en el
contexto del contrato individual de trabajo, con la consecuencia
reclamada en el texto sugerido, ha sido admitida por la CSJN, desde el
7/12/10, en oportunidad de resolver la causa "Álvarez, Maximiliano, y
otros c/Cencosud SA s/acción de amparo”.

30
Deberán explicarse los motivos por los cuales se considera al
despido como un hecho discriminatorio, por lo general, pasarán por
alguna de las causales de exclusión que de un modo amplio, se
encuentran previstas por el mismo art. 1o de la ley 23.592, también
por el convenio OIT 111. Hemos desarrollado estas causales en
nuestros trabajos Discriminación y despido (elDial, 19/9/06, DC9BB)
y El despido al testigo “infiel" como acto discriminatorio, aun en la
industria de la construcción (ED, 236-326).
Referencia jurisprudencial. “La carga probatoria que se impone al
empleador en los casos en que se alega discriminación no implica
desconocer el principio contenido en el art. 377 del Cód. Procesal, ni lo
específicamente dispuesto en la ley 23.592, ya que quien se considere
afectado por cualquiera de las causales previstas en ella debe
demostrar que posee las características que considera motivantes del
hecho que ataca y los elementos de hecho, o en su caso, la suma de
indicios de carácter objetivo en los que funda la ilicitud de éste,
quedando en cabeza del empleador acreditar que el despido tuvo por
causa una motivación distinta y a su vez excluyeme, por su índole, de
la animosidad alegada” (CNTrab, Sala II, 7/7/05, “Cresta, Erica V.
c/Arcos Dorados SA", TSS, 2005-864).

Causas voluntarias que atañen a ambas partes

a) Empleador preavisa finalización de contrato a plazo determinado 1

1
Referencia normativa. Art. 94, LCT.
1) Contrato de duración menor a un año
Preavisamos la terminación de su contrato de trabajo a plazo determinado el
día ...2. Haberes, liquidación final y certificados estarán a su disposición en los
plazos legales3.

2
El preaviso debe ser otorgado en todos los casos de contratos
con duración igual o mayor a un mes, con una anticipación no menor a
un mes ni mayor a dos meses. La parte que omite el preaviso, acepta
la conversión del contrato a plazo indeterminado.

3
Acerca de los plazos legales, ver § 16, a. 1.

2) Contrato de duración mayor a un año


Preavisamos la terminación de su contrato de trabajo a plazo determinado el
día ... Haberes, liquidación final, indemnización especial 4 y certificados estarán
a su disposición en los plazos legales5.

4
En estos casos, rige la indemnización especial del art. 250 de la
LCT.

5
Acerca de los plazos legales, ver § 16, a. 1.

b) Empleador comunica la extinción de contrato eventual,


POR CUMPLIMIENTO DEL EVENTO O CONDICIÓN RESOLUTORIA 6
Habiéndose agotado el evento para el cual fue contratado, consistente en ... y,
por lo tanto, cumplida la condición resolutoria pactada oportunamente,
notificárnosle la terminación del contrato eventual que nos vincula a partir del
día de la fecha. Haberes, liquidación final y certificados estarán a su
disposición en los plazos legales7.

6
Referencia normativa. Art. 99, LCT.
7
Acerca de los plazos legales, ver § 16, a. 1.

Causas involuntarias. Renuncia por incapacidad absoluta 1


Hallándome absolutamente2 incapacitado para trabajar, de acuerdo a lo
dictaminado en ...3 según certificados que coloco a su disposición, notifico la
extinción del contrato por la causal prevista en el art. 212 de la LCT. En
consecuencia, los intimo para que dentro del plazo legal paguen la
indemnización especial allí prevista, caso contrario accionaré 4.

1
Referencia normativa. Art. 212, párr. 4o, LCT.

2
Referencia jurisprudencial. “Si bien la ley de contrato de trabajo
no establece porcentajes para la determinación de la incapacidad
absoluta, permanente y definitiva, debe considerarse que el 66%
previsto en el art. 33, ley 18.037, constituye, en principio, la norma de
valuación más equitativa a los fines del otorgamiento de la
indemnización del art. 212 de la ley citada” (CNTrab, Sala III,
16/9/08, “Jiménez, Romelia O. c/Bilbao, Ricardo E.”, LLonline,
AR/JUR/9680/2008).

3
Por lo general será el dictamen de la Comisión Médica
interviniente en el proceso administrativo tendiente al reconocimiento
de una prestación de seguridad social con motivo de la incapacidad.
No se trata de un documento excluyente, puede ser reemplazado con
cualquier constancia médica fundada, donde se certifique la
incapacidad, que para ser absoluta debe ser del tipo permanente e
igual o superior al 66% sobre la TO. Cuanto mayor sea la jerarquía del
órgano certificante de la incapacidad, menor será el margen de
discusión al que podrá recurrir el empleador, que siempre cuenta con
la posibilidad de cuestionar en proceso judicial la existencia del
presupuesto normativo para la consecuente obligación al pago de la
indemnización especial. Ésta es más bien propia de la seguridad social
y sólo, por defecto, nuestro régimen coloca la obligación en cabeza del
empleador.

4
El plazo legal viene impuesto por el art. 255 bis de la LCT (texto
ley 26.593). Por remisión al art. 128 de la LCT, dicho plazo computado
a partir de la recepción de la pieza rescisoria, será de cuatro días
hábiles para los trabajadores remunerados por mes o quincena y de
tres días hábiles para los trabajadores remunerados por semana.

Resolución por incumplimiento (injuria) del trabajador 1

a) Por reiteración de incumplimientos2


En atención al grave incumplimiento de su parte, consistente en ... 3, siendo el
mismo reiteratorio de otros anteriores que fueron objeto de sanciones
correctivas, como ser ...4, que no lograron enmendar su actitud y colmaron
nuestro límite de tolerancia, haciendo por ello imposible la continuidad del
vínculo, notificárnosle su despido con justa causa y por su culpa a partir del
día de la fecha. Haberes pendientes, liquidación final y certificados de trabajo a
su disposición dentro de los plazos legales.

1
Referencias normativas. Arts. 242 y 243, LCT, y art. 1204, Cód.
Civil.
La LCT recibe, en su art. 242, la figura de la resolución por
incumplimiento, prevista por el art. 1204 del Cód. Civil. En el caso, se
trata de incumplimiento del trabajador que legitima la exteriorización
de la voluntad rescisoria del empleador, con la comunicación escrita,
conteniendo la expresión suficientemente clara de los motivos por los
cuales funda la ruptura del vínculo (art. 243, LCT). Con la recepción de
dicha comunicación por parte del trabajador, queda disuelto el
contrato y sin consecuencias indemnizatorias. Como las consecuencias
son imputables al incumplimiento del trabajador, pierde las
indemnizaciones por preaviso y antigüedad, conservando los demás
derechos ligados a la extinción del contrato (certificados, percepción
de liquidación final, etcétera). No cualquier incumplimiento legitima la
resolución del contrato por esta vía, debe tratarse de un
incumplimiento grave, muy grave, configurativo de injuria, esto es,
que por su gravedad no consienta la continuación del contrato. Como
sanción disciplinaria (la máxima sanción), el trabajador puede
cuestionar la medida dentro del plazo de treinta días previsto por el
art. 67 de la LCT y reclamar judicialmente el pago de las
indemnizaciones. En dicho proceso será el juez laboral quien
determine tanto la existencia del incumplimiento, como su entidad
para justificar la medida rescisoria, teniendo en consideración las
modalidades y circunstancias personales de cada caso.

2
Referencia jurisprudencial. "Los antecedentes desfavorables de
un operario pueden servir de apoyo a un despido si existe un último
hecho injurioso que pueda ser utilizado como causa inmediata y
directa de la decisión” (CNTrab, Sala IV, 26/ 11/09, "Tolaba, César D.
c/Taraborelli Automobile SA”, LLonline, AR/JUR/48982/ 2009).
“Resultó ajustado a derecho el despido dispuesto por el empleador
debido a la omisión del trabajador de dar aviso oportuno de su
enfermedad y del lugar en que se encontraba -art. 209, LCT-, pues, si
bien dicha falta genera la pérdida del derecho a cobrar remuneración,
la reiteración de estas omisiones y los antecedentes disciplinarios del
dependiente justificaron la adopción de la máxima sanción, en tanto
existió una injuria suficiente en los términos del art. 242 de la ley de
contrato de trabajo" (CNTrab, Sala IX, 9/10/07, "Gauna, Ariel A.
c/Piscis Seguridad SA”, LLonline, AR/ÍUR/7459/2007). "Resulta
justificado el despido de un trabajador que se desempeñaba en una
empresa alimenticia, si ha quedado acreditado que no usaba el barbijo
ni la cofia para realizar sus tareas, pues ello constituye suficiente
injuria en los términos del art. 242 de la ley de contrato de trabajo,
dado que la actividad del empleador se encuentra en el rubro
alimentación y que el uso de tales elementos resultaba indispensable
para sus labores, máxime cuando el dependiente fue apercibido en
reiteradas ocasiones por el mismo motivo” (CNTrab, Sala III,
30/4/10, "Quintana, Fernando M„ c/Productos Lipo SA y otro”,
LLonline, AR/JUR/17193/2010).

3
Acerca del contenido de esta comunicación, impuesta por el art.
243 de la LCT, ver § 16, b, nota 7. "Llega firme a esta instancia que la
demandada no ha ajustado su conducta rescisoria a lo prescripto por
el art. 243 de la LCT. Basta para ello releer el telegrama de despido.
De allí que los dichos de sus dos testigos apreciados conforme a las
reglas de la sana crítica (art. 386, CPCCN) carecen de la fuerza
convictiva que le pretende asignar el apelante, ya que ninguno de
ellos pudo describir una conducta del accionante que se emparentare
con la causal de despido invocada en el telegrama rescisorio” (CNTrab,
Sala VIII, 16/7/10, “Fernández Pérez, Horacio A. c/Consolidar Salud
SA”, DTonline, AR/JUR/42298/2010).

4
Incluir aquí la mención de los antecedentes anteriores que
fueron notificados por escrito, teniendo en cuenta la siguiente
referencia jurisprudencial: “Es injustificado el despido decidido por el
empleador si no probó la validez y autenticidad de la documentación
acompañada como supuestos antecedentes desfavorables que
justificarían la medida, lo que impide considerarlos como tales al
momento de analizar las circunstancias que rodearon al distracto,
máxime cuando, aun cuando pudiera pensarse que la inconducta del
trabajador no merezca sino la lisa y llana desaprobación y reproche,
no se demostró ni probó la aplicación de correctivos previos, en forma
progresiva y escalonada" (CNTrab, Sala VII, 13/3/08, "González,
Héctor A. c/Coto CICSA”, LLonline, AR/JUR/1933/2008).
b) Por incumplimiento grave
Dado que ...5 lo cual importa un incumplimiento gravísimo de su parte a los
deberes a su cargo y que por su gravedad no admite la continuación del
contrato, queda despedido por su exclusiva culpa a partir del día de la fecha.
Haberes pendientes, liquidación final y certificados de trabajo a su disposición
dentro de los plazos legales.

5
Incluir aquí la descripción clara del incumplimiento grave
configurativo de injuria, en los términos de los arts. 242 y 243 de la
LCT. A continuación, se ejemplifican distintos incumplimientos que así
fueron considerados por la jurisprudencia.

1) Trabajador que lesiona en modo intencional pacientes bajo su


cuidado6
... con fecha ... siendo las ... hs., Ud. fue sorprendido golpeando al paciente ...
bajo su cuidado, produciéndole incluso lesiones que comprometen la
responsabilidad de esta organización, en cuyo nombre Ud. actúa ...

6
Referencia jurisprudencial. "Estando acreditado que quien
desempeñaba tareas de maestranza y de cuidado de los residentes de
un cotolengo, propinó un puntapié a uno de ellos provocándole
lesiones, resulta justificado su despido, pues las especiales
condiciones de los internos en este tipo de instituciones no autorizan
ni mínimamente dicho actuar, aun cuando no tuviera antecedentes
disciplinarios de ese orden, en tanto constituye una injuria suficiente a
los fines de extinguir la relación” (CTrab Córdoba, Sala 10, 22/10/02,
"Romero, Oscar C. c/Cottolengo Don Orione y otros”, LLonline,
AR/JUR/70349/2010).

2) Que adultera registros del empleador para adjudicarse


comisiones por ventas en las cuales no intervino7
Habiéndose verificado que en ocasión de hallarse su supervisor de vacaciones,
Sr ..., durante el mes de ... del año ... Ud. se aprovechó de dicha circunstancia
para utilizar sus claves, ingresar a los registros informáticos de la firma y
adjudicarse las operaciones de venta nros. ... de nuestro registro,
reasignándoselas a su nombre, todo ello al efecto de percibir comisiones en
forma indebida ...

7
Referencia jurisprudencial. "Constituye injuria grave en los
términos del art. 242 de la ley de contrato de trabajo que impide la
prosecución de la relación laboral, el incumplimiento atribuido a un
trabajador que se desempeñaba como vendedor en un local de la
demandada y utilizando la clave de un gerente que se encontraba de
vacaciones reasignó a su nombre operaciones de ventas en las que no
había intervenido a efectos de que le fueran liquidadas las
correspondientes comisiones, pues, se trata de un incumplimiento que
frustra la expectativa acerca de la conducta leal y acorde con el deber
de fidelidad y buena fe" (CNTrab, Sala VI, 20/ 10/10, "Penando, Juan
M. c/Entertainment Depot SA”, LLonline, AR/JUR/71831/2010).

3) Que sustrajo de manera ilegítima elementos de la empresa8


Habiendo sido Ud. sorprendido el día ... en horario ... por los Sres. ... y ...
procediendo a retirar los siguientes elementos de propiedad de esta empresa,
a saber ..., sin justificar su proceder y dando cuenta con su actitud que
pretendía apropiarse de ellos en forma indebida ...

8
Referencia jurisprudencial. “Es justificado el despido del
trabajador dispuesto por su empleador al considerarlo partícipe de un
hecho ilícito consistente en la sustracción, sin autorización, de
elementos de su empresa, pues, la sentencia condenatoria dictada en
sede penal permite tener por acreditada la conducta deshonesta del
trabajador que constituyó un agravio a los intereses del empleador y
por ende una injuria que impide la prosecución del vínculo” (CNTrab,
Sala VI, 31/8/10, "Niz, Enrique R. c/Meller SA”, LLonline,
AR/JUR/48971/2010). "Corresponde confirmar la sentencia que
consideró justificada la medida rescisoria adoptada por la empleadora,
toda vez que se encuentra acreditado el retiro no autorizado de
insumos por parte del trabajador, en tanto aquellos fueron
encontrados en su mochila y no puede sostenerse que hubo violación
a la intimidad y privacidad del trabajador ya que al momento de la
revisación la demandada no tenía conocimiento de que la misma era
de su propiedad" (CNTrab, Sala IX, 20/4/10, “Hunt, Matías E. c/Juan
B. Justo 1333 SA”, DT, 2010-B-2341).

4) Quien sustrajo mediante adulteración de remitos 9


Dado que Ud. con fecha ... recibió las mercaderías consistentes en ...,
suscribiendo el pertinente remito y no pudo justificar el faltante de ..., siendo
que efectivamente ingresaron a la firma ...

9
Referencia jurisprudencial. “Resulta ajustado a derecho el
despido dispuesto por la empleadora por la causal de pérdida de
confianza, toda vez que se encuentra acreditado el faltante de
mercaderías, en tanto los actores suscribieron remitos por un número
mayor al que ingreso en la planta, lo que constituye injuria laboral"
(CNTrab, Sala IV, 31/8/10, "Muñoz, Alfredo R., y otro c/Deconti SA",
LLonline, AR/JUR/50279/2010).

5) Uso desviado y contrario a instrucciones de la red informática


suministrada por el empleador10
Habiéndose verificado que Ud., contrariando instrucciones expresas recibidas,
utilizó en forma desviada el acceso a la red informática suministrado por la
empresa, dado que la utilizó para descargar, en forma reiterada, material
pornográfico de una red privada ...
10
Referencia jurisprudencial. "Es justificado el despido de un
trabajador que, a pesar de las advertencias cursadas por sus
superiores, descargó, en forma reiterada, material pornográfico en
una red privada perteneciente a una empresa para la cual prestaba
tareas como técnico programador por encargo de su empleadora, pues
dicho proceder constituye injuria suficiente para justificar la ruptura
del vínculo” (CNTrab, Sala X, 30/6/10, “A., B.G. c/Sisdev Soc. de
Hecho y otros”, LLonline, AR/JUR/32212/2010).

6) Por riña en el lugar de trabajo11


Habiéndose verificado que Ud. agredió 12 físicamente a su compañero de
trabajo Sr. ... el día ... en horario de ...

11
Referencia jurisprudencial. “Para determinar si una riña en el
lugar de trabajo configura injuria en los términos del art. 242 de la ley
de contrato de trabajo, corresponde analizar las distintas actitudes
asumidas por los protagonistas y en el supuesto de que hubieran
existido agresiones físicas recíprocas debe determinarse quién fue el
agresor y quién la víctima y si ésta asumió una actitud defensiva”
(CNTrab, Sala III, 30/11/09, "Moya, Nelson E. c/CTL SA”, LLonline,
AR/JUR/48326/2009).

12
En estos casos, la medida debe recaer sobre el agresor.
Referencia jurisprudencial. "Resulta injustificado el despido de la
trabajadora accionante, decidido por el empleador en virtud de la riña
física que habría mantenido con una compañera de trabajo, pues los
elementos probatorios aportados a la causa no permiten establecer
que fuera ella quien inició la pelea ni que hubiera incurrido en una
conducta anterior que justificara la agresión recibida” (CN Trab, Sala
I, 31/3/10, "Frías, Cintia V. c/Chang Ki Taik y otro”, DT, 2010- A-
1493).

c) Por abandono de trabajo13


Ante su silencio a nuestra intimación cursada mediante "carta documento" 14 y
no habiendo retomado tareas pese a la intimación, hacemos efectivo el
apercibimiento allí dispuesto y consideramos disuelto el contrato por abandono
de trabajo a Ud. imputable. Haberes pendientes por liquidación final y
certificados de trabajo a su disposición en los plazos legales.

13
Referencia jurisprudencial. "La figura del abandono de trabajo,
requiere para su configuración: a) la inejecución por el trabajador, sin
aviso, de la prestación laboral; b) la intimación de reintegro, dentro de
un plazo razonable según las circunstancias, y c) la persistencia del
trabajador en su conducta omisiva, durante el plazo fijado" (CNTrab,
Sala VIII, 28/3/08, "Morales, Marcelo D. c/Cfhclar SA”, LLonline,
AR/JUR/3106/2008).

14
Referencia jurisprudencial. “Dado que la característica principal
del abandono de trabajo es ‘el silencio del dependiente’, no cabe tener
por configurado dicho abandono en el caso de que exista intercambio
telegráfico que evidencie una situación conflictiva entre las partes,
pues ello descarta el ánimo abdicativo del trabajador” (CNTrab, Sala
VII, "Ramos, María C. c/Camillis, María del C., y otros", LLonline,
AR/JUR/32589/2010).

d) Rechazo del trabajador15


Rechazo despido notificado su "carta documento n" CD
["telegrama del ..."], por ser falsa la causal alegada y, a todo evento,
desproporcionada la medida. Niego que ... 16. En consecuencia, los intimo para
que dentro del plazo de 48 hs. “dejen sin efecto la arbitraria medida y
otorguen tareas habituales”, "... y la sustituyan por medida que permita la
continuidad del contrato” “paguen dentro del plazo legal las indemnizaciones
derivadas del despido (integración del mes del despido -art. 233, LCT-,
indemnización sustitutiva del preaviso -art. 232, LCT- e indemnización por
antigüedad -art. 245, LCT-, caso contrario accionaré sin más trámite ni
intimación, tornándose operativo el recargo indemnizatorio previsto por el art.
2° de la ley 25.323.

15
Por revestir el carácter de sanción disciplinaria, el despido
fundado en incumplimiento debe ser rechazado por el trabajador
dentro de los treinta días que, como plazo de caducidad, establece el
art. 67 de la LCT.

16
Espacio destinado al cuestionamiento de la existencia de los
hechos imputados por el empleador en la comunicación de despido o
la justificación que éstos no poseen el carácter de "injuria” o
incumplimiento grave, con el alcance previsto por el art. 242 de la LCT.

Resolución por incumplimiento (injuria) del empleador (DESPIDO


INDIRECTO)1

1
Referencia normativa. Arts. 242, 243 y 246, LCT.
Los arts. 242 y 243 de la LCT son comunes para ambas partes, por el
primero se define a la injuria como causal que habilita la resolución
del vínculo y, por el segundo, se imponen los recaudos para su
comunicación a la otra parte, con cuya recepción queda extinguido el
contrato. Restringido al contexto de la injuria provocada por el
empleador, se adiciona la previsión contenida en el art. 246 de la LCT,
que reconoce a favor del trabajador afectado, el derecho a la
percepción de las indemnizaciones previstas en los arts. 232
(sustitutiva del preaviso), 233 (integración del mes del despido) y 245
(por antigüedad o común por despido) de la misma ley.
a) Luego de silencio a intimación2
Ante su silencio a mis concretas intimaciones cursadas por TCL n° CD ...3 de
fecha el cual confirma la veracidad de los hechos allí expuestos y persistiendo
los incumplimientos graves4 que la originaron, en especial ... 5, todo lo cual
hace imposible la continuidad del contrato en tales términos, no me dejan otra
alternativa que hacer efectivo el apercibimiento anterior y considerarme
despedido por su exclusiva culpa. En consecuencia, los intimo para que dentro
del plazo legal6 paguen remuneraciones adeudadas, la liquidación final por
egreso, con más las indemnizaciones derivadas del despido (integración del
mes del despido -art. 233, LCT-, indemnización sustitutiva del preaviso -art.
232, LCT- e indemnización por antigüedad -art. 245, LCT-), caso contrario
accionaré sin más trámite ni intimación, tornándose operativo el recargo
previsto por el art. 2o de la ley 25.3237.

2
Referencias normativas. Art. 57, LCT, y ley 24.487.
Referencia jurisprudencial. “El silencio de la demandada ante la
intimación cursada por el trabajador para que se registre la relación
laboral y se le abone los salarios adeudados, constituye de por sí un
proceder injurioso apto para justificar la ruptura de la relación
laboral” (CNTrab, Sala III, 28/9/06, “Andrade, Walter F. c/Martín,
Gonzalo G., y otro", LLonline, AR/JUR/6682/2006).

3
La identificación del TCL gratuito se encuentra en la oblea
autoadhesiva que coloca el Correo en la copia certificada que entrega
al trabajador. No coincide con el número del formulario preimpreso.
Comienza con las siglas “CD", luego se siguen los nueve dígitos de la
numeración y termina con la sigla “AR”. Correo Argentino brinda, por
Internet, un servicio de seguimiento de estas piezas, al cual se accede
con la dirección www.correoargentino.com.ar. Una vez allí, tecleando
en "Correspondencia con origen y destino nacional", en el menú de
identificación de la pieza, se carga la numeración respectiva, cuidando
de hacerla comenzar con "AR”, encontrando luego la información del
seguimiento requerido. La impresión de la pantalla de la hoja n° 2,
donde consta si la pieza fue entregada, ayuda a solucionar los
problemas derivados del transcurso del tiempo, ya que el Correo no
guarda los registros históricos. Dado que la elección del medio de
notificación hace responsable a quien lo elige de su resultado, siempre
es conveniente adoptar el recaudo de confirmar el resultado de la
intimación enviada al empleador, no descartándose el mayor recaudo
de su reiteración, antes de extinguir el vínculo por silencio.

4
Puede utilizarse esta expresión o la de "injuria" que surge del
art. 242 de la LCT. En el texto se optó por la del “incumplimiento
grave”, seguida de la referencia a que con ella se hace imposible la
continuidad del vínculo, al solo efecto de lograr una mayor claridad. En
definitiva, la injuria laboral se compone de esos atributos, también
configurativos del llamado pacto comisorio (art. 1204, Cód. Civil) que
habilita a una de las partes, para rescindir el contrato con sustento en
el incumplimiento de la otra.

5
El art. 243 de la LCT exige para la comunicación rescisoria, que
debe ser escrita, un cierto grado de autonomía en su redacción. Debe
contener la "expresión suficientemente clara de los motivos en que se
funda la ruptura del contrato”.

6
Por plazo legal, ver § 16, a. 1.

7
El art. 2° de la ley 25.323 describe como hecho necesario para la
aplicación de su consecuencia, que es un recargo indemnizatorio del
50% sobre las indemnizaciones que se indican en el texto sugerido,
que el empleador hubiera sido "fehacientemente intimado por el
trabajador” y luego “no le abonare las indemnizaciones” en cuestión y
“lo obligare a iniciar acciones". Algún criterio restrictivo, que
consideramos no se ajusta al texto de la norma y termina imponiendo
mayores requisitos que los previstos por el legislador, exige una
segunda intimación, luego de la extinción contractual y transcurridos
los plazos del art. 255 bis de la LCT. Así fue resuelto en forma aislada
por algún fallo, que con dicho fundamento privó al trabajador del
recargo indemnizatorio (CNTrab, Sala I, 31/8/10, “Nieto, Roberto R.
c/Centurión Seguridad SRL”, LLonline, AR/JUR/51006/2010). La
jurisprudencia mayoritaria sigue el texto de la ley y no exige una
segunda intimación (id., Sala VI, 27/8/10, “Aramayo, Susana B.
c/Embajada de la República Popular China y otro", LLonline,
AR/JUR/50941/2010; id., Sala VII, 25/8/10, "Ponce, Raúl O.
c/Orígenes AFJP SA", LLonline, AR/JUR/48987/2010).

b) Persistiendo incumplimientos con respuesta a intimación 8


Rechazo su carta documento n° ... 9 por falsa, maliciosa e improcedente.
Ratifico en todos sus términos mi TCL anterior. Con su respuesta, Uds.
ratifican la veracidad de los incumplimientos que le fueron imputados, puesto
que ...10. Por lo expuesto, persistiendo los incumplimientos graves indicados en
mi TCL anterior, especialmente es que siendo imposible la continuidad del
contrato en tales términos, no me dejan otra alternativa que hacer efectivo el
apercibimiento anterior y considerarme despedida por su exclusiva culpa. En
consecuencia, los intimo para que dentro del plazo legal 12 paguen
remuneraciones adeudadas, la liquidación final por egreso, con más las
indemnizaciones derivadas del despido (integración del mes del despido -art.
233, LCT-, indemnización sustitutiva del preaviso -art. 232, LCT- e
indemnización por antigüedad -art. 245-); caso contrario accionaré sin más
trámite ni intimación, tornándose operativo el recargo previsto por el art. 2°
de la ley 25.323.

8
Referencia jurisprudencial. "Cuando se invocan varias causas
para justificar la denuncia del contrato de trabajo, basta acreditar una
de ellas para concluir que la ruptura resultó ajustada a derecho si se
trata de un incumplimiento que por su gravedad, no consiente la
continuidad de la relación, ni siquiera a título provisorio” (CNTrab,
Sala III, 18/10/07, "Carreras, Juan C. c/Lapidus, Luis J., y otros”,
LLonline, AR/JUR/9675/2007).

9
La identificación de la pieza recibida, tendrá que ver con el medio
de comunicación elegido por el empleador. Puede ser carta documento
de Correo Argentino o compañía privada (OCA, Andreani, etc.),
telegrama u otro medio.

10
La claridad en la exposición de los motivos de la rescisión en este
caso, debe hacerse cargo (cuando corresponda) de los argumentos
expuestos por el empleador en su respuesta, afirmando que de ellos
no resulta una solución u oferta de solución para los incumplimientos
imputados en la intimación, sobre todo cuando el empleador se limita
a cuestionar las bases fácticas de ella con la puntual negativa.

11
Acerca de la autonomía de la comunicación rescisoria, ver § 16, nota
13.

12
Acerca del plazo legal y entidad de la intimación a los fines del art.
2° de la ley 25.323, ver § 19, nota 3.

1) Negativa de trabajo o incumplimiento al deber de dar ocupación 13


... La injustificada negativa de trabajo el día ... cuando me presenté a hacerlo
en presencia de testigos y me fue negado el acceso al establecimiento ...

13
Referencia normativa. Art. 78, LCT.
Referencia jurisprudencial. “Resulta justificada la situación de despido
indirecto en que se colocó un trabajador, si existió una efectiva
sustracción de la empleadora del deber de ocupación que se
materializó a través de la negativa de trabajo acreditada mediante la
prueba de testigos, por lo cual se encuentra configurada una injuria
patronal que hacía insostenible el mantenimiento del vínculo" (CN
Trab, Sala II, 20/4/10, "Castillo, Teresita J. c/Actionline de Argentina
SA", LLonline, AR/JUR/20374/2010). “Es ajustada a derecho la
situación de despido en que se colocó quien prestaba tareas para el
supermercado demandado, pues, las declaraciones de los testigos
acreditan la negativa de trabajo invocada por aquél en su telegrama
rescisorio, al no dejarlo ingresar al establecimiento donde prestaba
tareas y que el trato recibido del gerente de la sucursal en que
prestaba tareas era malo, con lo cual quedaron probadas las injurias
invocadas en la comunicación cursada" (CNTrab, Sala III, “Gómez,
Marcelo c/Jumbo Retail Argentina SA”, DT, 2010-B-2665). “Es
justificado el despido indirecto en que se colocó un trabajador que
luego de reintegrarse a su trabajo y solicitar por prescripción médica
cumplir funciones en tareas administrativas y jornada reducida fue
dispensado de prestar tareas con goce de haberes, habiéndose el
empleador limitado a manifestar que no contaba con vacantes sin
producir prueba conducente al respecto” (id., Sala VI, 18/05/10,
“Moretti, Desdémona J. c/Swiss Medical SA”, LLonline, cita
AR/JUR/26734/2010).

2) Incumplimiento consistente en cuestionamiento a la existencia


de relación laboral14
... agravado todo ello por su temerario 15 cuestionamiento a la existencia de la
relación laboral ...

14
Referencia jurisprudencial. "Debe tenerse por justificado el
despido indirecto toda vez que el desconocimiento del vínculo
efectuado por la empresa para la que prestaba servicios el trabajador
constituyó, de por sí, una negativa de trabajo y una conducta
injuriante de tal magnitud que impedía la prosecución de la relación
laboral -art. 242, LCT-" (CNTrab, Sala VII, 30/3/09, “Vera, Pablo A.
c/SA Alba Fábrica de Pinturas Esmaltes v Barnices y otro”, LLonline,
AR/JUR/6548/2009).
15
Referencia normativa. Art. 275, LCT.

3) Negativa a subsanar irregularidades de registro16


... su cuestionamiento a las bases fácticas de mi intimación, dando cuenta que
no procederá a subsanar las irregularidades de registro que allí fueron
indicadas ...

16
Referencia jurisprudencial. "Resulta justificado el despido
indirecto en el cual se colocó el trabajador si se acreditó por medio de
prueba testimonial el incorrecto registro de la categoría laboral y de
una remuneración inferior a la realmente percibida, todo lo cual
constituye injuria grave que no consiente la prosecución del vínculo"
(CNTrab, Sala III, 16/7/10, “Barrientos, Sergio D. c/Ami Music SA y
otro", DT, 2010-B-2659). "Es procedente la multa prevista en el art.
15, ley 24.013, aun cuando el trabajador se haya considerado
despedido antes del vencimiento del plazo previsto en el art. 11 de
dicha ley, atento a que el hecho de que el empleador desconociera la
fecha de inicio de la relación laboral invocada por el dependiente lleva
a considerar que aún cuando hubiera esperado el mencionado plazo, la
vinculación laboral tampoco se hubiera registrado debidamente" (id.,
Sala II, 12/8/10, "Taborda, Lautaro A. c/Módena Auto Sport SA”,
LLonline, AR/JUR/50179/2010).

4) Falta de pago de horas extras17


... la falta de pago de las horas extras reclamadas ...

17
Referencia jurisprudencial. “La confirmación de la realización de
horas extras, por parte del actor, y su falta de pago adecuado,
justifican la medida rescisoria adoptada por éste, y tornan
procedentes los rubros diferidos a condena” (CNTrab, Sala VIII,
16/7/10, "Coronel, Rafael L. c/High Top Security SA y otro", LLonline,
AR/JUR/42319/2010). "Reviste carácter injuriante la oposición de la
empleadora al pago de las horas extras efectivamente laboradas ante
las intimaciones cursadas por el trabajador (arts. 242 y 246, LCT) y,
dada la índole alimentaria de la referida deuda salarial, cuya falta de
pago tornó imposible la continuidad del vínculo laboral, justifica la
denuncia del contrato de trabajo" (CNTrab, Sala IX, 31/3/10,
"Gamboa, Roberto F. c/Short Time SRL y otro”, LLonline,
AR/JUR/7138/2010).

5) Respuesta negativa a la intimación de pago por diferencias


salariales18
... la falta de pago de las diferencias salariales reclamadas ...

18
Referencia jurisprudencial. "Resulta justificado el despido
indirecto en el que se colocó un dependiente de la Asociación del
Fútbol Argentino, como conse cuencia de la negativa del principal de
abonar diferencias reclamadas en concepto de bonificación por
antigüedad, pues tal actitud se revela por si sola como válida para
disolver el contrato por configurar injuria con los alcances de los arts.
242 y 246 de la ley de contrato de trabajo" (CNTrab, Sala X, 7/7/10,
"Frezza, Luis M. c/Asociación del Fútbol Argentino, AFA”, LLonline,
AR/JUR/42184/2010). "Resulta ajustado a derecho el despido
indirecto en que se colocó el trabajador dependiente de una
administradora de fondos de jubilaciones y pensiones pues quedó
demostrado que ésta le abonó de modo insuficiente su remuneración,
omitiendo especialmente liquidar debidamente las comisiones
derivadas de las operaciones por él concertadas" (CNTrab, Sala IX,
22/4/10, “Gallino, Guillermo V. c/MET AFJP SA”, LLonline,
AR/JUR/17040/2010).

6) Negativa del empleador a liquidar salarios sobre la base de jornada


normal, persistiendo su liquidación de manera proporcional o “a tiempo
parcial"19
... su negativa al pago de la remuneración debida, siendo intolerable la
continuación del contrato con el pago de salarios proporcionales o inferiores a
los devengados por la jornada cumplida ...

19
Referencia jurisprudencial. “La negativa del empleador a
reconocer el derecho del trabajador a percibir su remuneración en
base a una jornada normal -y no a tiempo parcial- la cual se vio
objetivada con la negativa argumentada en la respuesta a la
requisitoria que le fue formulada en tal sentido y bajo apercibimiento
de considerarse en situación de despido, constituyó una injuria que
imposibilitó la prosecución del vínculo laboral y justifica la decisión
rescisoria” (CNTrab, Sala II, 30/4/10, "Goncalves, Mariana c/Milkafe
SA y otros”, LLonline, AR/JUR/20682/2010).

c) Sin intimación previa20


En atención a los hechos aberrantes que me vi obligado por Uds. a padecer el
día ... en horario de ... en mi lugar habitual de trabajo, cuando fui víctima de
la agresión (“física”, “verbal muy grave”, “sexual") proveniente de ...
(empleador o quien ejerce su voluntad en el lugar de trabajo), consistente
en ... (descripción de los hechos) que me ocasionó daño en ... (salud, corporal,
moral, etc.), obligándome a promover la respectiva denuncia por ante ..., lo
cual representa un incumplimiento gravísimo de su parte a mi derecho a
contar con condiciones dignas y seguras en el trabajo, que no admite la
continuación del vínculo y es configurativo de injuria, notifico a Uds. mi
determinación de considerarme despedido por su exclusiva culpa. En
consecuencia, los intimo ...21.

20
Una prolongación de la desigualdad real del contrato de trabajo
es la exigencia al trabajador de intimar previamente al empleador, en
prácticamente todos los casos donde, por ser víctima de
incumplimientos laborales, luego ejerce su derecho a colocarse en
situación de despido indirecto con reclamo indemnizatorio. Aun
tratándose de un contrato bilateral, con prestaciones recíprocas y
hasta reflejas (al deber de uno, le corresponde un derecho del otro),
solo al trabajador se le exige intimar previamente a la otra parte, para
que subsane tal o cual incumplimiento, antes de habilitar la extinción
del contrato con derecho a indemnización. Así, por ejemplo, si el
trabajador hurta bienes del empleador, tal hecho es generalmente
admitido con carácter de injuria, que habilita el despido directo y con
justa causa. Ningún juez obliga al empleador a intimar previamente al
trabajador, para que le devuelva la cosa hurtada y que luego continúe
el contrato como que el suceso nunca ocurrió. Sin embargo, cuando es
el empleador que omite depositar las retenciones que practica sobre el
salario del trabajador, o recibe sus servicios personales y no cumple
con el pago de salarios, se exige la previa intimación del dependiente
a regularizar el incumplimiento y sólo con su persistencia (atributo no
exigido por el texto del art. 242, LCT) se habilita el despido indirecto
con derecho a indemnización. Es el criterio jurisprudencial y doctrinal
imperante. De todos modos entendemos que, aun en dicho contexto,
dado un incumplimiento extremo como ser la violencia laboral directa
del empleador (por vía de los hechos, como ser maltrato físico,
agresión sexual, etc.), habilitaría al trabajador afectado para
colocarse en situación de despido indirecto con derecho a ser
indemnizado, aun cuando no existiera previa intimación, tal como se
plantea en el texto propuesto.

21
Se sigue la intimación para que dentro del plazo legal se paguen
las indemnizaciones, tal como se ha propuesto en los anteriores.

d) Despido indirecto derivado de ejercicio abusivo del ius variandi 22


1) Por traslado del establecimiento sin compensación
Rechazo los términos de su carta documento n° ... por falsa, maliciosa e
improcedente. Ratifico en todos sus términos mi colacionado anterior. Ratifico
que el traslado del establecimiento, me provoca un perjuicio en tiempo y en
dinero que Uds. no pretenden compensar, dada la claridad de su respuesta,
importando ello un acto contrario a lo dispuesto por el art. 66 de la LCT. En
consecuencia, insistiendo Uds. en ejercitar en forma abusiva el ius variandi
dado que no reconocen los perjuicios que me provoca la determinación que
han adoptado, hago efectivo apercibimiento anterior y ejercito la opción del
art. 66 de la LCT considerándome despedido por su exclusiva culpa. En
consecuencia, intimóles plazo legal paguen ... 23.

22
Referencia normativa. Art. 66, LCT. El texto se vincula con los del
§ 6 de este capítulo.

23
Se sigue la intimación para que dentro del plazo legal se paguen
las indemnizaciones, tal como se ha propuesto en los anteriores.

2) Por modificación de horarios de trabajo24


Ratifico que el cambio de horarios de trabajo por Uds. pretendido importa uso
abusivo del ius variandi, porque la medida es irrazonable e invade el tiempo
que tengo diagramado para otras actividades ...

24
Referencia jurisprudencial. “El horario laboral integra la
estructura de la relación laboral, por lo que no puede ser modificado
unilateralmente por el empleador, excepto por condiciones esenciales,
que debe acreditar. La legitimidad del ejercicio del ius variandi está
subordinada a la no alteración sustancial del contrato, o a la ausencia
de perjuicio material, y debe responder a los fines de la empresa y
exigencias de la producción, es decir, que median razones objetivas
derivadas de la organización que obligan a tomar esa medida. En
autos, están ausentes estos elementos” (CNTrab, Sala VIII, 30/5/10,
"Ligorria, Flavia M. c/Distribuidora Rabopa SA y otro”, LLonline,
AR/JUR/I75I6/20I0).

e) Despido indirecto derivado de la infracción al fuero sindical 25


Persistiendo “el despido” [“la suspensión” y "la modificación de condiciones de
trabajo”] luego de mi intimación cursada por TCL n° CD ... de fecha ... y
siendo intolerable la continuidad del vínculo en los términos que Uds. plantean,
no me dejan otra alternativa que hacer efectivo el apercibimiento anterior,
ejercitando la opción prevista a mi favor por el art. 52 de la LAS
considerándome despedido por su exclusiva culpa. En consecuencia, los intimo
para que dentro del plazo legal26 paguen remuneraciones adeudadas, la
liquidación final por egreso, con más las indemnizaciones derivadas del
despido (integración del mes del despido -art. 233, LCT-, indemnización
sustitutiva del preaviso -art. 232, LCT- e indemnización por antigüedad -art.
245, LCT-, la especial del art. 52 de la ley 23.551), caso contrario accionaré
sin más trámite ni intimación, tornándose operativo el recargo previsto por el
art. 2o de la ley 25.32327.

25
Referencia normativa. Art. 52, LAS. El texto se vincula con los
detallados en el § 14.

26
Por plazo legal, ver § 16, a, 1.

27
Por los alcances de la intimación a los fines previstos por el art.
2" de la ley 25.323, ver § 19, nota 2. Según la doctrina legal
emergente de los plenarios de la CNTrab n° 313 ("Casado”) y 320
("Iurleo”), el recargo indemnizatorio se vería limitado a los supuestos
previstos por el art. 2° de la ley 25.323 y no comprendería otras
indemnizaciones, como la especial del art. 52 de la LAS.

D) Posteriores a la extinción del contrato


Incumplimiento al pago indemnizatorio

a) En casos de despido directo e incausado


Ante despido directo y sin expresión de causa, notificado mediante su carta
documento n° CD ..., telegrama del ..., y habiendo transcurrido en exceso el
plazo previsto por el art. 255 bis de la LCT, sin que cumplieran en debida
forma con las obligaciones derivadas de la extinción del contrato, intimo a Uds.
para que, dentro del plazo de 48 hs, paguen las indemnizaciones derivadas del
despido (integración del mes del despido -art. 233, LCT-, indemnización
sustitutiva del preaviso -art. 232, LCT- e indemnización por antigüedad -art.
245, LCT-), caso contrario accionaré sin más trámite ni intimación, tornándose
operativo el recargo previsto en art. 2o de la ley 25.3231.

1
Para los alcances interpretativos del tiempo propio de la
intimación de pago a que alude el art. 2° de la ley 25.323, ver § 19, a.

b) Despido indirecto (intimación posterior A LA COMUNICACIÓN


RESCISORIA)
Habiendo transcurrido en exceso el plazo previsto por el art. 255 bis de la LCT
sin que cumplieran en debida forma con las obligaciones derivadas de la
extinción del contrato, intimo para que dentro del plazo de 48 hs. paguen las
indemnizaciones derivadas del despido (integración del mes del despido -art.
233, LCT-, indemnización sustitutiva del preaviso -art. 232, LCT-, e
indemnización por antigüedad -art. 245, LCT-), caso contrario accionaré sin
más trámite ni intimación, tornándose operativo el recargo previsto en art. 2°
de la ley 25.323.

c) Motivada en pago parcial de indemnizaciones

1) Pago parcial motivado en falsa fecha de ingreso


Ante despido directo e incausado, notificado por su "carta documento n° CD
...”, "telegrama del ...”, seguida del pago parcial de remuneraciones,
liquidación final e indemnizaciones calculadas de manera incorrecta, notifico a
Uds. que impugno dicha liquidación, toda vez que ella fue practicada ... 2 sobre
bases apócrifas referidas a mi fecha de ingreso (la real fecha de ingreso fue
del ... en su establecimiento sito en .... haciendo idénticas tareas a las
cumplidas con posterioridad a la falsa fecha de ingreso por Uds. registrada
del ..., sólo que intermediando entre ambas fechas como empleador
meramente registral la firma ...). Por lo expuesto, los intimo para que dentro
del plazo de 48 hs. paguen la liquidación final por egreso, más las
indemnizaciones derivadas del despido (integración del mes del despido -art.
233, LCT-, indemnización sustitutiva del preaviso -art. 232, LCT- e
indemnización por antigüedad -art. 245, LCT-), con más la especial del art. 1o
de la ley 25.323 derivada de la irregularidad de registro consistente en falsa
fecha de ingreso, todo ello calculado sobre las bases reales de contratación,
deduciéndose los pagos realizados “a cuenta" por Uds. (art. 260, LCT), caso
contrario accionaré sin más trámite ni intimación, tornándose operativo el
recargo previsto por el art. 2o de la ley 25.3233.

2
Insertar aquí las razones por las cuales se impugnan la
liquidación practicada por el empleador y, en su caso, los certificados
de trabajo. Las razones más comunes pasan por el cómputo de un
salario inferior en la base de cálculo (por omitir comisiones o rubros
"no remuneratorios” que, con sustento en las previsiones del convenio
OIT 95, sobre la protección del salario, sí tendrían tal carácter y
debieran computarse en la base de cálculo, doctrina compartida por la
Corte Suprema en el caso "Pérez c/Disco", o una antigüedad que no
coincide con la real, sea debido a la irregularidad de registro derivada
de una falsa fecha de ingreso, sea por el habitual incumplimiento de
no reconocer la antigüedad adquirida por el trabajador en su
desempeño previo, con intermediación de una agencia de servicios
eventuales.
3
La previsión de hecho contenida en el art. 2o de la ley 25.323
pasa por la falta de pago de las indemnizaciones derivadas del despido
allí previstas, sin distinguir los casos donde hubo pago parcial ni
contemplarlos para reducir los recargos indemnizatorios que allí se
regulan, siempre como consecuencia de haberse obligado al
trabajador a litigar. La jurisprudencia tiende a acompañar este
criterio, estableciendo la procedencia de los recargos indemnizatorios
en forma completa, aun cuando hubo pago parcial. Así, se ha resuelto
que “resulta procedente la indemnización prevista en el art. 2o de la
ley 25.323, pues, si bien es cierto que la demandada realizó un pago al
momento del distracto, dicho pago resultó parcial, ya que no ha
abonado las indemnizaciones debidas conforme a derecho, siendo de
aplicación el art. 742 del Cód. Civil que debe complementarse con el
art. 260 de la ley de contrato de trabajo” (CNTrab, Sala VII, 13/2/07,
"Jerez, Francisca I. c./Danone Argentina SA”, LLonline,
AR/JUR/481/2007). En igual sentido, que "a los fines del progreso de
la sanción establecida en el art. 2° de la ley 25.323, la circunstancia
de que la empleadora demandada haya satisfecho en forma parcial la
obligación indemnizatoria no la exime del pago de la multa en
cuestión, en tanto la actora cuestionó en forma fehaciente la eficacia
cancelatoria del pago efectuado y la accionada con su actitud dio
origen a la promoción de una acción judicial para obtener el pago
íntegro de las sumas debidas" (CNTrab, Sala II, 17/ 11/06,
“Scognamillo, María F. c./Danone Argentina SA”, LLonline,
AR/JUR/9077/2006). Aunque también se ha interpretado por la Sala
VIII, que “el pago insuficiente de las indemnizaciones de los arts. 232,
233 y 245, LCT no constituye un presupuesto que habilite la
procedencia de la misma” (CNTrab, Sala VIII, 22/10/10, “Costa-
gliola, Darío V. c/Massuh SA”, LLonline, AR/JUR/72261/2010). Esta
última interpretación tiende a pulverizar el campo de aplicación de la
indemnización especial regulada por el art. 2o de la ley 25.323, puesto
que agrega un impedimento no exigido por el legislador y que no se
justifica, dado que el empleador que realiza un pago parcial, tiene la
oportunidad de enmendar su error pagando la diferencia y no lo hace,
prefiere obligar al trabajador a litigar, debiendo por tanto cargar con
las consecuencias que fueron legalmente previstas para ese daño,
adicionalmente infringido al trabajador.

2) Pago parcial por no contemplar la base remuneratoria correcta


Ante despido directo e incausado, notificado su carta documento n" CD ...,
seguido del pago parcial de remuneraciones e indemnizaciones devengadas
por dicha causa, puesto que omitieron computar como base de cálculo mi
mejor remuneración mensual devengada, excluyendo los llamados rubros “no
remuneratorios”, pese a que integraban mi ingreso habitual pagado en dinero
y constituyen salario conforme el convenio OIT 95, los intimo para que, dentro
del plazo de 48 hs., paguen las diferencias devengadas por tal motivo sobre la
liquidación final por egreso, sobre las indemnizaciones derivadas del despido
(integración del mes del despido -art. 233, LCT-; indemnización sustitutiva del
preaviso -art. 232, LCT- e indemnización por antigüedad -art. 245, LCT-), caso
contrario accionaré sin más trámite ni intimación, tornándose operativo el
recargo indemnizatorio previsto por el art. 2° de la ley 25.323.

Certificados de trabajo

a) Intimación a la entrega de certificados1

1) Transcurridos los treinta días de la extinción del contrato


Atento el tiempo transcurrido2, por haber pasado más de treinta días de la
extinción del vínculo sin que me hubiesen hecho entrega de los certificados y
constancias del art. 80 de la LCT (remuneraciones, depósito de aportes y
contribuciones previsionales, de obra social y sindicales, y certificado de
trabajo con las constancias establecidas en la ley 24.576 sobre la calificación
profesional obtenida en el puesto de trabajo desempeñado), los intimo por el
plazo perentorio de 48 hs. hagan entrega de éstos, bajo apercibimiento de lo
normado en el art. 45 de la ley 25.345.

1
Referencias normativas. Art. 80, LCT (texto art. 45, ley 25.345);
ley 24.576; art. 3o, decr. 146/01; res. gral. AFIP, 2316; res. ANScS
601/08.
El párr. 2° del art. 80 de la LCT (según ley 24.576) dispone la
obligación del empleador a entregar al trabajador, en oportunidad de
la extinción del contrato por cualquier causa, un certificado de trabajo
que debe contener las siguientes indicaciones: a) tiempo de
prestación de los servicios (fecha de ingreso y egreso); b) naturaleza
de los servicios (categoría del trabajador o tareas desempeñadas); c)
remuneraciones percibidas; d) aportes y contribuciones efectuadas a
los organismos de seguridad social; e) calificación profesional
obtenida en el o los puestos de trabajo desempeñados.

2
Los certificados y constancias deben entregarse cuando el
trabajador ‘‘lo requiriese a la época de la extinción de la relación" (art.
80, LCT). La norma, fija un plazo de cumplimiento para la obligación
del empleador, en dos días hábiles contados desde la recepción del
requerimiento que a tal efecto formulare el trabajador. La
reglamentación (decr. 146/01, art. 3o) amplía dicho plazo, al exigir
que el trabajador aguarde treinta días corridos, desde la extinción del
vínculo, para cursar la intimación prevista en la norma que se
pretende reglamentar. En atención al exceso reglamentario, sobre el
particular se ha resuelto: “Es inconstitucional el decr. 146/01 del
Poder Ejecutivo nacional, en cuanto establece que el trabajador
quedará habilitado para requerir el certificado de trabajo del art. 80
de la ley de contrato de trabajo transcurridos treinta días desde la
extinción del vínculo laboral, pues dicha reglamentación excede la
línea divisoria existente entre la delegación de poder para hacer la ley
la de conferir cierta autoridad al Poder Ejecutivo a fin de reglar los
pormenores y detalles necesarios para su ejecución” (CNTrab, Sala
VII, 16/7/10, “Premiani, Vicente N. c/Embajada del Reino de Arabia
Saudita", LL, 2010-E-257, y LLonline, AR/JUR/39526/2010).
Referencia jurisprudencial. “Corresponde rechazar la indemnización
prevista en el art. 45 de la ley 25.345 si, la intimación a entregar los
certificados de trabajo fue cursada en el mismo telegrama mediante el
cual el actor se consideró despedido, pues, dicha interpelación no
respetó lo exigido por el art. 80, LCT, en cuanto a producir el
emplazamiento con posterioridad a la disolución del vínculo” (CNTrab,
Sala VII, 30/9/10, “Bonder, Silvina F. c/Perevent Empresa de
Servicios Eventuales SA y otro”, LLonline, AR/JUR/60480/2010).

2) Intimación cursada juntamente con la comunicación de despido


indirecto
Quedan intimados a entregar, en el plazo de dos días hábiles a contar luego de
transcurridos treinta días corridos desde la recepción de la presente 3, los
certificados y constancias del art. 80 de la LCT (remuneraciones, depósito de
aportes y contribuciones previsionales, de obra social y sindicales, y certificado
de trabajo con las constancias establecidas en la ley 24.576 sobre la
calificación profesional obtenida en el puesto de trabajo desempeñado), bajo
apercibimiento de lo normado en el art. 45 de la ley 25.345.

3
Referencia jurisprudencial. "Corresponde acoger el reclamo de la
indemnización establecida en el art. 80, LCT, dado que en la
comunicación de la decisión de considerarse despedida la actora hizo
saber a la demandada sobre la intimación a entregarle en el plazo de
dos días hábiles a contar luego de transcurridos treinta días corridos
desde el despido los certificados de trabajo y certificado de servicios y
remuneraciones” (CNTrab, Sala V, 26/8/10, “Soria Moreno, Gabriela
A. c/Luna, Mónica N.”, DTonline, AR/JUR/51374/2010). En contra:
"La intimación fehaciente a la que aluden el art. 80, LCT, y el art. 3a
del decr. 146/01, sólo puede surtir sus efectos, es decir el inicio del
cómputo de los dos días y el posterior derecho a una indemnización
por falta de entrega del certificado de trabajo, una vez que haya
transcurrido el plazo de treinta días acordado al empleador para
cumplir con la exigencia legal” (CNTrab, Sala III, 31/5/10,
“Giacometti, Vicente c/Urga, Miguel A.", LLonline,
AR/JUR/24214/2010).

Respuesta
Contestando la intimación que nos cursara mediante su TCL n° CD notificamos
a Ud. que los certificados que reclama se encuentran a su disposición en ... 4,
donde deberá concurrir a retirarlos. Queda intimado a cumplir con dicha
intimación dentro del plazo de 48 hs., caso contrario procederemos a
consignarlos a su costa5.

4
Referencia jurisprudencial. "Corresponde revocar la multa
dispuesta en origen con base en el art. 80, LCT, toda vez que el
certificado de trabajo fue efectivamente puesto a disposición del
trabajador y no se ha acreditado que el demandado hubiera negado su
entrega ante el requerimiento, más aún, la certificación en cuestión
fue acompañada al contestar demanda y de la misma constancia se
desprende que fue certificada por la institución bancaria en fecha
contemporánea a la desvinculación, por lo que la puesta a disposición
efectuada por el demandado fue sincera" (CNTrab, Sala I, 12/7/10,
"Gaure, Ricardo G. c/Sanches, Fernando A.”, LLonline,
AR/JUR/42209/2010).
En estos casos, para que tal conducta del empleador lo exima de las
consecuencias previstas por el art. 80, LCT, la fecha de certificación de
la firma (por lo general bancaria o notarial) debe ser contemporánea
con la fecha en que ofreció los certificados, los que, además, debieron
estar redactados en legal forma y sobre la base de los datos reales de
la relación laboral. La mera manifestación de colocarlos a disposición
del trabajador en el intercambio, no lo exime luego de dichas
consecuencias, si la fecha de la certificación es contemporánea con la
agregación en juicio de la documentación o si ésta fue hecha sobre la
base de datos falsos.

5
Referencia jurisprudencial. "La puesta a disposición es
insuficiente para demostrar cumplida la obligación prevista en el art.
80 de la ley de contrato de trabajo, e impide considerar que la
empleadora haya tenido verdadera voluntad de entregar esa
documentación, máxime cuando no acreditó que el trabajador no
hubiera concurrido a retirar los certificados reclamados, ni tampoco
recurrió al instituto de la consignación" (CNTrab, Sala IV, 31/8/10,
"Bies, Gastón C. A. c/Hewlett Packard Argentina SRL y otro”, DT,
2010-B-2955). "Corresponde hacer lugar a la indemnización prevista
en el art. 80, último párrafo, de la ley de contrato de trabajo, en tanto
la entrega de los certificados de trabajo al dependiente en
oportunidad de la extinción laboral es una obligación del empleador
que debe ser cumplida en forma inmediata a la desvinculación -en el
tiempo que razonablemente puede demorar su confección- y si el
trabajador no concurrió a la sede de la empresa o establecimiento a
retirarlos, el empleador debió, previa intimación, consignarlos
judicialmente” (CNTrab, Sala III, 16/7/10, “Iannace, Fernando D.
c/Data Televisión SA y otros”, LLonline, AR/JUR/42600/2010).
Por otra parte se sostuvo: "Corresponde rechazar la demanda por
consignación deducida por el empleador, si los certificados de trabajo
aportados no cumplen con la normativa vigente en la materia, esto es,
la res. gral. de la AFIP 2316 y la res. de la ANSeS 601/08, pues el
trabajador no está obligado a recibir las certificaciones de aportes que
se aparten de los requisitos de validez impuestos por la normativa
vigente, toda vez que a partir del primer día hábil de agosto de 2008,
la obligación prevista en el art. 80 de la LCT sólo se considerará
efectivamente cumplida cuando se entreguen los formularios previstos
por las resoluciones citadas" (CNTrab, Sala IV, 22/10/10,
"Agrocomodities SA c/Barroti Acuña, Marcelo G. s/consignación",
LLonline, AR/JUR/69939/2010).

b) Impugnación a los certificados recibidos e intimación A SU


CONFECCIÓN Y ENTREGA EN LEGAL FORMA

Intimación del trabajador


Dejo impugnados los certificados del art. 80 de la LCT que me fueron
entregados en oportunidad del cese, por “omitir consignar mi real
empleador”6, [“no computar ni la real antigüedad” 7, “ni la real remuneración
devengada”, “no incluir constancia de depósito de aportes” 8, “constancias de la
ley 24.576”]9. En consecuencia, habiendo pasado más de treinta días de la
extinción del vínculo sin que me hubiesen hecho entrega en legal forma de los
certificados y constancias del art. 80 de la LCT (remuneraciones, depósito de
aportes y contribuciones previsionales, de obra social y sindicales, y certificado
de trabajo con las constancias establecidas en la ley 24.576 sobre la
calificación profesional obtenida el puesto de trabajo desempeñado), los intimo
por el plazo perentorio de 48 hs. hagan entrega de los mismos subsanando los
incumplimientos antes consignados, bajo apercibimiento de lo normado en el
art. 45 de la ley 25.345.

6
Referencia jurisprudencial. “Los certificados de trabajo
entregados al trabajador por la empresa de servicios eventuales, es
insuficiente a fin de considerar cumplida la obligación establecida en
el art. 80 de la ley de contrato de trabajo si se acreditó que la usuaria
era empleadora directa del trabajador, en tanto aquellos no reflejan
fielmente los datos reales del vínculo laboral” (CNTrab, Sala IX,
30/9/10, "Cantero, Miguel F. c/Syphon SA y otro”, LLonline,
AR/JUR/61296/2010).
7
Referencia jurisprudencial. “Resulta procedente condenar al
empleador, en el marco de un proceso por despido, a hacer entrega al
trabajador de una nueva certificación de servicios, si la otorgada
oportunamente no consignó la fecha real de ingreso” (CNTrab, Sala
VI, 16/7/10, "Cernadas, Oscar A. c/Rigecin Labs SA”, LLonline.
AR/JUR/42892/2010).

8
Referencia jurisprudencial. “Corresponde desestimar el agravio
planteado conforme la demandada sólo entregó a la dependiente uno
de los instrumentos reclamados en la demanda (la certificación de
servicios y remuneraciones) y omitió, en cambio, suministrarle los
otros dos (el certificado de trabajo y la constancia documentada
previstos, respectivamente, en el segundo y primer párrafo del art. 80
de la LCT), por lo que corresponde también confirmar la condena a
abonar la indemnización contemplada en el citado art. 80 de la LCT”
(CNTrab, Sala IV, 15/7/10, "Viveros, Romina E. c/Compañía
Financiera Argentina SA”, LLonline, AR/JUR/42638/2010).

9
Referencia jurisprudencial. "Resulta evidente el incumplimiento
del empleador con lo normado en el art. 80, LCT, toda vez que
habiendo sido intimado, mediante pieza telegráfica cursada por el
trabajador, a que hiciera entrega de la certificación correspondiente,
cumplió parcialmente la carga legal en virtud de que omitió entregar
el ‘certificado de trabajo’ en el que debe constar, además de lo
previsto en el citado artículo, la calificación profesional obtenida en el
o los puestos de trabajo desempeñados, hubiere o no realizado el
trabajador acciones regulares de capacitación” (CNTrab, Sala VI,
16/7/10, "Racedo, Juan M. c/HSBC La Buenos Aires Seguros SA”,
LLonline, AR/JUR/42681/2010).

Incumplimientos al depósito de aportes1


Intimación cursada por el trabajador2
Dado que conforme “base de datos online de la AFIP Mis Aportes”3 Uds. han
omitido depositar las retenciones practicadas 4 sobre mis remuneraciones de
“los meses de ... hasta ... inclusive”, “las quincenas de ...” con destino a
"seguridad social” (figura en condición de “impago") y “obra social” (figura en
condición de “pago parcial")5, los intimo para que dentro del plazo previsto por
dicha normativa exhiban comprobantes que acrediten la regularización de
dicha irregularidad mediante el depósito total de las retenciones practicadas
con más sus intereses, bajo apercibimiento de ley6.

1
Referencias normativas. Art. 132 bis (texto según ley 25.345, art.
43, LCT); art. 1o, decr. 146/01. Referencia jurisprudencial. “Es
improcedente condenar al empleador al pago de la sanción prevista en
el art. 132 bis de la ley de contrato de trabajo, si el trabajador no
concretó una intimación en la cual invocara la existencia de aportes
retenidos de su remuneración que no hubieran sido depositados y en
la que requiriera en forma explícita al empleador que efectuara dicho
depósito conforme lo dispone el art. 1° del decr. 146/01” (CNTrab,
Sala II, 19/10/10, "Diácono, Valeria I. c/Found Over SA y otros",
LLonline, AR/JUR/78710/2010).

2
Los presupuestos de hecho que tipifica el art. 132 bis para la
procedencia de la sanción están dados por a) retención de haberes
efectuada por el empleador, para cumplir con depósitos con destino a
la seguridad social (aportes a cargo del trabajador al sistema de obras
sociales -ley 23.660- o seguridad social -ley 24.241-) u otros a los
cuales estuviere obligado el trabajador (los indicados por la LCT, art.
132); b) incumplimiento del empleador con el depósito de estas
retenciones al momento de la extinción del contrato por cualquier
causa; c) intimación cursada por el trabajador al empleador, para que
dentro del término de treinta días corridos “ingrese los importes
adeudados, más los intereses y multas que pudieren corresponder”, y
d) persistencia de la falta completa del depósito luego de los treinta
días de recibida la intimación.

3
Aquí corresponde precisar la fuente de información consultada
por el trabajador y con la cual verificó la falta de depósito de las
retenciones practicadas sobre su salario. La referencia a la base de
datos online de la AFIP “Mis Aportes” es por tratarse de la
mayormente consultada para estos casos, pero también pueden ser
obtenidos por el trabajador ante la ANSeS. Cuando se trata de otro
tipo de retenciones (cuota sindical), las bases de datos deben ser
consultadas con los destinatarios de las retenciones no depositadas
(asociación sindical).

4
Referencia jurisprudencial. "Es improcedente condenar al
empleador al pago de la sanción prevista en el art. 132 bis de la ley de
contrato de trabajo, pues no se acreditó la existencia de retención de
aportes por parte de aquél, ello aun cuando la relación laboral no se
encontraba registrada, en tanto que dicha irregularidad podría ser
objeto de otro tipo de sanción pero no de la contenida en mentada
normativa" (CNTrab, Sala II, 31/8/10, “Ortiz, Félix V. c/Embajada de
la República de Corea”, LLonline, AR/JUR/50548/2010).

5
La intimación tiene por objeto principal requerir el cumplimiento,
por parte del deudor, dentro del plazo de treinta días que fija la
reglamentación.

6
La consecuencia prevista por la norma es una “sanción”
conminatoria a cargo del empleador y a favor del trabajador,
"equivalente a la remuneración que se devengaba mensualmente a
favor de este último al momento de operarse la extinción del contrato
de trabajo” y ello “hasta que el empleador acreditare de modo
fehaciente haber hecho efectivo el ingreso de los fondos retenidos”.

Respuesta
Contestando su requerimiento, debemos rechazar sus bases fácticas y la
intimación que en ellas se apoya, dado que nos hallamos al día 7 con el
cumplimiento del depósito de las retenciones que indica. Los certificados y
constancias pertinentes se encuentran a su disposición.
Por lo tanto, se deben precisar las remuneraciones sobre las cuales se
practicaron las retenciones (con mención de los períodos) y el destino con el
cual dichas retenciones fueron practicadas, esto es, para ser depositadas con
tal o cual imputación. Los datos resultarán de los duplicados de recibos de
remuneración en poder del trabajador. Si no hay retención de haberes (caso
de empleo clandestino o no registrado o períodos impagos de remuneración)
no procede esta sanción.

7
La norma exige “el efectivo ingreso de los fondos retenidos". De
manera tal que el empleador sólo podría válidamente eximirse de las
consecuencias sancionatorias, ingresando el total de los importes
retenidos con más los intereses que exija la autoridad administrativa y
a partir del momento en que así lo acredite en el expediente donde
tramite el reclamo del trabajador. La norma no exime de
consecuencias sancionatorias al empleador, por el hecho que éste
hubiera incluido la deuda de aportes (que no es suya propia, sino del
trabajador) en algún plan de facilidades de pago. La norma tampoco
ha previsto excepciones para el caso de concurso preventivo del
empleador, donde las restricciones al pago de deudas tienen que ver
con las propias del deudor y no con la administración o gestión de
recursos ajenos, como sería el dinero que retiene de los salarios
pagados a su personal. Ninguna situación de concurso preventivo
debiera justificar o posibilitar que el deudor se apropie del dinero que
no es suyo. Sin embargo, no fue entendido de ese modo en una
oportunidad, habiéndose resuelto: "Corresponde confirmar el
decisorio de grado en cuanto rechazó la multa peticionada por el
trabajador con sustento en el art. 132 bis de la ley de contrato de
trabajo, toda vez que la empleadora, ante la intimación que le fue
cursada, sostuvo que en el marco del concurso preventivo en el que se
encontraba, le resultaba imposible dar cumplimiento al pago de los
aportes afectados a la masa concursal hasta tanto dicha operación
fuera autorizada por el juez del concurso, sin que se hubiere
demostrado la existencia de autorización y posterior omisión del
principal de cumplir con las obligaciones a su cargo” (CNTrab, Sala VI,
15/6/10, “Godoy, Roberto A. c/Publicidad Enrique César Silvetti SA”,
LLonline, AR/JUR/32791/2010).

LA LIQUIDACIÓN DEL OBJETO DEL RECLAMO

A) Remuneraciones y liquidación final

Remuneración percibida y devengada


“Remuneración”, “percibida” y "devengada” son conceptos que utilizaremos
con frecuencia en este capítulo. "Remuneración” es la contraprestación en
dinero por el trabajo dependiente (art. 103, LCT; art. 60, ley 24.241, y
convenio OIT 95, sobre la protección del salario, art. Io), aquello que es
"debido” por el empleador al trabajador como consecuencia de su contrato de
trabajo y que no siempre coincide con lo efectivamente pagado o "percibido”
por el dependiente. "Remuneración”, a los fines prácticos, es el género, tanto
la contraprestación que “debe” el empleador como aquello que es percibido por
el trabajador. Las dos especies que utilizaremos aquí, sólo con fines prácticos,
están dadas en la "remuneración devengada” como aquella que es debida por
el empleador al trabajador (art. 119, LCT) y la "remuneración percibida”, que
es el importe en dinero efectivamente recibido por el trabajador como
consecuencia de su contrato de trabajo. Ambas pueden coincidir, cuando el
empleador ajusta lo pagado ("percibida”) a lo debido (“devengada”). También
hay coincidencia cuando lo pagado es por monto superior a lo debido, ya que
las normas laborales son del tipo “mínimo" y pueden ser válidamente
superadas por las condiciones particulares de trabajo. Pero cuando lo pagado
("percibido”) es por monto inferior a lo debido (“devengado”), será un "pago
insuficiente” (art. 260, LCT) y "a cuenta” de lo "devengado”, generando el
derecho del trabajador al reclamo de diferencias salariales, aun si hubiera
suscripto los recibos sin reservas. Las retenciones obligatorias (art. 132, LCT)
practicadas sobre el salario, tienen por finalidad atender obligaciones del
trabajador que, por lo general, tienen por destinatarios a terceros (organismos
de seguridad social, sindicato, mutual, etc.) y, por tal motivo, no inciden en los
aspectos cuantitativos de las obligaciones que generan los rubros aludidos en
el presente capítulo. De allí que la distinción entre remuneración "bruta”
(aquella anterior a las retenciones obligatorias) y “neta” (la efectivamente
percibida por el trabajador luego de practicadas) será indiferente en nuestros
cálculos, todos ellos utilizarán la remuneración "bruta”. En consecuencia,
entenderemos por remuneración devengada, el importe bruto, sin las
retenciones o descuentos que debe practicar el empleador y que no afectan la
cantidad del salario que debe pagarse al trabajador, aun cuando no lo perciba
íntegramente, porque debe cumplir previamente con otras obligaciones.

a) Trabajador mensualizado
"Mensualizado” alude a la modalidad de determinación del salario vigente en el
caso, que es “por tiempo” de trabajo (art. 104, LCT) y ese “tiempo” está dado
por la jornada legal o convencional en un mes calendario completo. La
remuneración que debe ser pagada (art. 119, LCT) o “remuneración
devengada”, en el caso es como mínimo la fijada en las escalas salariales
complementarias al CCT de que se trata. Utilizaremos dos liquidaciones del
salario sobre la base de dos casos de trabajadores mensualizados, tal como
son fijados por el respectivo convenio y sus escalas complementarias.

1) Empleado de comercio (CCT 130/75)


El CCT no requiere de prueba en juicio, pero debe ser debidamente
individualizado (art. 8o, LCT). En el ámbito de la Ciudad de Buenos Aires,
predomina un criterio estricto, habiéndose resuelto: "El principio iura novit
curia sólo alcanza a las normas de origen estatal, que se presumen por todos
conocidas, no a las que, como el convenio colectivo de trabajo, son de génesis
contractual” (CNTrab, Sala I, 21/12/92, “Origaen, Miguel E., y otros c/EFA”,
DT, 1993-B-1624). Ese criterio se basa en utilizar la limitación del art. 8° de la
LCT, para asignarle un alcance acotado o restringido por este artículo. Si el
CCT constituye una norma, una fuente de regulación del contrato de trabajo,
debe seguirse un criterio amplio en su aplicación, que se condiga con la
consagrada irrenunciabilidad de los derechos, con ese alcance, por el art. 12
de la LCT. El llamado principio de congruencia no es propio del proceso laboral,
sino más bien del proceso civil, donde la controversia es entre iguales. Su
aplicación al proceso laboral termina conspirando con la aplicación de las
normas de fondo, tal como lo hemos expuesto en otros trabajos (Trabajo
decente versus proceso laboral, LL, 2010-B-1081, y La aplicación de las
normas laborales a la luz de los objetivos estratégicos trazados por la OIT,
“Revista de Direito do Trabalho”, Editora Revista dos Tribunais, San Pablo,
Brasil, año 35, n" 133, ene.-mar. 2009, p. 301 a 321). Con mejor criterio se
ha interpretado: “Las normas emergentes de una convención colectiva de
trabajo son auténtico derecho positivo, por lo que en virtud del principio iura
novit curia corresponde traerlas ex oficio a consideración, máxime teniendo en
cuenta que el art. Io de la LCT establece que la relación laboral y el contrato de
trabajo se rigen, entre otras normas, por las convenciones colectivas o laudos
con fuerzas de tales -inc. c, art 1o, LCT-, sin que sea necesario diligenciar
prueba alguna para acreditarlo” (STJ Chaco, 26/4/02, “A., F. C. c/ICSA, y
otros”, LLon- line, AR/JUR/7012/2002). La única limitación que debiera existir
al respecto, tiene que ver con la aplicabilidad del CCT sólo cuando “contengan
normas más favorables a los trabajadores", aspecto que incidirá en el modo de
cálculo de remuneraciones y otros rubros, al computar como "remuneratorios"
rubros que, según la negociación colectiva, se le asigna carácter de “no
remuneratorio”. Volveremos sobre este último tópico.

a) Cálculo de la mejor remuneración mensual devengada, según escalas


salariales complementarias al CCT 130/75, vigentes al mes de noviembre de
2016
La liquidación que practicamos incluye, como “remuneratorios”, los rubros que
en la negociación colectiva no tienen ese carácter. La inclusión tiene por objeto
contemplar la incidencia de estos rubros, en aquellos otros que luego se
calculan sobre la base de la remuneración "devengada” (vacaciones, SAC,
indemnizaciones) y su fundamento normativo está dado por las razones
anteriormente explicadas que surgen del art. 8° de la LCT (el carácter “no
remuneratorio” constituye una condición menos favorable para el trabajador y
que, como tal, no debiera aplicarse con fines de disminuir los demás rubros
vinculados al salario).

Categoría profesional: Vendedor B.


La categoría profesional es el módulo utilizado por el CCT para la fijación de
salarios (arts. 37 y 172, LCT), mediante la descripción de tareas para cada una
de ellas, que representan el diferente valor económico del trabajo (art. 4o,
LCT).
Antigüedad: 1 año
La antigüedad en el empleo constituye un parámetro habitualmente utilizado
para diferenciar salarios y otras condiciones de trabajo. Su objetividad permite
mejorar la condición del trabajador con más trayectoria en su empleo (tal el
caso de las vacaciones, art. 150, LCT). Muchos CCT la reciben de ese modo,
estableciendo remuneraciones “adicionales” o "accesorias” en función de ella,
que deben ser pagadas juntamente con la remuneración principal (art. 127,
LCT). El art. 18 de la LCT es el que determina el cómputo de la antigüedad. En
nuestro ejemplo, el trabajador tiene un año de antigüedad cumplido a la época
de determinación del salario.
Mejor remuneración mensual devengada (noviembre 2016)
1. Salario básico vendedor "B” s/escalas CCT 130/75
2. Adicional “no remuneratorio” acuerdo anterior
3. Adicional “no remuneratorio” acuerdo junio 2010
4. Antigüedad de 1 año cumplidos (0,75% por año)
5. Premio presentismo, art. 40, CCT 130/75 (1 + 2+3 + 4 /12)
Subtotal sin SAC
6. Incidencia del SAC
Subtotal con SAC
$ 1.759,58 $ 452,67
$ 589,25
$ 21,01
$ 235,21
$ 3.057,72 $ 254,81
$ 3.312,53

b) Remuneración “percibida" en el mismo mes


En caso de trabajadores registrados será la resultante del recibo de
remuneración correspondiente. Por las razones antes indicadas puede coincidir
o no con la devengada. En caso de incumplimiento de registro total será la
efectivamente pagada al trabajador por el período de que se trate. En caso de
incumplimiento de registro parcial (falso monto de remuneraciones) será la
remuneración registrada que surge del recibo de remuneración, con más el
complemento pagado sin registrar. Para nuestro ejemplo, utilizamos un caso
de liquidación de haberes por valor inferior al devengado. La remuneración
"percibida”, con mayor incidencia en las liquidaciones a practicar más
adelante, será la "última”, esto es, la correspondiente al último período
liquidado y pagado por el empleador.

Remuneración percibida (noviembre 2016)


1. Sueldo “jornada reducida” $ 877,75
2. Pasaje a remunerativo $ 95,96
3. Antigüedad $ 6,58
4. Comisiones $ 276,92
5. Horas adicionales$ 159,59
6. Presentismo $ 118,07
7. Adicional “no rem.” acuerdos anteriores $ 330,00
8. Pasaje a remunerativo -$ 82,50
9. Bonus asistencia perfecta $ 100,00
10. Adicional no rem. Ac. Jun 2010 $ 276,60
11. Adicional no rem. Ac. Jun 2010 $ 49,50
12. Presentismo no rem. $ 47,80
Total percibido $ 2.256,27
2) Gastronómico
a) Cálculo de la mejor remuneración mensual devengada, según escalas
salariales complementarias al CCT 389/04, vigentes al mes de septiembre de
20016
Fecha de ingreso: 10/8/00 Fecha de egreso: 5/3/10
Categoría profesional: Gastronómico, camarero, cat. 2, CCT 389/04 (hotel
categoría “A”).
Mejor remuneración devengada (septiembre 2016)
1. Sueldo básico $ 2.064,90
2. Antigüedad $ 123,90
3. Complemento de servicio $ 247,80
4. Adicional Ciudad de Buenos Aires $ 123,90
5. A cta. aumentos aplicables a AHRCC $ 310,00
6. A cta. aumentos aplicables a AHRCC $ 310,00
Subtotal sin SAC $ 3.180,50
7. Incidencia de SAC $ 265,04
Mejor remuneración devengada c/incidencia
de SAC $ 3.445,54

b) Trabajador jornalizado
“Jornalizado” alude a la modalidad de determinación del salario vigente en el
caso. También es “por tiempo” de trabajo (art. 104, LCT), sólo que ese
“tiempo” está determinado en lapsos menores al mes, por lo general, por hora
o día de trabajo (de allí proviene su denominación, ya que tradicionalmente se
asimila "jornada” al trabajo realizado con la luz del día, que es el intervalo
existente entre el alba y el crepúsculo) dado por la jornada legal o
convencional de trabajo en un mes calendario completo. La determinación del
salario sobre la base de un "jornal horario” es la que predomina para los
obreros de la industria (textiles, madereros, caucho, alimentación, algunas
ramas de la indumentaria o vestido, entre otros). El "jornal diario" está menos
generalizado (relojeros, alguna rama de la indumentaria o vestido). Para
ambos casos, por lo general predomina la liquidación de esos en forma
“quincenal”. Las bases de cálculo predominantes en nuestra materia, están
dadas por la remuneración "mensual”. En los casos de trabajadores
“jornalizados”, a dicha base se llega a partir del cómputo de la cantidad de
horas o días trabajados en un mes.

1) Obrero de la industria textil


a) Cálculo de la mejor remuneración mensual devengada, según escalas
salariales complementarias al CCT 500/07, vigentes al mes de septiembre de
2016
Por iguales razones a las indicadas en el supuesto anterior, también aquí
incluimos con carácter “remuneratorio” los rubros que en la negociación
colectiva no tienen ese carácter.
Fecha de ingreso: 15/2/10 Fecha de egreso: 26/10/16
Categoría profesional: Operario textil, tareas generales, enconados cat. “C”,
CCT 500/07 (rama medias).
Horario de trabajo: Tres turnos rotativos, de rotación semanal, turno A, de
lunes a sábados de 06.00 a 14.00 hs.; turno B, de lunes a viernes de 14.00 a
22.00 hs., y turno C, de lunes a viernes de 22.00 a 06.00 hs. del día siguiente.
La remuneración se calcula en base al jornal fijado para la categoría “C” en
escalas salariales complementarias al CCT 500/07 ($ 8,57); horas normales
(195), premio al presentismo del 20% al cual era acreedor el actor por cumplir
los requisitos del convenio (art. 23) y la asignación “no remunerativa" fija de $
500, acordada en escalas salariales complementarias al convenio.
Mejor remuneración mensual devengada (septiembre de 2016)
1. Horas ordinarias (195 hs. x $ 8,57) $1.671,15
2. Antigüedad (195 hs. x $ 8,57) $ 25,35
3. Presentismo (20% sobre 1 + 2) $ 339,30
4. Asignación “no remunerativa” del CCT 500/07 $ 500,00
Subtotal sin SAC $ 2.535,80
6. Incidencia del SAC $ 211,32
Mejor remuneración mensual con SAC $ 2.747,12
b) Remuneración percibida en el mismo mes
Remuneración percibida (septiembre de 2016)
Primera quincena
1. Horas ordinarias (97 hs. x $ 8,57) $ 831,29
2. Antigüedad (97 hs. x $ 8,57) $ 12,61
3. Presentismo (20% sobre 1 + 2) $ 168,78
Total percibido en 1a quincena $ 1.012,68
Segunda quincena
4. Horas ordinarias (98 hs. x $ 8,57) $ 839,86
5. Antigüedad (98 hs. x $ 8,57) $ 12,74
6. Presentismo (20% sobre 1 + 2) $ 170,52
7. Asignación "no remunerativa” del CCT 500/07 $ 500,00
Total percibido en 2a quincena $ 1.523,12
Remuneración percibida en el mes $ 2.535,80

2) Jornalizado remunerado por día


a) Cálculo de la mejor remuneración mensual devengada, según escalas
salariales complementarias al CCT 533/07, vigentes al mes de agosto de 2010
Fecha de ingreso: 15/2/10
Fecha de egreso: 26/10/16
Categoría profesional: Oficial (relojero).
Horario de trabajo: lunes a viernes de 08.00 a 17.00 hs.
La remuneración se calcula en base al jornal diario fijado para la categoría
“oficial” en escalas salariales complementarias al CCT 533/07 ($ 97); días
normales trabajados en el mes de septiembre de 2010 (20), un día feriado,
premio al presen- tismo del 10% al cual era acreedor el actor por cumplir los
requisitos del convenio (art. 17) y el adicional por antigüedad (en el ejemplo
dos años) que su art. 24 determina en el importe de medio jornal diario (4
horas) correspondientes a la categoría de aprendiz ($ 8,73 la hora, según las
mismas escalas).
Mejor remuneración mensual devengada (septiembre de 2010)
1. Básico días trabajados (20 x $ 97) $ 1.940,00
2. Feriado (1 x$ 97)$ 97,00
3. Presentismo, art. 17, CCT (10% sobre 1 + 2) $ 203,70
4. Adic. Antigüedad 2 años, art. 24, CCT
[(4 x $ 8,73 x 2)] $ 69,84
Subtotal sin SAC $ 2.310,54
5. Incidencia del SAC $ 192,55
Mejor remuneración mensual con SAC
$ 2.503,09

b) Remuneración percibida en el mismo mes


Para diferenciar ambas remuneraciones utilizaremos, como ejemplo, un caso
de liquidación de haberes sobre falsa categoría profesional (medio oficial en
lugar de oficial).
Remuneración percibida (septiembre de 2016)
1. Básico días trabajados Medio Oficial (20 x $ 91) $ 1.820,00
2. Feriado (1 x $ 91) $ 91,00
3. Presentismo, art. 17, CCT (10% sobre 1 + 2) $ 191,10
4. Adic. Antigüedad 2 años, art. 24, CCT
[(4 x $ 8,73) x 2] $ 69,84
Total percibido en el mes $ 2.171,94

c) Empleo no registrado
1) Remuneración devengada
En caso de omisión total de registro (art. 8o, LNE), la remuneración devengada
estará dada por la que está determinada por las escalas salariales
complementarias al CCT aplicable (art. 119, LCT), la pactada con el empleador
o percibida por el trabajador. De estas tres utilizaremos la "mejor” para
nuestros ejemplos, dejando a salvo los casos puntuales donde corresponde
aplicar otra remuneración. En los casos de remuneraciones variables, la mejor
remuneración mensual devengada del último año, será la utilizada para el
cálculo de la indemnización común por despido (art. 245, LCT) y aquellas otras
indemnizaciones vinculadas a ésta (art. 15, ley 24.013; arts. Io y 2°, ley
25.323, entre otras).

2) Remuneración percibida
En los casos de omisión total de registro (art. 8o, LNE), la remuneración
percibida será aquella pagada por el empleador.
En los casos de incumplimiento de registro por falso monto de remuneración
(art. 10, LCE), la remuneración percibida estará dada por aquella que fue
registrada en recibos de remuneración, duplicados y demás registros (art. 52,
LCT), con más el complemento pagado sin registrar, según al siguiente
ejemplo.

Última remuneración percibida (octubre 2016)


Remuneración registrada
1. Sueldo básico $ 2.722,00
2. Feriados trabajados $ 108,88
3. Antigüedad $ 228,47
4. Acuerdo “no remunerativo” $ 544,40
5. Redondeo $ 0,45
Subtotal remuneración registrada $ 3.604,20
Remuneración no registrada
6. Complemento pagado sin registrar $ 1.200,00
Total percibido $ 4.804,20

Liquidación de salarios adeudados (sobre jornada normal)


Para simplificar los ejemplos utilizaremos los anteriores y calcularemos
períodos completos de deuda salarial. En todos los casos, las liquidaciones se
hacen sobre la base de la mejor remuneración mensual devengada, quedando
su monto definitivo al cálculo que surja de la prueba pericial contable. Se trata
de liquidaciones confeccionadas por abogados en la práctica judicial. El monto
definitivo es el realizado por el perito contador y utiliza los valores vigentes a
cada mes. En los casos donde, por contingencias procesales (omisión de
exhibir registros, empleo clandestino, rebeldía del empleador, etc.), dicha
prueba no se produce, serán de aplicación las presunciones legales (arts. 53 y
55, LCT; arts. 71 y 86, LO; art. 39, LPBA).

a) Trabajador mensualizado
1) Empleado de comercio
Empleado de comercio (3 meses)
1. Salarios septiembre a noviembre 2016
($ 3.057,12 x3) $9.171,36
2. Incidencia de SAC s/el rubro $ 764,28
Total salarios c/SAC $ 9.935,64

2) Gastronómico
Gastronómico (2 meses)

1. Salarios agosto y septiembre de 2010


($ 3.180,50x 2) $6.361,00
2. Incidencia de SAC s/el rubro $ 530,08
Total salarios c/SAC $ 6.891,08

b) Trabajador jornalizado
1) Obrero de la industria textil (jornalizado por hora)
Obrero de la industria textil (4 meses)
1. Salarios junio a septiembre de 2016
($ 2.535,80x4) $10.143,20
2. Incidencia de SAC s/el rubro $ 845,27
Total salarios c/SAC $10.988,47

2) Oficial relojero (jornalizado por día)


Oficial relojero (3 meses)
1. Salarios agosto y septiembre de 2016
($2.310,54x3) $6.931,62
2. Incidencia de SAC s/el rubro $ 577,64
Total salarios c/SAC $ 7.509,26

c) Empleo no registrado
Empleo no registrado (3 meses)
1. Salarios agosto a octubre de 2016
($4.804,20x3) $14.412,60
2. Incidencia de SAC s/el rubro $ 1.201,05
Total salarios c/SAC $ 15.613,65

d) Comisiones y salarios por unidad de obra


Constituyen “remuneración” conforme lo determina el art. 108 de la LCT y
deben ser liquidadas "sobre las operaciones concertadas”. Los modos más
habituales de remuneración sobre la base de comisiones se fijan por un
porcentaje sobre el importe de las ventas concertadas por el trabajador
(viajantes de comercio, vendedores de automóviles, etc.) o del producido por
los servicios brindados por éste (peluqueros) o un importe fijo por operación.
Esta modalidad remuneratoria se encuentra regulada en el art. 112 de la LCT y
consiste en la retribución mediante un importe fijo, por cada pieza o medida
producida con intervención del trabajador. Su liquidación, no difiere de los
casos donde la comisión es pactada mediante una retribución fija por cada
operación.

1) Comisiones adeudadas (fijadas en porcentaje sobre ventas) Liquidación de


comisiones adeudadas
(porcentajes sobre ventas)
Rubro Ventas % Importe
1. Comisiones adeudadas
en agosto 2016 $ 15.000 5 $ 750,00
2. Comisiones adeudadas
en septiembre 2016 $ 28.500 5 $ 1.425,00
3. Comisiones adeudadas
en octubre 2016 $ 23.400 5 $1.170,00
Subtotal $ 3.345,00
4. Incidencia de SAC s/el rubro $ 278,75
Total del rubro c/SAC $ 3.623,75

2) Salarios por unidad de obra o comisiones a importe fijo por operación


Salarios por unidad de obra o comisiones a importe fijo
Rubro Cant. $ Importe
1. Salarios / comisiones adeudadas
en agosto 2016 300 15 $ 4.500,00
2. Salarios / comisiones adeudadas
en septiembre 2016 320 15 $ 4.800,00
3. Salarios / comisiones adeudadas
en octubre 2016 280 16 $ 4.480,00
Subtotal $ 13.780,00
4. Incidencia de SAC s/el rubro $ 1.148,33
Total del rubro c/SAC $ 14.928,33

Liquidación de diferencias salariales (sobre jornada normal)


Los reclamos por diferencias salariales tienen como base normativa a lo
dispuesto por el art. 260 de la LCT. Su causa es por el pago de una
remuneración en valores inferiores a los debidos. En nuestros ejemplos
trataremos la diferencia entre la remuneración percibida (liquidada en valor
inferior por el empleador) y la devengada (aquello que se debió pagar). Son
diferencias calculadas sobre la “jornada normal”, atendiendo a que las
liquidaciones de horas extras (completas o sus diferencias) se practican por
separado.

a) Liquidación a valores históricos


Esta modalidad es la utilizada regularmente por el perito contador en su
informe. Puede ser utilizada en la fundamentación del reclamo, incluida en la
demanda judicial. La columna “devengado” refleja la remuneración que se
adeuda al trabajador, previamente calculada sobre la base de los diferentes
rubros que la componen (dentro de la jornada normal de trabajo) en las
escalas salariales de aplicación. La columna “percibido” es completada con la
remuneración pagada, en valor inferior, por el empleador, sea la que resulta
de los registros (empleo registrado) o la informada por el trabajador (empleo
clandestino). Las diferencias entre una y otra pueden provenir por las causas
más diversas, sea por el simple pago de valores inferiores o por la liquidación
sobre la base de parámetros incorrectos (falsa fecha de ingreso, falsa
categoría profesional, cantidad de horas inferior a las efectivamente trabajadas
en casos de trabajadores jornalizados, etcétera).

Diferencias salariales
Período Devengado Percibido Diferencia
1. Ene-16 $ 1.395,65 $ 850,00 $ 545,65
2. Feb-16 $ 1.395,65 $ 850,00 $ 545,65
3. Mar-16 $ 1.395,65 $ 1.000,00 $ 395,65
4. Abr-16 $ 1.395,65 $1.000,00 $ 395,65
5. May-16 $ 1.395,65 $1.000,00 $ 395,65
6. Jun-16 $ 1.395,65 $1.000,00 $ 395,65
7. Jul-16 $ 1.395,65 $1.000,00 $ 395,65
8. Ago-16 $ 1.395,65 $ 1.000,00 $ 395,65
9. Set-16 $ 1.395,65 $ 1.150,00 $ 245,65
10. Oct-16 $ 1.395,65 $1.150,00 $ 245,65
11. Nov-16 $ 1.395,65 $ 1.150,00 $ 245,65
Subtotal diferencias adeudadas $4.202,15
12. SAC s/diferencias $ 350,18
Total del rubro c/SAC $ 4.552,33

b) Liquidación estimativa para el reclamo


Es la modalidad habitualmente utilizada para fundamentar el reclamo en una
demanda judicial. Sobre todo en los casos de diferencias por importes
mensuales similares o constantes y en períodos prolongados, de hasta el plazo
de la prescripción (veinticuatro meses). Se trata de un cálculo estimativo que
tiene por base la diferencia adeudada en un mes y luego ese valor es
multiplicado por el período del reclamo. El cálculo definitivo se difiere al que
realizará el perito contador en su informe. Con ello, se evita recurrir al reclamo
por monto indeterminado y se logra un mecanismo que cubre el requerimiento
que exigen la LO (art. 65, inc. 3) y la LPBA (art. 26, inc. f). El cálculo
estimativo reviste utilidad práctica, tanto en la conciliación como para el
juzgador al resolver sobre los montos del reclamo (art. 56, LO; art. 47, LPBA).
Diferencias salariales Período: septiembre de 2010 a agosto de 2016 (24
meses + SAC)
1. Remuneración devengada en agosto de 2016
(s/jornada normal) $ 4.693,20
2. Remuneración percibida en agosto de 2016 $ 3.125,40
3. Diferencia salarial mensual en agosto de 2016
(s/jornada normal) $ 1.567,80
4. Diferencia en 24 meses a valores constantes $ 37.627,20
5. SAC s/rubro $ 3.135,60
Total diferencias salariales c/SAC $ 40.762,80

Liquidación de horas extras


Conforme a lo normado por el art. 1o de la ley 11.544 y el art. 196 de la LCT,
la jornada de trabajo diurna no podrá exceder de las cuarenta y ocho horas
semanales ni de las ocho horas diarias (que pueden llegar a 9 horas diarias en
el caso de distribución desigual de horas, para posibilitar el "sábado inglés").
Para la jornada nocturna (de 21.00 a 6.00 hs.), rige la limitación de las siete
horas o cuarenta y dos semanales. En jornada mixta (diurna - nocturna), cada
hora nocturna equivale a una hora con ocho minutos de trabajo diurno (art.
200, LCT; art. 2o, ley 11.544). La jornada de trabajo prestado en condiciones
declaradas insalubres por la autoridad administrativa es de seis horas diarias o
treinta y seis semanales y cada hora de trabajo insalubre equivale a una hora
con treinta y tres minutos de trabajo no insalubre (siempre y cuando el trabajo
insalubre sea de hasta tres horas por día). En trabajo por turnos o por
equipos, la jornada se mide en ciclos de tres semanas, no pudiendo exceder de
ciento cuarenta y cuatro horas ni de cincuenta y seis horas la semana.
En estos casos, cuando los turnos coinciden con horario nocturno, por cada
siete días de 8 hs. de trabajo nocturno, rige un descanso compensatorio de un
día (art. 9°, decr. 16.115/33). Según el art. 201 de la LCT, el empleador
deberá pagar al trabajador que prestare servicios en horas suplementarias un
recargo del 50% calculado sobre el salario habitual si se tratare de días
comunes, y del 100% en días sábado después de las 13 hs, domingo y
feriados. El trabajo prestado en estos últimos días, dentro de la jornada legal,
no genera recargo alguno, solo derecho a descanso compensatorio.
Las horas trabajadas en exceso de la jornada pactada, que no superen la
jornada legal, deben ser pagadas a valor simple y sin recargo (CNTrab, en
pleno, n" 226, 25/6/81, “D’Aloi, Salvador c/Selsa SA”, DT, 1981-997).

a) Trabajador mensualizado
1) Horas extras en jornada diurna
Tratándose de jornada con trabajo en días sábados y domingos y descanso
compensatorio en la semana, no se agrega el recargo del 100% y sólo el
recargo simple del 50%. El mes promedio de 30,4 días es el utilizado por el
legislador para calcular el "ingreso base” en el art. 12.2 de la LRT.
Remuneración devengada en jornada legal: $ 3.057,12 (sin SAC)
Horario de trabajo: 6 días de trabajo a la semana, de 11.00 a 20.00 hs., con
un franco los miércoles. Cubría de ese modo 54 horas de trabajo efectivo en la
semana (9 hs. diarias x 6 días), que representan 6 hs. extras a la semana al
50%. En un mes promedio de 30,4 días (365/12), las 6 horas extras a la
semana representan 26 horas extras al 50% al mes, de acuerdo al siguiente
cálculo: 6/7 x 30,4.
Período del reclamo: últimos 24 meses 4- SAC.
Liquidación de horas extras Período 1/10/08 al 31/10/10, 24 meses + SAC
1. Horas extras al 50% trabajadas al mes: 26
2. Horas extras al 50% trabajadas: 624 en 24 meses (x24)
3. 624 horas extras al 50%
($ 3.057,12/208x624 x 1,5) $ 13.757,04
Subtotal $ 13.757,04
4. Incidencia de SAC sobre el rubro
($ 13.757,04 / 12) $ 1.146,42
Total adeudado c/SAC $14.903,46
Donde:
"50%” y “1,5" es el recargo del 50% que surge del art. 201 de la LCT.
"26” son las horas extras de trabajo al mes, calculadas en función de las seis
horas extras de trabajo a la semana.
"624” el número de horas extras del período del reclamo, resultante de
multiplicar por los veinticuatro meses las 26 horas mensuales.
"$ 3.057,12” es la remuneración mensual devengada, utilizada como base de
cálculo, ésta remunera el trabajo por la jornada legal o vigente en el caso
concreto.
"208” es la cantidad de horas normales en un mes promedio de 30,4 días,
cumpliendo la jornada legal diurna de cuarenta y ocho hs. semanales. Resulta
del siguiente cálculo: 48 / 7 x 30,4 = 208. El cálculo del valor hora simple,
resultará de dividir la remuneración devengada, por la cantidad de horas
normales trabajadas en el período de un mes. Cuando la jornada sea la legal
de trabajo diurno (cuarenta y ocho hs. semanales), dicha cantidad de horas en
un mes promedio será de doscientas ocho, aunque también suele utilizarse la
base de doscientas horas en la jornada mensual, calculando la disminución
resultante de un feriado promedio al mes. El criterio de las doscientas horas es
el utilizado por el art. 18 de la ley 23.660 para medir su alcance en relación al
salario base de aportes.

2) Horas extras en jornada mixta (diurna-nocturna)


Remuneración devengada en jornada legal: $ 3.312,53 (sin SAC).
Horario de trabajo: 6 días de trabajo a la semana, de 15.00 a 24.00 hs., con
un franco los jueves. Cubría de ese modo 9 horas diarias de trabajo, de ellas 6
en horario diurno y 3 en nocturno. En la semana, cubría 54 hs. de trabajo, 36
en horario diurno y 18 en nocturno.
Adicionando 8 minutos por cada hora de trabajo nocturno (18x8= 144)
resultan dos horas con 24 minutos adicionales de recargo (o 2,4 hs.) sobre las
54 horas de tiempo efectivo, con lo cual se cuenta una jornada computable de
56,4 horas de trabajo a la semana, de las cuales 8,4 son horas extras o en
exceso de las 48 horas normales de trabajo. En un mes promedio de 30,4 días
(365 / 12), las 8,4 horas extras a la semana representan 36,48 horas extras al
50% al mes, según el siguiente cálculo: 8,4 / 7 x 30,4.
Período del reclamo: últimos 14 meses + SAC.
Liquidación de horas extras Período 14 meses + SAC
1. Horas extras al 50% trabajadas al mes: 36,48
2. Horas extras al 50% trabajadas
en 14 meses (x 14): 510,72
3. 624 horas extras al 50%
($ 3.312,53/208x610,72 xl,5)
4. Incidencia de SAC sobre el rubro
$ 12.200,30 $ 1.016,69 $ 13.216,99
Total adeudado c/SAC

b) Trabajador jornalizado
Remuneración devengada: jornal horario básico $ 8,25 + adicional por
antigüedad $ 0,14 la hora + adicional por pre- sentismo del 20%.
Horario de trabajo: lunes a viernes de 08.00 a 18.00 hs. y sábados de 08.00 a
12.30 hs. Cumplía de este modo una jornada semanal de 54,5 horas, es decir,
6,5 horas en exceso de la jornada legal. En un mes promedio de 30,4 días,
computaba 208,5 horas normales (48 / 7 x 30,4) y 28,23 horas extras al 50%
(6,5 / 7 x 30,4).
Período del reclamo: 15/1/09 al 31/5/10 (16 meses y 17 días).
Liquidación de horas extras
Período 15/1/09 al 31/5/10,
16 meses y 17 días
1. Horas extras al 50% trabajadas al mes: 28,2
2. Horas extras al 50% trabajadas
en 16 meses (x 16) 451,7
3. Horas extras al 50% trabajadas
en 17 días (28,2/30,4 x 17) 15,8
Horas extras adeudadas por el período 467,5
4. 467,5 horas extras al 50%
[($ 8,25 + $ 0,14) x 467,5 x 1,2 x 1,5] $ 7.059,68
5. Incidencia de SAC sobre el rubro $ 588,31
Total adeudado c/SAC $ 7.647,99

La liquidación final por egreso


a) La remuneración del mes de egreso (o despido)
Cuando la fecha de la extinción del contrato de trabajo, mediante la recepción
por el destinatario de la comunicación rescisoria, no coincide con el último día
del mes, corresponde la liquidación del salario correspondiente a la última
fracción del período trabajada hasta esa fecha. El rubro adopta distintas
denominaciones, como ser “remuneración del mes del despido” o "del mes de
egreso”. Como contraprestación por trabajo efectivo, tiene carácter
remuneratorio. Genera, por lo tanto, contribuciones a cargo del empleador y
aportes a cargo del trabajador. Devenga, asimismo, la proporción del SAC.

1) Trabajador mensualizado
Remuneración devengada en jornada legal: $ 3.057,12 (sin
SAC).
Fecha de egreso: 18 de enero de 2016.
Período a liquidar: 18 días.
Remuneración mes egreso (mensualizado)
18 días ($ 3.057,12/31 x 18)$ 1.775,10
Donde:
"$ 3.057,12” es la remuneración devengada por un mes de trabajo completo.
"31” son los días del mes de enero.
"18” la proporción del salario a calcular, que son los 18 días del mes del
egreso.
“$ 1.775,10” es la remuneración devengada por el período de 18 días
calculado.

2) Trabajador jornalizado
Remuneración devengada: jornal horario básico $ 8,25 + adicional por
antigüedad $ 0,14 la hora + adicional por presentismo del 20%.
Horario de trabajo: lunes a viernes de 06.00 a 14.00 hs. y sábados de 06.00 a
13.00 hs.
Fecha de egreso: 13 de enero de 2016.
Período a liquidar: 13 días (79 hs. normales y 8 hs. feriado).
Remuneración mes egreso (jornalizado)
1. 79 horas normales [($ 8,25+ $ 0,14) x 79 x 1,2] $ 795,37
2. 8 horas feriado 1/1/11 [($8,25 +$0,14) x 8 xl,2] $ 80,54
Total (sin SAC) $ 875,91
Donde:
“79” son las horas normales que resultan del horario de trabajo cumplido, en
los días que transcurren en la parte del mes a liquidar.
“$ 8,25” es el jornal horario fijado en escalas salariales complementarias de la
actividad.
“$ 0,14” es el adicional por antigüedad por hora de trabajo, fijado en escalas
salariales complementarias de la actividad.
"1,2” es el coeficiente con el cual se calcula el adicional por presentismo del
CCT, del 20%.

b) La indemnización por vacaciones no gozadas


El pago de las vacaciones no gozadas al momento de la extinción del contrato,
constituye una excepción a la prohibición del pago del descanso,
expresamente contemplada en el art. 162 de la LCT. El rubro tiene carácter
indemnizatorio por definición (art. 156, LCT) y, por lo tanto, no genera aportes
ni contribuciones. Sin embargo, como éste debe calcularse sobre la base de la
remuneración devengada y ella genera SAC, debe contemplarse su incidencia
sobre el rubro.

1) Cálculo de la cantidad de días proporcionales


La proporción de la que aquí nos ocupamos es la prevista por el art. 156 de la
LCT para el caso de extinción del contrato de trabajo por cualquier causa y
resulta aplicable para calcular la indemnización por este rubro. Se computan
como trabajados, también los días en que el trabajador no preste servicios por
gozar de una licencia legal o convencional, o por estar afectado a una
enfermedad inculpable o por infortunio en el trabajo, o por otras causas no
imputables a éste (art. 152, LCT). Según lo dispuesto por el art. 150 in fine de
la LCT, la antigüedad del trabajador contemplada para el cálculo de la cantidad
de días que le corresponden de licencia por el año calendario correspondiente,
es al 31 de diciembre. Dada su antigüedad mayor a cinco años que no excede
de diez en nuestro ejemplo (art. 150, inc. b, LCT). Debe tenerse en cuenta que
la cantidad de días de licencia anual fijados por la LCT son mínimos y pueden
ser ampliados o mejorados por negociación colectiva o individual.
Fecha de egreso: 24 de octubre.
Días del año trabajados hasta la fecha de egreso: 297. Antigüedad al 31 de
diciembre del año de egreso: 6 años. Licencia completa: 21 días corridos.
Cálculo de la licencia proporcional a indemnizar:
(297 x 21) / 365 = 17,09 días
La proporción es calculada utilizando una regla de tres simple, donde:
“291” es la cantidad de días trabajados, que generaron derecho a licencia
anual.
“21” es la cantidad de días de licencia anual que le hubieran correspondido por
el trabajo en el año completo, es decir, en los "365” días.
“17,09 días” es la cantidad de días de licencia proporcional no gozada, por
haber trabajado “297” días en lugar de los “365” requeridos para adquirir el
derecho a la licencia completa de 21 días.
2) Cálculo de la indemnización en remuneraciones mensuales homogéneas
El punto se encuentra regulado por el art. 155, inc. a, de la LCT. El valor de
cada día de licencia anual, resulta de dividir por veinticinco el importe de la
remuneración que perciba al momento de su otorgamiento.
Remuneración devengada en jornada legal: $ 3.057,12 (sin SAC).
Días de licencia anual proporcional a liquidar: 17,09. Cálculo: ($ 3.057,12 /
25) x 17,09 = $ 2.089,85 Donde:
“$ 3.057,12” es el valor de la remuneración devengada al momento del
cálculo, que en los supuestos de remuneraciones mensuales homogéneas (de
igual valor), coincidirá con la remuneración vigente al momento del egreso.
“25” es el coeficiente divisor que indica el art. 155, inc. a, de la LCT. No
coincide con la cantidad de días del mes, lo cual lleva a un resultado en dinero,
de monto superior al que correspondería por los 17,09 días de salario. La
diferencia en favor del trabajador mensualizado se denomina “plus
vacacional”. Con ella se equipara la situación del trabajador mensualizado
frente a la de los trabajadores remunerados por jornal (horario o diario) para
la liquidación de este rubro.

3) Cálculo de la indemnización en remuneraciones fijadas por jornal diario


La regulación está dada por el art. 155, inc. h, de la LCT. Debe liquidarse por
cada día de vacaciones, el importe que le hubiere correspondido percibir al
trabajador en la jornada anterior a la fecha en que comience en el goce de
éstas (en el caso, será la de egreso) tomando la remuneración que deba
pagarse por día.
Remuneración devengada por día: $ 130 (básico) + $ 10 (antigüedad) + $ 15
(presentismo), total $ 155.
Días de licencia anual proporcional a liquidar: 17,09.
Cálculo: $ 155 x 17,09 = $ 2.648,95
Donde:
"$ 155” es el valor de la remuneración devengada al momento del cálculo, por
cada día de trabajo.
“17,09” es la cantidad de días proporcionales que hemos calculado para
nuestro ejemplo.

4) Cálculo de la indemnización en remuneraciones fijadas por jornal horario


La regulación está dada por el art. 155, inc. b, de la LCT. Al igual que en el
caso anterior, debe liquidarse por cada día de vacaciones el que le hubiera
correspondido percibir al trabajador en la jornada anterior, sólo que aquí dicho
cálculo es realizado conforme la cantidad de horas habitualmente trabajadas
que, en el ejemplo, coincide con las ocho horas de la jornada legal. Cuando la
jornada cumplida supera dicha cantidad de horas, debe liquidarse hasta el
máximo de nueve horas por día, teniendo en cuenta que las horas que
exceden son horas extras, por naturaleza variables y su incidencia debe
calcularse del mismo modo que las remuneraciones variables, tal como
veremos seguidamente. La norma también aclara que cuando por motivos
circunstanciales la jornada sea inferior a la legal, deberá tomarse esta última
para realizar el cálculo, esto es, las ocho horas del ejemplo.
Remuneración devengada: jornal horario básico $ 8,25 + adicional por
antigüedad $ 0,14 la hora + adicional por presentismo del 20%.
Horario de trabajo: lunes a viernes de 06.00 a 14.00 hs. y sábados de 06.00 a
13.00 hs.
Días de licencia anual proporcional a liquidar: 17,09.
Cálculo: [($ 8,25 + 0,14) x 1,2] x 8 x 17,09 = $ 1.376,50
Donde:
“($ 8,25 + 0,14) x 1,2” es la conformación de la remuneración devengada por
hora de trabajo.
“8” es el multiplicador utilizado para arribar a la jornada legal, que son 8 horas
diarias.
“17,09” es la cantidad de días proporcionales que venimos utilizando para
nuestro ejemplo.
Por las razones explicadas anteriormente, si la jornada cumplida supera las
ocho horas diarias, deberá modificarse este coeficiente, computando el
máximo de nueve horas de trabajo.
5) Cálculo de la indemnización en remuneraciones variables
A este punto se ocupa el art. 155, inc. c, de la LCT. Las remuneraciones
variables inciden en la modalidad del cálculo de la indemnización, tanto cuando
constituyen la modalidad principal de liquidación del salario (por ejemplo,
trabajador remunerado exclusivamente a comisión o destajo) como cuando
complementan remuneraciones homogéneas o fijas (horas extras). Nos
ocupamos del primero de dichos casos con un ejemplo de liquidación del rubro
en remuneraciones variables.
La opción entre uno y otro promedio es a beneficio del trabajador (art. 155,
inc. c, LCT).
Total de remuneraciones devengadas en el semestre anterior (sin SAC): $
18.342,72.
Promedio mensual, del semestre anterior: $ 3.057,12 (resultante de $
18.342,72 / 6).
Total de remuneraciones devengadas en el año anterior (sin SAC): $
35.241,24.
Promedio mensual, del año anterior: $ 2.936,77 (resultante de $ 35.241,24 /
12).
Mejor promedio a utilizar, el del último semestre: $ 3.057,12. Días de licencia
anual proporcional a liquidar: 17,09.
Cálculo: ($ 3.057,12 / 25) x 17,09 = $ 2.089,85
Donde:
“$ 3.057,12” es el valor de la remuneración mensual devengada al momento
del cálculo, que por tratarse de remuneraciones variables, es el mejor
promedio (del último semestre, en lugar del último año).
“25” es el coeficiente divisor que indica el art. 155, inc. a, de la LCT.
6) Cálculo de la indemnización en remuneraciones mixtas (fijas + variables)
Cuando el salario está compuesto por remuneraciones fijas, más otras
variables (comisiones, horas extras), estas últimas deben integrar la base de
cálculo del salario vacacional (art. 155, incs. c y d, LCT). Su liquidación debe
hacerse complementando ambas remuneraciones, la fija de acuerdo a lo que
hemos desarrollado en los puntos anteriores, más la variable, calculada en el
promedio que indica la norma.
Remuneración devengada: jornal horario básico $ 8,25 + adicional por
antigüedad $ 0,14 la hora + adicional por presentismo del 20% + horas extras
variables.
Horario de trabajo: lunes a viernes de 06.00 a 15.00 hs. + horas extras.
Días de licencia anual proporcional a liquidar: 17,09. Cálculo de salario sobre
horas normales:
[($ 8,25 + 0,14) x 1,2] x 9 x 17,09 = $ 1.548,56 Donde:
“($ 8,25 + 0,14) x 1,2” es la conformación de la remuneración devengada por
hora de trabajo.
“9” es el multiplicador utilizado para arribar a la jornada normal diaria, que en
el caso coincide con el máximo de 9 horas.
“17,09” es la cantidad de días proporcionales que venimos utilizando para
nuestro ejemplo.
Por las razones explicadas anteriormente, si la jornada cumplida supera las
ocho horas diarias, deberá modificarse este coeficiente, computando el
máximo de nueve horas de trabajo.
Cálculo de salario sobre remuneraciones variables:
Total de remuneraciones variables (horas extras en el caso) devengadas en el
semestre anterior (sin SAC): $ 9.116,65.
Promedio mensual, del semestre anterior: $ 1.519,44 (resultante de $
9.116,65/6).
Total de remuneraciones variables (horas extras en el caso) devengadas en el
año anterior (sin SAC): $ 17.156,15.
Promedio mensual, del año anterior: $ 1.429,68 (resultante de: $ 17.156,15 /
12).
Mejor promedio a utilizar, el del último semestre: $ 1.519,44. Cálculo: ($
1.519,44/25) x 17,09 = $ 1.038,69 Donde:
Indemnización resultante (fija + variable): $ 1.548,56 + $ 1.038,69 = $
2.587,25
La opción entre uno y otro promedio es a beneficio del trabajador (art. 155,
inc. c, LCT).
En el caso del ejemplo, el trabajador percibe además de las remuneraciones
fijas, otras accesorias que deben ser contempladas para el cálculo de la
indemnización. El art. 155, inc. b, así lo establece al disponer: "Si el trabajador
remunerado por día o por hora hubiese percibido además remuneraciones
accesorias tales como por horas complementarias, se estará a lo que prevén
los incisos siguientes”. El inc. c describe la modalidad de cálculo en
remuneraciones variables, que es la utilizada en el ejemplo. A su vez, el inc. d
clarifica la situación de los trabajadores mensualizados, para quienes también
deben ser contempladas en la liquidación, las remuneraciones variables a
promedio.

c) El SAC
El SAC es la doceava parte del total de las remuneraciones devengadas en un
año calendario (art. 121, LCT). Debe ser “pagado sobre el cálculo del 50% de
la mayor remuneración mensual devengada por todo concepto dentro de los
semestres que culminan en los meses de junio y diciembre de cada año” (art.
1o, ley 23.041).

1) Semestres completos
El SAC estará dado por el 50% de la mejor remuneración mensual devengada
por todo concepto en el semestre.
Mejor remuneración mensual devengada en el semestre: $ 3.057,12
SAC del semestre: $ 3.057,12 / 2 = $ 1.528,56

2) Semestres incompletos (pago proporcional)


Cálculo de SAC proporcional.
Fecha de egreso: 18 de octubre.
Mejor remuneración mensual devengada en el semestre: $ 3.057,12.
Remuneración proporcional del mes de egreso: $ 3.057,12 / 31x18 = $
1.775,10.
SAC proporcional 2a cuota: [($ 3.057,12 x 3) + $ 1.775,10]/ 12 = $ 912,21.
La procedencia del SAC proporcional se encuentra prevista por el art. 123 de la
LCT. A su vez, dicha proporción debe ser calculada sobre la base de la mejor
remuneración del semestre, tal como lo indica el art. Io de la ley 23.041.

3) Cálculo del SAC proporcional a otros rubros


a) Sobre rubros adeudados por conceptos salariales varios (horas extras,
diferencias salariales, comisiones adeudadas, etcétera).
Cálculo: $ 15.074,43 / 12 = $ 1.256,20.
Donde:
“$ 15.074,43” es el importe previamente liquidado por el rubro de que se
trate.
“12” es el divisor utilizado para calcular la doceava parte, que constituye el
SAC.
Cuando se trata de salarios completos adeudados, la proporción debe
calcularse sobre la base del mejor mes del semestre (art. 1o, ley 23.041). En
los demás casos, la proporción se calcula directamente sobre el total del rubro,
por aplicación de la regla general contenida en el art. 123 de la LCT.

b) Sobre vacaciones proporcionales no gozadas.


Cálculo: $ 2.089,85 / 12 = $ 174,15.
Donde:
“$ 2.089,85” es el importe previamente liquidado, de vacaciones
proporcionales no gozadas.
“12” es el divisor utilizado para calcular la doceava parte, que constituye el
SAC.
Es la proporción que resulta del art. 123 de la LCT. La mejor remuneración del
semestre es utilizada sólo para el cálculo del SAC correspondiente al semestre.
Cuando debemos estimar la incidencia del SAC sobre otros rubros que son
calculados sobre la remuneración devengada (no siempre la “mejor”),
utilizamos la proporción general que indica la referida norma.
El cálculo de la incidencia del SAC en otros rubros indemnizatorios se incluye a
continuación y en cada caso.
B) Indemnizaciones comunes, especiales y sanción, DERIVADAS DE
LA EXTINCIÓN DEL CONTRATO

Indemnizaciones comunes del despido


a) Integración del mes del despido
Referencia normativa. Art. 233, LCT. El rubro procede en los casos de
extinción del contrato de trabajo por voluntad del empleador, luego del
período de prueba, sin que medie preaviso y cuando la extinción se produce en
fecha que no coincide con el último día del mes. Está dada por el importe
equivalente al de los salarios por los días faltantes hasta el último día del mes
en que se produce el despido. Como es un importe equivalente a salarios y
éstos generan SAC, corresponde incluir su incidencia. Sobre el particular, se ha
resuelto: “Para calcular la indemnización del art. 15 de la ley 24.013, además
de considerar la reparación dispuesta en el art. 245 de la LCT y la sustitutiva
del preaviso, corresponde computar la integración del mes de despido, pues se
trata de un complemento de esta última, y la base de cálculo de estos últimos
rubros debe integrarse con la parte proporcional del sueldo anual
complementario” (CNTrab, Sala III, 16/9/96, "Bustamante, Silvia R.
c/Chaufan, Zoila, y otro”, LLonline, AR/JUR/5865/1996). En igual sentido, se
dijo: "Corresponde el pago del aguinaldo sobre la indemnización por falta de
preaviso y sobre la integración del mes de despido, pues, en el caso contrario,
no sería completa, por tratarse el sueldo anual complementario de un salario
diferido” (SCBA, 8/7/80, “Perrugoría, Abel A. c/Esso SA”, DT, 1980-1304).
Dado su carácter indemnizatorio, el rubro no genera aportes o contribuciones.
Remuneración mensual devengada: $ 3.057,12 (sin SAC). Fecha de egreso:
18 de enero de 2016.
Período a liquidar: 13 días.
Cálculo: $ 3.057,12 / 31 x 13 = $ 1.282,02.
Cálculo de la incidencia del SAC:
$ 1.282,02/12 = $ 106,83.
Donde:
"$ 3.057,12” es la remuneración mensual base de cálculo.
"31” es la cantidad de días correspondientes al mes del despido y por los
cuales hubiera correspondido la remuneración utilizada como base de cálculo.
"13” es la cantidad de días faltantes para completar el mes del despido.
"12” es el divisor utilizado para calcular la doceava parte, que constituye el
SAC (art. 123, LCT).
La remuneración que debe utilizarse para este cálculo es la mensual que
debiera percibir el trabajador en el mes del despido. No necesariamente será
la mejor. A falta de regulación específica, en caso de remuneraciones
variables, donde no pudiera determinarse un valor exacto, prevalece el criterio
de estar al promedio de los últimos seis meses, habiéndose resuelto: “Los
rubros indemnización sustitutiva del preaviso, integración del mes de despido,
vacaciones proporcionales no gozadas y aguinaldo proporcional deben
calcularse, en principio, conforme a la última remuneración percibida
correspondiendo, en caso de retribuciones variables, tomarse un promedio de
los últimos seis meses a valores actualizados” (CNTrab, Sala III, 28/10/91,
"Rojas, Marina N. c/Celoprint SA", DT, 1991-B-2206). Aunque también se ha
aplicado el criterio de un promedio más extendido: “A los fines de determinar
la suma que corresponde percibir al trabajador cuya remuneración era variable
en concepto de salarios adeudados, indemnización sustitutiva de preaviso y la
integración del mes de despido, corresponde tomar el promedio de las doce
últimas remuneraciones, por aplicación analógica de los arts. 255 y 208 de la
LCT” (CNTrab, Sala II, 8/4/08, “Gómez Manduca, Daniel O. c/Grupo Logístico
SRL”, LLonline, AR/JUR/2242/2008; id., Sala VII, 22/2/08, “Rodríguez, Antonio
c/HSBC Bank Argentina”, LLonline, AR/JUR/250/2008).
Son los faltantes hasta completar los treinta y un días del mes de enero,
contados a partir del vencimiento del día en que se produjo la extinción del
contrato, que en nuestro ejemplo fue el día 18 de dicho mes.

b) La indemnización sustitutiva del preaviso


El rubro se encuentra regulado por el art. 232 de la LCT y corresponde en los
casos de extinción voluntaria del contrato de trabajo, sin cumplimiento del
preaviso. La indemnización, que está a cargo de la parte que omite el
preaviso, resultará de un importe "equivalente a la remuneración que
correspondería al trabajador durante los plazos” del preaviso, fijados por el
art. 231 de la LCT con alcance general. En el régimen de la pequeña empresa,
el preaviso tendrá siempre la duración de un mes, contado a partir del día
siguiente al de la comunicación rescisoria (art. 95, ley 24.467).
Referencia normativa. Arts. 92 bis y 231, LCT.

1) En período de prueba
La remuneración que debe utilizarse para este cálculo es la mensual que
debiera percibir el trabajador por los quince días del preaviso, regulados para
el caso que se omita éste en la extinción voluntaria del contrato de trabajo
durante el período de prueba (primeros tres meses del contrato a plazo
indeterminado).
El período del preaviso a liquidar, en caso de contrato de trabajo a plazo
indeterminado y a prueba durante los primeros tres meses, será de quince
días (art. 231, LCT). Como se trata de indemnización sustitutiva por preaviso
omitido y, en consecuencia, tiempo no trabajado, el plazo de preaviso
(omitido) no integra la antigüedad del trabajador. Por definición tiene carácter
indemnizatorio y, por lo tanto, no genera ni aportes ni contribuciones.
Referencia jurisprudencial. “Corresponde calcular la indemnización sustitutiva
del preaviso con más la incidencia del sueldo anual complementario, pues
conforme lo dispone el art. 232 de la ley 20.744 la suma que la trabajadora
hubiera devengado durante el lapso omitido incluye dicho rubro, cuyo
devengamiento tiene lugar con cada prestación laboral y es necesario para
alcanzar la equivalencia ordenada por la norma” (CNTrab, Sala VI, 30/4/01,
“W., A. M. c/La Delicia Felipe Fort SA”, LL, 2001-D-829; id., Sala II, "Lomsan,
Walter F. c/Citibank N. A.”; id., 28/2/96, DJ, 1996-2-608; id., Sala X,
27/10/97, "Márquez Aranguri, Julio C. c/Centro Gallego de Buenos Aires”, DJ,
1999-3-150).
Remuneración mensual devengada: $ 3.057,12 (sin SAC). Fecha de
contratación a prueba: 1 de noviembre de 2015. Fecha de egreso: 18 de enero
de 2016.
Preaviso a liquidar: 15 días.
Cálculo: $ 3.057,12 / 31 x 15 = $ 1.479,25.
Cálculo de la incidencia del SAC: $ 1.479,25/12 = $ 123,71
Donde:
“$ 3.057,12” es la remuneración mensual base de cálculo.
“31” es la cantidad de días correspondientes al mes del despido y por los
cuales hubiera correspondido la remuneración utilizada como base de cálculo.
“15” es la cantidad de días del preaviso omitido.

2) Finalizado el período de prueba


También aquí, la remuneración que debe utilizarse para este cálculo es la
mensual que debiera percibir el trabajador en el período del preaviso omitido.
No necesariamente será la mejor. En caso de remuneraciones homogéneas se
ha resuelto: "A los fines de calcular la indemnización sustitutiva de preaviso e
integración del mes de despido corresponde tomar como base de cálculo la
remuneración correspondiente al último mes trabajado, en lugar de la mejor
remuneración mensual, normal y habitual, pues ello resulta más acorde con la
directiva del art. 232 de la LCT en cuanto establece que quien fue privado del
aviso previo debe recibir un monto similar al que correspondería si hubiese
continuado trabajando” (CTrab Córdoba, Sala III, 23/9/10, “Rearte, Rodrigo M.
c/ServiMax SRL”, LLonline, AR/JUR/59175/2010). A falta de regulación
específica, en caso de remuneraciones variables, donde no pudiera
determinarse un valor exacto, predomina el criterio llamado "normalidad
próxima”, que estaría dado por la aplicación de un promedio de los últimos
meses, habiéndose resuelto en tal sentido que "a los fines de efectuar el
cálculo del rubro 'indemnización sustitutiva de preaviso’ e integración, a favor
de quien se desempeñaba como supervisor para un compañía de seguros,
debe aplicarse el criterio de ‘la normalidad próxima’ de modo que debe
determinarse la remuneración en base a un promedio de los últimos seis
meses” (CNTrab, Sala V, 30/9/09, “Lence, Carlos G. J. c/HSBC New York Life
Seguros de Vida Argentina SA”, LL, 2009-F-571; id., Sala I, 27/4/10, “Bevilac-
qua, Norberto L. c/Bausch & Lomb Argentina SRL”, DT, 2010- 2359).
Los plazos del preaviso y, consecuentemente, la indemnización sustitutiva que
aquí estamos calculando, varían según la antigüedad del trabajador al
momento de la extinción del contrato, conforme a lo dispuesto por el art. 231
de la LCT. La antigüedad debe calcularse como lo determina el art. 18 de dicha
ley, es decir, desde el comienzo de la vinculación aun si ella se hubiera
producido mediante contratos a plazo o eventuales confirmados más tarde con
su continuación por contrato a plazo indeterminado para el mismo empleador.
El período del preaviso a liquidar dependerá, como se dijo, de la antigüedad
del trabajador al momento de la extinción. Durante la vigencia del contrato de
trabajo a prueba (primeros tres meses, art. 92 bis, LCT) será de quince días,
finalizado éste y hasta los cinco años será de un mes y de dos meses con más
de cinco años (art. 231, LCT).
Ver referencia jurisprudencial inserta en el cálculo de la incidencia del SAC
para el ejemplo anterior.
Remuneración mensual devengada: $ 3.057,12.
Fecha de egreso: 18 de enero de 2016.
Antigüedad al momento del despido: 6 años y fracción. Preaviso a liquidar: 2
meses.
Cálculo: $ 3.057,12 x 2 = $ 6.114,24 Cálculo de la incidencia del SAC: $
6.114,24/12 = $ 509,52.
Donde:
“$ 3.057,12” es la remuneración mensual base de cálculo.
“2” son los meses que, en el caso, hubieran correspondido como preaviso por
la antigüedad del trabajador al momento del despido (mayor a cinco años, art.
231, LCT).
En caso de antigüedad de hasta cinco años al momento del despido, la fórmula
de liquidación sería la siguiente: $ 3.057,12 x 2 = $ 6.124,24.

c) La indemnización común por despido o por antigüedad


Referencia normativa. Art. 245, LCT. El texto de este artículo implementa la
modalidad de cálculo de la llamada "indemnización común por despido”. A su
vez, el importe del resarcimiento así alcanzado constituye también base de
cálculo de otras indemnizaciones especiales (tal el caso de la indemnización
del art. 247 de la misma ley), o de recargos de indemnización, previstos para
supuestos específicos (art. 15, ley 24.013, y arts. 1° y 2o, ley 25.323). La
norma y, consecuentemente, la modalidad del cálculo indemnizatorio fue
reformada en más de una oportunidad desde 1974 a la fecha. Las
modificaciones más trascendentes pasaron por eliminar su tope máximo (art.
48, ley 23.697), luego por reimplantarlo, pero con otra metodología (art. 153,
ley 24.013) y, finalmente, por sustituir la expresión "percibida" por
"devengada” en la remuneración mensual que debe tomarse como base de
cálculo (art. 5°, ley 25.877). En su versión simplificada, esta indemnización es
equivalente a un mes de remuneración, por cada año de antigüedad del
trabajador o fracción mayor a tres meses, siempre al momento del despido. La
antigüedad debe computarse conforme a lo dispuesto por el art. 18 de la LCT,
siendo aplicable lo explicado en oportunidad de tratar la indemnización
sustitutiva del preaviso.
La remuneración que debe utilizarse como base de cálculo es la mejor,
mensual, normal y habitual devengada durante el último año. El componente
sustancial de la definición es la "remuneración”, a ella se aplican los demás
adjetivos. Resultan de aplicación los comentarios realizados al ocuparnos de la
"remuneración” al comienzo de este capítulo y a ellos nos remitimos. De ese
modo, la base de cálculo estará dada por aquello que percibe el trabajador
como consecuencia de su contrato de trabajo, que es la “sustancia” utilizada
como parámetro básico para describir el importe indemnizatorio. Dicha
"sustancia” queda condicionada por cinco adjetivos o componentes que
determinan con mayor precisión la base de cálculo y que están dados por
"mejor”, "mensual", “normal y habitual”, "devengada" y “durante el último
año”. Aun cuando la ley no aclara a favor de quién es “la mejor”, se entiende
(art. 9°, LCT) que es la mejor para el trabajador y, por lo tanto, la de mayor
cuantía (la mejor para el empleador, sería la de menor importe). Los cuatro
adjetivos sólo pueden ser interpretados en su conjunto. Las condiciones de
normalidad y habitualidad obligan a incluir toda remuneración que integrara
las condiciones propias del contrato de trabajo que se extingue, aun cuando se
trate de importes variables (como ser comisiones, horas extras, salario
vacacional). En tales casos, la base de cálculo no podrá constituir un
promedio, sino la mejor remuneración. Debe tratarse de una remuneración
mensual, pero mensualmente devengada.
La expresión "devengar”, tal como la encontramos en el Diccionario de la Real
Academia Española, que encuentra sus raíces etimológicas en institutos
propios del derecho romano primitivo {vindicare, atribuirse, apropiarse), sólo
tiene acepción posible en el contexto de la retribución por servicios personales.
Significa adquirir derecho a alguna percepción o retribución por razón de
trabajo, servicio u otro título. La adquisición del derecho debe haber tenido
lugar, dentro del primer espacio temporal fijado por la norma, que es de un
mes, el cual se debe dar dentro del segundo espacio temporal, que es del
último año (o el tiempo de prestación de servicios cuando fuera menor).
Remuneración mensual "devengada" significa, en primer lugar, que se trate de
la remuneración debida, cuando ésta ha sido distinta de la efectivamente
cumplida por el empleador. En segundo lugar, que no debe estarse al monto
de la remuneración pagada, cuando lo fue en un valor inferior al adeudado,
sino a este último. La base de cálculo aquí descripta queda, a su vez,
condicionada en su cálculo a los cuatro supuestos que ejemplificaremos a
continuación.
1) Liquidación dentro de los topes (mínimo y máximo) previstos por el art. 245
de la LCT
En este caso, la base de cálculo no excede el equivalente de tres veces el
importe mensual de la suma que resulta del promedio de todas las
remuneraciones previstas en el CCT aplicable al trabajador al momento del
despido. Aun cuando se trate de trabajadores "no convencionados” o excluidos
del convenio, la norma siempre los considera comprendidos en alguno, ya que
resultará aplicable aquél vigente para el “establecimiento donde preste
servicios” y, en caso de existir más de uno, el "convenio más favorable” para
el trabajador (art. 9°, LCT).
Antigüedad a la fecha del despido: 5 años y 6 meses. Mejor remuneración
mensual, normal y habitual devengada durante el último año (con inclusión de
rubros "no remuneratorios”) (sin incidencia del SAC): $ 3.057,12.
Salario promedio del CCT aplicable: $ 2.254.
Tope máximo del art. 245 LCT: $ 6.762 (resultante de: $ 2.254 x 3).
Tope mínimo del art. 245, LCT: $ 3.057,12.
Cálculo de la indemnización: $ 3.057,12 x 6 = $ 18.342,72.
Donde:
“$ 3.057,12” es la remuneración base de cálculo, utilizada en el ejemplo.
Resulta aquí de aplicación, por ser inferior al tope máximo, que está dado por
la cantidad de $ 6.762 y superior al tope mínimo, previsto en la parte final del
art. 245 de la LCT que es de un mes de remuneración (sin tope máximo).
“6” es el multiplicador resultante de la antigüedad del trabajador al momento
del despido, que en nuestro ejemplo es de cinco años y fracción mayor a tres
meses, computable como un año más, esto es, seis períodos. Si la fracción
fuese igual o menor a tres meses, no sería considerada como un período más
y, en tal caso, el multiplicador sería de "5”.
“$ 18.342,72” es la indemnización común por despido del art. 245 de la LCT
resultante y aplicable al caso, dado que se prevé dentro de los topes máximo y
mínimo de la norma.
El período de preaviso cumplido, es decir, trabajado, es computable como
antigüedad para el cálculo de la indemnización. No es computable en caso de
haber sido omitido. Así fue resuelto: “Cuando el dependiente ha sido
despedido intempestivamente no corresponde sumar a la antigüedad el plazo
del preaviso omitido” (CNTrab, en pleno, 1/10/70, “Quevedo, Clara L.
c/Consorcio de Propietarios Gascón 899”, LL, 140-316, y DT, 1970-786).
Referencia jurisprudencial. “Para el cálculo de la indemnización por despido no
deben ser promediadas las remuneraciones variables, mensuales, normales y
habituales (art. 245, LCT)” (CN Trab, en pleno, n° 298, “Brandi, Roberto A.
c/Lotería Nacional SE s/despido”, TSS, 2000-998). “Descartada la
configuración de un supuesto fraude a la ley laboral, la bonificación abonada
por el empleador sin periodicidad mensual y con base en un sistema de
evaluación de desempeño del trabajador, no debe computarse a los efectos de
determinar la base salarial prevista en el primer párrafo del art. 245 de la ley
de contrato de trabajo” (CNTrab, en pleno, n° 322, “Tulosai, Alberto P.
c/Banco Central de la República Argentina”, DT, 2010-51).
En su “fisiología”, constituye "remuneración” “la contraprestación que debe
percibir el trabajador como consecuencia del contrato de trabajo” (art. 103,
LCT), y agrega el art. 1° del convenio OIT 95 que "salario” es "la remuneración
o ganancia, sea cual fuere su denominación o método de cálculo, siempre que
pueda evaluarse en efectivo, fijada por acuerdo o por la legislación nacional, y
debida por un empleador a un trabajador en virtud de un contrato de trabajo,
escrito o verbal, por el trabajo que este último haya efectuado o deba efectuar
o por servicios que haya prestado o deba prestar”.
En los últimos años se ha hecho corriente la "patología” de implementar
“beneficios estimables en dinero” a percibir por el trabajador, pero con la
calificación de “no remuneratorios" por el acto normativo que obliga o permite
su pago al empleador. Con argumentos que consideramos comunes para todas
estas formas patológicas de remunerar al trabajador, en oportunidad de
resolver la inconstitucionalidad del carácter "no remuneratorio” de los "tiques
canasta” tal como estaban regulados por el art. 103 bis de la LCT (versión ley
24.700, reformada por la ley 26.341), la Corte Suprema interpretó: “La
naturaleza jurídica de una institución debe ser definida, fundamentalmente,
por los elementos que la constituyen, con independencia del nombre que el
legislador, o los particulares, le atribuyan (doctrina de ‘Inta Industria Textil
Argentina S.A. s/apelación’, Fallos, 303:1812 y su cita), sobre todo cuando
cualquier limitación constitucional que se pretendiese ignorar bajo el ropaje del
nomen juris sería inconstitucional (Fallos, 329:3680)..., la noción de
remuneración que ha sido enunciada en manera alguna podría entenderse de
alcances menores que la acuñada en el art. 11 del convenio OIT 95 sobre la
protección del salario, corresponde declarar la inconstitucionalidad del art. 103
bis, inc. c, de la ley de contrato de trabajo (texto según ley 24.700), relativo a
los vales alimentarios, en cuanto niega a éstos naturaleza salarial” (CSJN,
1/9/09, "Pérez, Aníbal R. c/Disco SA”, DT, 2009-1011).
Sin perjuicio de entender que dichos argumentos también resultan aplicables a
los incrementos salariales calificados como "no remuneratorios” por la
negociación colectiva, corresponde al respecto agregar que, en la “fisiología”
de nuestro sistema de fuentes de regulación del contrato de trabajo, no todo
acuerdo colectivo es aplicable sin más al contrato individual de trabajo, ya que
según el esquema de orden público laboral previsto por nuestra LCT, el CCT
sólo será aplicable en tanto y en cuanto “contengan normas más favorables a
los trabajadores” (art. 8o, LCT), aspecto enfatizado por el artículo siguiente.
Los aspectos menos favorables o desfavorables, como es el caso de
incrementos salariales que no serían computables como base de cálculo de
otros rubros (aguinaldos, vacaciones, licencias o suspensiones del contrato,
indemnizaciones por su extinción), no resultan de aplicación. De allí que en los
esquemas de liquidaciones propuestos nos ajustamos a la “fisiología” del
sistema e incluimos la incidencia de los "no remuneratorios” sobre la base de
cálculo.
En jurisdicción de la Cámara se sostuvo: “1. No corresponde incluir en la base
salarial prevista en el primer párrafo del art. 245 de la ley de contrato de
trabajo, la parte proporcional del sueldo anual complementario” (CNTrab, en
pleno, n° 322, 19/ 11/09, "Tulosai, Alberto R c/Banco Central de la República
Argentina”, DT, 2010-51).
El cálculo y publicación, tanto del promedio de remuneraciones como del tope
resultante, lo dispone el Ministerio de Trabajo, Empleo y Seguridad Social (art.
245, LCT). Habitualmente, la autoridad administrativa primero homologa el
acuerdo salarial y luego realiza el cálculo del promedio de remuneraciones y
del tope indemnizatorio. Ambos actos administrativos, adoptan la forma de
resolución (por lo general, de la Secretaría de Trabajo) y son publicados en el
"Boletín Oficial”. También están disponibles por Internet, en los buscadores del
sitio oficial del Ministerio de Trabajo (www.trabajo.gov.ar).

2) Liquidación ajustada al tope mínimo previsto por el art. 245, LCT


El tope mínimo está dado por la parte final del art. 245 de la LCT. La
indemnización “en ningún caso podrá ser inferior a un mes de sueldo calculado
sobre la base del sistema establecido en el primer párrafo”. Dado que en el
párr. 1o no están contemplados los lineamientos que afectan la base de cálculo
al tope máximo (de tres veces el promedio de remuneraciones del CCT
aplicable), el tope mínimo estará dado, entonces, por un mes de remuneración
con prescindencia del tope máximo.
Antigüedad a la fecha del despido: 1 año y 2 meses.
Mejor remuneración mensual, normal y habitual devengada durante el último
año: $ 7.500.
Salario promedio del CCT aplicable: $ 2.254.
Tope del art. 245 LCT: $ 6.762 (resultante de: $ 2.254 x 3). Cálculo de la
indemnización con el tope máximo: $ 6.762 x 1 = $ 6.762.
Cálculo de la indemnización con el tope mínimo:
$ 7.500 x 1 = $ 7.500.
Prevalece esta última liquidación.
Donde:
“$ 7.500” es la remuneración base de cálculo, utilizada en el ejemplo. Resulta
aquí de aplicación, según el tope mínimo fijado en la parte final del art. 245 de
la LCT. La indemnización, en ningún caso, puede ser inferior a un mes de la
base de cálculo, con prescindencia de los topes máximos.
“1” es el multiplicador resultante de la antigüedad del trabajador al momento
del despido, que en nuestro ejemplo es de un año y fracción menor a tres
meses, que por ser inferior a tres meses, no es considerada como un período
más.

3) Liquidación con tope máximo que no afecta más del 33% de la base de
cálculo
Referencia jurisprudencial. "La limitación a la base salarial de la indemnización
por despido sin justa causa -art. 245, párrs. 2o y 3H, LCT- sólo debe aplicarse
hasta el 33% de la mejor remuneración mensual normal y habitual
computable, pues lo contrario significaría consentir un instituto jurídico que
termina incumpliendo con el deber inexcusable del art. 14 bis de la Const.
Nacional, acerca de que el trabajo gozará de la protección de las leyes y que
éstas asegurarán al trabajador protección contra el despido arbitrario y
condiciones equitativas de labor, contrariando el principio de razonabilidad del
art. 28 de la ley suprema” (CSJN, 14/9/04, “Vizzoti, Carlos A. c/AMSA SA”, DT,
2004-1211, y TSS, 2004-766). La Corte Suprema no es un tribunal de
casación y sus sentencias sólo son obligatorias porque ella lo dice, no hay
norma que imponga tal carácter. Sin perjuicio de ello, sus decisiones cuentan
con amplio acatamiento por parte de los tribunales inferiores, tanto por
razones prácticas (si el caso vuelve a la Corte, lo más probable es que
mantenga el criterio anterior, salvo cambio de composición o de doctrina, lo
que suele ocurrir) como por la elevada jerarquía académica de sus integrantes
y las sentencias que dicta.
Por tal motivo, incluimos los dos criterios de liquidación que se siguen, aunque
dejando a salvo nuestra opinión personal, en el sentido que esta doctrina de la
Corte Suprema afecta la división de poderes, puesto que no le corresponde al
Judicial legislar. La correcta aplicación de nuestro sistema de control de
constitucionalidad, hubiera llevado a prescindir de la norma inconstitucional (y
el consecuente tope máximo del art. 245, LCT), mandando calcular la
indemnización sin tope alguno, dejando a salvo la función legislativa
(corrección mediante norma de alcance general) a cargo del Congreso de la
Nación. La doctrina “Vizzoti”, aplicada a otros institutos del art. 14 bis
constitucional, puede derivar en la convalidación de normas que, directa o
indirectamente (por disponibilidad colectiva, por ejemplo), afecten el 33% de
otros derechos del tipo indisponibles e irrenunciables, como ser salarios,
vacaciones, preaviso, prestaciones o indemnizaciones derivadas de
enfermedades o accidentes del trabajo, otras indemnizaciones, etcétera. Nos
hemos ocupado con mayor profundidad del tema, en nuestro trabajo, Ahora la
Corte legisla: peligrosa solución a medias para nuestro sistema legal de
protección contra el despido arbitrario, El Dial, edición electrónica del 5/10/04,
cita DC4B2).
Antigüedad a la fecha del despido: 4 años y 3 meses. Mejor remuneración
mensual, normal y habitual devengada durante el último año: $ 8.375.
Salario promedio del CCT aplicable: $ 2.254.
Tope máximo del art. 245, LCT: $ 6.762 (resultante de: $ 2.254 x 3).
Tope mínimo del art. 245, LCT: $ 8.375.
Cálculo de la indemnización con prescindencia del tope máximo: $ 8.375 x 4 =
$ 33.500.
Cálculo del límite de confiscatoriedad del tope máximo:
($ 8.375 x 0,67) x 4 = $ 22.445.
Cálculo de la indemnización con aplicación del tope máximo: $ 6.762 x 4 = $
27.048.
Indemnización aplicable (la calculada con aplicación del tope máximo): $
27.048.
Donde:
"$ 33.500” es el resultado del cálculo de la indemnización del art. 245 de la
LCT, sólo con los parámetros del párr. I o, esto es, con prescindencia del tope
máximo.
"$ 22.445” resulta de calcular la misma indemnización, reducida al máximo de
discrecionalidad regulatoria que admite la Corte Suprema en la doctrina del
caso "Vizzoti".
"$ 27.048” es el resultado que se obtiene al calcular la indemnización del art.
245 de la LCT, con aplicación del tope máximo allí previsto, que conforme la
doctrina "Vizzoti”, en tanto no afecta más del 33% de la base de cálculo
(mejor remuneración mensual devengada), no derivaría en una reducción
irrazonable a la modalidad escogida para proteger el despido arbitrario.
"$ 6.452” es el recorte del derecho de protección contra el despido arbitrario,
que en nuestro ejemplo el trabajador pierde por aplicación de la doctrina del
caso “Vizzoti” (resultante de $ 33.500 - $ 27.048 = $ 6.452). Según ella, la
pérdida debe ser soportada por el trabajador, puesto que es inferior al 33%
del cálculo indemnizatorio con prescindencia del tope. En nuestro ejemplo,
dicho importe representa casi un 20% del monto indemnizatorio calculado con
prescindencia del tope. Con la doctrina "Vizzoti”, la excepción reglamentaria
puede llegar a afectar hasta el 33% de dicho importe, que sería en el caso, por
$ 11.055.
El cálculo y publicación, tanto del promedio de remuneraciones como del tope
resultante, se encuentra a cargo del Ministerio de Trabajo, Empleo y Seguridad
Social (art. 245, LCT). Habitualmente, la autoridad administrativa primero
homologa el acuerdo salarial y luego realiza el cálculo del promedio de
remuneraciones y del tope indemnizatorio. Ambos actos administrativos
adoptan la forma de resolución (por lo general, de la Secretaría de Trabajo) y
son publicados en el Boletín Oficial. También se encuentran disponibles por
Internet, en los buscadores del sitio oficial del Ministerio de Trabajo
(www.trabajo.gov.ar).
4) Liquidación con tope máximo que afecta más del 33% de la base de
cálculo
Antigüedad a la fecha del despido: 8 años y 7 meses. Mejor remuneración
mensual, normal y habitual devengada durante el último año: $ 10.375.
Salario promedio del CCT aplicable: $ 2.254.
Tope máximo del art. 245, LCT: $ 6.762 (resultante de: $ 2.254 x 3).
Tope mínimo del art. 245, LCT: $ 10.375.
Cálculo de la indemnización con prescindencia del tope máximo: $ 10.375 x 9
= $ 96.615.
Cálculo del límite de confiscatoriedad del tope máximo:
($ 10.375 x 0,67) x 9 = $ 62.561,25.
Cálculo de la indemnización con aplicación del tope máximo: $ 6.762 x 9 = $
60.858. Indemnización aplicable (la calculada con aplicación del límite de
confiscatoriedad): $ 62.561,25.
Donde:
"$ 96.615” es el resultado del cálculo de la indemnización del art. 245 de la
LCT, sólo con los parámetros del párr. I o, esto es, con prescindencia del tope
máximo.
“$ 62.561,25” resulta de calcular la misma indemnización, reducida al máximo
de discrecionalidad regulatoria que admite la Corte Suprema en la doctrina del
caso "Vizzoti”. En el caso, es la indemnización a aplicar según dicha doctrina,
previa declaración de inconstitucionalidad del tope máximo del art. 245 de la
LCT.
“$ 60.858” es el resultado que se obtiene, calculando la indemnización del art.
245 de la LCT, con aplicación del tope máximo allí previsto, que conforme la
doctrina “Vizzoti”, en el ejemplo sería inconstitucional (si así se declara) por
afectar en más del 33% de la base de cálculo (mejor remuneración mensual
devengada).
“$ 34.053,75” es el recorte del derecho de protección contra el despido
arbitrario que, en nuestro ejemplo, el trabajador pierde por aplicación de la
doctrina del caso "Vizzoti” (resultante de: $ 96.615 - $ 62.561,25 = $
34.053,75).

5) Reintegro del trabajador (con deducción de indemnización anterior)


Referencia normativa. Art. 255, LCT.
Antigüedad anterior: 4 años y 6 meses.
Indemnización recibida por la antigüedad anterior: $ 15.285,60 (resultante de:
$ 3.057,12 x 5 = $ 15.285,60).
Antigüedad del período posterior, desde el reingreso y hasta el despido a
indemnizar: 3 años y 6 meses.
Antigüedad de ambos períodos: 8 años.
Mejor remuneración mensual, normal y habitual devengada durante el último
año del último tramo de la relación: $ 4.504.
Cálculo de la indemnización (computando ambos períodos):
($ 4.504 x 8) - $ 15.285,60 = $ 20.746,40
Donde:
"$ 4.504” es la remuneración base de cálculo, utilizada en el ejemplo, que es
la mejor remuneración mensual devengada por el trabajador en el último año
del último período de su relación laboral y, por lo tanto, computable como
base de cálculo para su segundo despido.
“8” es el multiplicador resultante de la antigüedad total del trabajador,
considerando ambos períodos de la relación y al momento del segundo
despido.
“$ 15.285,60” es la indemnización común por despido del art. 245 de la LCT
que percibió el trabajador del mismo empleador, en ocasión de producirse el
despido durante su primer tramo de vinculación. Según lo dispuesto por el art.
255 de la LCT, es la indemnización que debe deducirse del importe calculado
sumando ambos tramos de antigüedad. Igual criterio debe seguirse, si por el
primer tramo percibió las otras indemnizaciones vinculadas a dicha
antigüedad, previstas por el art. 255 de la LCT (de los arts. 246, 247, 250,
251, 253 y 254 de la misma ley). Ello así, en tanto y en cuanto no se vea
disminuida la indemnización calculada en función de la que hubiera
correspondido al trabajador si su período de servicios hubiera sido sólo el
último, caso al que corresponde la liquidación del ejemplo que se sigue.
En nuestro ejemplo prescindimos de las referencias a los topes máximos o
mínimos que resultan de la aplicación de lo explicado en las liquidaciones
anteriores, a las que nos remitimos.

6) Reingreso del trabajador (sin deducción de indemnización anterior)


Referencia normativa. Art. 255, LCT. En esta liquidación, como el cómputo del
importe indemnizatorio percibido por la extinción anterior deriva en la
reducción de la indemnización calculada sólo en función del último tramo,
prevalece ésta sobre la anterior, tal como lo determina dicha norma.
En nuestro ejemplo, prescindimos de las referencias a los topes máximos o
mínimos que resultan de lo explicado en las liquidaciones anteriores, a las que
nos remitimos.
Antigüedad anterior: 4 años y 6 meses.
Indemnización recibida por la antigüedad anterior: $ 15.285,60 (resultante de:
$ 3.057,12 x 5 = $ 15.285,60).
Antigüedad del período posterior, desde el reingreso y hasta el despido a
indemnizar: 3 años y 6 meses.
Antigüedad de ambos períodos: 8 años.
Mejor remuneración mensual, normal y habitual devengada durante el último
año del último tramo de la relación: $ 3.204.
Cálculo de la indemnización (con descuento de la anterior): ($ 4.504 x 8) - $
15.285,60 = $ 10.346,40.
Cálculo de la indemnización (computando solo la antigüedad posterior al
reingreso): $ 3.204 x 4 = $ 12.816. Prevalece el último cálculo, computando la
antigüedad adquirida sólo desde el reingreso y por $ 12.816.
7) Reingreso del trabajador jubilado
Referencia normativa. Art. 253, LCT. La norma exige que el trabajador haya
cesado en su relación y luego reingresado para el mismo empleador. Es el
supuesto que contemplamos en el ejemplo de liquidación. No hay norma
expresa que hubiera previsto ampliar la previsión normativa del art. 253 de la
LCT a los casos donde, no habiendo ni cese ni reingreso, el trabajador jubilado
continúe trabajando para un mismo empleador. Para estos casos, en
jurisdicción de la Ciudad de Buenos Aires, se ha aprobado la siguiente doctrina
legal: “Es aplicable lo dispuesto por el art. 253, último párrafo, LCT al caso de
un trabajador que sigue prestando servicios sin interrupción a las órdenes del
mismo empleador, luego del goce del beneficio de la jubilación” (CNTrab, en
pleno, n° 321, 5/6/09, “Couto de Capa, Irene M. c/Aryva SA s/ley 14.546”,
DT, 2009-773, y LL, 2009-E-132). No compartimos ni el criterio, ni la
obligatoriedad de esta doctrina aprobada con un número inferior de jueces al
exigido por la ley. Por ella se desarticula por completo el art. 252 de la LCT.
Aquello que el legislador ha previsto como una facultad para el empleador
(disolver el contrato del trabajador jubilado sin indemnización), con la
cuestionada doctrina pasa a convertirse en algo que ocurre ipso facto y sin
norma expresa que así lo hubiera previsto. Privar al trabajador despedido de
su indemnización por antigüedad en los casos donde la ley no lo exige en
forma expresa, además de importar una caducidad de derechos no admitida
por la misma LCT que se pretende interpretar (art. 259), pulveriza el derecho
constitucional de protección contra el despido arbitrario y genera una
oportunidad de discriminación y enriquecimiento indebido para el empleador.
La jurisprudencia tradicional de la prov. de Buenos Aires, con mejor criterio no
acompaña la doctrina así aprobada por la Cámara de Apelaciones del Trabajo.
Antigüedad anterior al cese por jubilación: 35 años y 6 meses.
No hubo indemnización por antigüedad en el período anterior, dada la
utilización por parte del empleador, de la facultad otorgada por el art. 252 de
la LCT y la extinción del contrato, sin derecho a indemnización, con la
jubilación del trabajador, seguida del cese efectivo.
Antigüedad del período posterior, desde el reingreso y hasta el despido a
indemnizar: 3 años y 6 meses.
Mejor remuneración mensual, normal y habitual devengada durante el último
año del último tramo de la relación: $ 3.204.
Cálculo de la indemnización (computando sólo la antigüedad posterior al
reingreso): $ 3.204 x 4 = $ 12.816.
En nuestro ejemplo prescindimos de las referencias a los topes máximos o
mínimos que resultan de lo explicado en las liquidaciones anteriores, a las que
nos remitimos.
La antigüedad anterior no es computable, dado que conforme la previsión legal
(art. 253, LCT), se han dado los tres elementos fácticos que la excluyen:
jubilación del trabajador, cese efectivo y posterior reingreso a las órdenes del
mismo empleador.

Las indemnizaciones reducidas o excepcionales DEL DESPIDO


Las denominamos de este modo, porque, con carácter excepcional, se derivan
en una reducción a la indemnización común por despido del art. 245 de la LCT.
Dado ese carácter, el trabajador se encuentra habilitado para cuestionar la
invocación de las condiciones fácticas invocadas por el empleador en la
comunicación rescisoria y, en el proceso judicial posterior, estará a cargo de
éste la prueba de los extremos de hecho que habilitan el pago de la
indemnización reducida.

a) Por falta o disminución de trabajo o fuerza mayor


Referencia normativa. Art. 247, LCT. En nuestro ejemplo prescindimos de las
referencias a los topes máximos o mínimos, que resulta de lo explicado en las
liquidaciones anteriores, a las que nos remitimos.
Antigüedad del trabajador al momento del despido: 6 años y 6 meses.
Mejor remuneración mensual, normal y habitual devengada durante el último
año del último tramo de la relación: $ 3.204.
Cálculo de la indemnización del art. 247 de la LCT:
($ 3.204 x 7) / 2 = $ 11.214.
Donde:
“($ 3.204 x 7)” es el cálculo de la indemnización común por despido, en el
ejemplo utilizado. Resulta de multiplicar la remuneración devengada base de
cálculo, por el número “7” que resulta de la antigüedad en el empleo de seis
años y fracción mayor a tres meses.
“/ 2” es el divisor utilizado para obtener un resultado equivalente a la mitad de
la indemnización común por despido, tal como lo exige el art. 247 de la LCT.

b) Supuestos asimilados
Estos supuestos están dados por las indemnizaciones reducidas y de
excepción, contempladas en la LCT y a las cuales resulta aplicable la fórmula
del art. 247 que hemos ejemplificado en el punto anterior y a la que nos
remitimos.
Dichos supuestos surgen de los siguientes.
1) Extinción del contrato de trabajo por incapacidad física del trabajador e
imposibilidad del empleador a asignarle tareas acordes (arts. 212 y 254, LCT).
2) Extinción por muerte del empleador, cuando sus condiciones personales
hubieran sido la causa determinante de la relación laboral y sin las cuales ésta
no podría proseguir (art. 249, LCT).
3) Extinción del contrato a plazo determinado, por vencimiento del plazo,
mediando preaviso, estando el contrato íntegramente cumplido y con duración
no inferior a un año (arts. 95 y 250, LCT).
4) Extinción cuando el despido hubiera sido motivado por quiebra del
empleador y ésta declarada como debida a causas que no le son imputables
(art. 251, LCT).
5) Extinción por pérdida de la habilitación especial requerida para la
realización de los servicios que hicieron al objeto de la contratación del
trabajador, cuando la inhabilitación provenga de dolo o culpa grave
inexcusable de su parte (art. 254, LCT).

Las indemnizaciones especiales


a) Por irregularidades de registro en la ley nacional de empleo (LNE)
Los rubros previstos por los arts. 8 a 10 y 15 de la LNE, que se liquidan en
este acápite, constituyen “indemnizaciones especiales”, dado que ellas tienen
por causa el resarcimiento a los daños adicionales (no comprendidos en las
indemnizaciones tarifa- das comunes), provocados al trabajador, sea con
motivo de los incumplimientos de registro imputados al empleador (arts. 8o a
10), o por el despido adoptado en represalia o como consecuencia de los
reclamos de aquél (art. 15). El propio legislador le ha asignado a estos rubros
carácter indemnizatorio y lo ha hecho de un modo expreso, al disponer casi
por igual, en los arts. 8o a 10 que, dado los presupuestos de hecho allí
previstos, el empleador "abonará al trabajador afectado” (obviamente por la
falta o irregularidad de registro) “una indemnización”. A su vez, el art. 15 de la
misma ley confiere al trabajador despedido (y el despido sin justa causa es un
hecho ilícito, que provoca un daño, del que se sigue la obligación de
"indemnizar”) el derecho a percibir “el doble de las indemnizaciones”. La
sustancia que se duplica aquí, es claramente indemnizatoria y la duplicación,
un mero accidente, determinado por el despido directo (despido represalia) o
indirecto (sólo cuando se vincula con las irregularidades de registro, de
acuerdo al párr. 2o del art. 15), que ha sido previsto por el legislador,
expresando en dicha norma, su voluntad a que sea reparado en modo
adicional, el mayor daño sufrido por el trabajador en dicha contingencia.
En modo alguno puede justificarse que se trate de "multas”, como
erróneamente califica alguna doctrina, la cual fue de ese modo recibida por
numerosas sentencias que con ligereza e inusitada liviandad intelectual
califican como "multas” a estas indemnizaciones. Las únicas "multas” del
derecho del trabajo son las correspondientes a su régimen de infracciones (ley
25.212), no hay otras en el terreno del contrato individual de trabajo, an
cuando lo más parecido sea la “sanción” del art. 132 bis de la LCT. No se trata
de un error circunstancial, como muchos otros que han alterado nuestra
lengua castellana, de Cervantes a la fecha, sino de un error ideológicamente
engañoso, una falacia que tiende a desarticular los mecanismos de reparación
de daños por irregularidades de registro, pretendiendo hacer creer al juzgador
que con la aplicación de "multas” estaría “castigando” al empleador cuando, en
verdad, se limita a la reparación de un daño adicional sufrido por el trabajador.
1) Omisión total de registro
Fecha de ingreso: 6/9/06.
Fecha de egreso: 13/9/16.
Mejor remuneración mensual devengada, en agosto de 2010: $ 8.000 (sin
SAC) o bien $ 8.666,67 (con incidencia de SAC).
Cálculo de la indemnización:
Liquidación del rubro indemnización (art. 8o, LNE)
Período no registrado: 6/9/06 al 13/9/16, 120 meses y 7 días Remuneraciones
no registradas al mes c/SAC: $ 8.666,67
1. Remuneraciones c/SAC devengadas
por 120 meses ($ 8.666,67 x 120) $ 1.040.000,40
2. Remuneraciones c/SAC devengadas
por 7 días ($ 8.666,67 / 30,4 x 7) $ 1.995,61
Total remuneraciones devengadas
no registradas $ 1.041.996,01
Indemnización art. 8o, LNE (total / 4) $ 260.499,00
3. Tope mínimo del art. 8°, LNE (3 meses) $ 26.000,01
Donde:
“30,4” es el divisor utilizado, sobre la remuneración mensual, para calcular la
remuneración por día (30,4 son los días de un mes promedio que surge de:
365 / 12 = 30,4. No hay norma que específicamente lo haya previsto para el
cálculo de la remuneración diaria. Sin embargo, fue el criterio empleado por el
legislador en el art. 12 de la LRT para determinar el procedimiento de cálculo
del ingreso base).
Referencia normativa. El art. 8o de la LNE prevé que en los casos de
irregularidad total de registro, el trabajador afectado tendrá derecho a “una
indemnización equivalente a la cuarta parte del importe de las remuneraciones
devengadas desde el comienzo de la vinculación, computadas a valores
reajustados de acuerdo a la normativa vigente”. Ello así, en tanto y en cuanto
se hubieran dado las cuatro condiciones exigidas por el art. 11 de la misma ley
(texto ley 25.345, art. 47) y su reglamentación (decr. 2725/91, art. 3o), a
saber: a) intimación cursada al empleador, por el trabajador o su asociación
sindical, para que en el plazo de treinta días corridos proceda a subsanar las
irregularidades de registro de que se trate, consignando "la real fecha de
ingreso y las circunstancias verídicas que permitan calificar a la inscripción
como defectuosa”; b) remitir de inmediato o dentro de las veinticuatro horas
hábiles siguientes, copia a la AFIP de dicho requerimiento; c) que todo ello
ocurra estando vigente la relación laboral, y d) que el empleador no diera total
cumplimiento a la intimación dentro del plazo indicado. Estas cuatro
condiciones son comunes para la procedencia de las indemnizaciones
especiales previstas por los arts. 8o a 10 de la LNE. Habrá “omisión total de
registro” cuando el empleador no hubiera dado cumplimiento a los registros
que indica el art. 7o de la LNE (Libro Especial y SURL). Esta obligación está a
cargo del empleador directo y rige aun en caso de hallarse la relación
registrada con empleador ficticio, intermediario u homme de paille. En el
ámbito de la Ciudad de Buenos Aires, sobre el particular resulta aplicable la
doctrina aprobada por plenario n° 323: “Cuando de acuerdo con el primer
párrafo del art. 29 de la LCT se establece que el trabajador ha sido empleado
directo de la empresa usuaria de sus servicios, procede la indemnización
prevista en el art. 8o de la ley 24.013 aunque el contrato de trabajo haya sido
inscripto solamente por la empresa intermediaria” (CNTrab, en pleno, 30/6/10,
"Vásquez, María L. c/Telefónica de Argentina SA y otro s/despido”, LL, 2010-D-
392, y DT, 2010-2079).
O, en caso de continuar la relación (dado que no es requisito extinguirla
para que proceda esta indemnización, art. 14, LNE), la fecha de vencimiento
del plazo de treinta días otorgado conforme la intimación del art. 11 de la LNE.
Los arts. 8° a 10 de la LNE indican que la base de cálculo estará dada por "las
remuneraciones devengadas, computadas a valores reajustados de acuerdo a
la normativa vigente”. Los valores "reajustados”, a que alude la norma, no se
refieren a algún tipo de reajuste por depreciación monetaria, tal como se
hallaba previsto por el art. 276 de la LCT (virtualmente derogado por la ley
23.928), sino que más bien, en los casos de relaciones clandestinas
prolongadas, con variaciones en el salario, no debiera recurrirse a los valores
históricos, sino a los actuales, derivados de los incrementos salariales
acordados en forma colectiva o individual.
Dado que la indemnización del art. 8° de la LNE debe calcularse sobre la base
de las remuneraciones devengadas y dicho concepto se ve integrado por el
SAC, que se devenga mes a mes (aun cuando su percepción sea diferida).
Referencia jurisprudencial. “El sueldo anual complementario debe ser incluido
en el concepto remuneratorio adoptado para el cálculo de la indemnización
prevista por el art. 8o de la ley 24.013, sobre los casos de empleo no
registrado” (CNTrab, Sala IV, 31/5/00, "Olivera, Ignacio C. c/PAMI”, LL, 2000-
E-894).
Según lo dispuesto por el último párrafo del art. 8", LNE, el importe de esta
indemnización no puede ser inferior a tres veces el salario que resulte de la
aplicación del art. 245 de la LCT. En el ejemplo se ha aplicado el salario con
más la incidencia del SAC. En jurisdicción de la Ciudad Autónoma de Buenos
Aires (por aplicación de la doctrina aprobada por la CNTrab, plenario 322,
“Tulosai”), el tope mínimo debería calcularse sin la incidencia del SAC.

2) Falsa fecha de ingreso


Fecha real de ingreso: 5/4/99.
Fecha falsamente registrada: 16/5/01.
Diferencia entre ambas fechas: 25 meses y 12 días.
Remuneración devengada, reajustada a valores del momento de la intimación:
$ 4.976 (sin incidencia de SAC) o bien $ 5.390,67 (con incidencia de SAC con
+ $ 414,67 resultantes de $ 4.976 / 12).
Cálculo de la indemnización del art. 9o de la LNE
Liquidación del rubro indemnización (art. 9o, LNE)
Período no registrado: 5/4/99 al 16/5/01,
25 meses y 12 días
1. Remuneración mensual reajustada c/SAC
2. Remuneraciones c/SAC devengadas por 25 meses ($ 5.390,67 x 25)
3. Remuneraciones c/SAC devengadas por 12 días ($ 5.390,67/30,4 x 12)
Total remuneraciones devengadas no registradas
Indemnización art. 9oley 24.013 (total / 4)
$ 5.390,67
$ 134.766,75 $ 2.127,90
$ 136.894,65 $ 34.223,66
Referencia normativa. El art. 9° de la LNE prevé que en los casos donde el
empleador consignare en la documentación laboral una falsa fecha de ingreso,
el trabajador afectado tendrá derecho a "una indemnización equivalente a la
cuarta parte del importe de las remuneraciones devengadas desde la fecha de
ingreso hasta la fecha falsamente consignada, computadas a valores
reajustados de acuerdo a la normativa vigente”, ello así, en tanto y en cuanto
se hubieran dado las cuatro condiciones que hemos expuesto al ocuparnos de
la indemnización del art. 8o de la misma ley.

3) Falso monto de remuneraciones


Fecha de ingreso: 16/5/01.
Fecha de egreso: 17/8/10.
Períodos comprendidos en la irregularidad: 111 meses y 2 días.
Complemento remuneratorio pagado sin registrar: $ 600 mensuales + SAC por
$ 50 (que surgen de: $ 600/ 12).
Cálculo de la indemnización:
Liquidación del rubro indemnización (art. 10, LNE)
Período no registrado: 16/5/01 al 17/8/10, 111 meses y 2 días
1. Remuneración mensual reajustada c/SAC: $ 650,00
2. Remuneraciones c/SAC devengadas
por 111 meses ($ 650 x 111) $72.150,00
3. Remuneraciones c/SAC devengadas
por 2 días ($ 650/30,4 x 2) $ 42,76
Total remuneraciones devengadas
no registradas $ 72.192,76
Indemnización art. 10, LNE (total 14) $ 18.048,19
Referencia normativa. El art. 10 de la ley 24.013 prevé que en los casos donde
el empleador consignare en la documentación laboral una falsa remuneración
(por lo general, con menor monto registrado al real pagado), el trabajador
afectado tendrá derecho a “una indemnización equivalente a la cuarta parte
del importe de las remuneraciones devengadas y no registradas, debidamente
reajustadas desde la fecha en que comenzó a consignarse indebidamente el
monto de la remuneración”; ello es así, en tanto y en cuanto se hubieran dado
las cuatro condiciones que hemos expuesto al ocuparnos de la indemnización
del art. 8o de la misma ley. Esta indemnización requiere que exista el registro
de la relación laboral. Es compatible con la indemnización especial del art. 9°
de la LNE, en los casos donde, con posterioridad al registro de la falsa fecha de
ingreso, se adiciona el incumplimiento específico de registrar un importe
remuneratorio inferior al pagado. En este caso, la indemnización especial que
tratamos deberá calcularse a partir del registro de la relación laboral y no por
los períodos anteriores, que estarían cubiertos por la indemnización especial
del art. 9o de la LNE. O, en su caso, la fecha de inicio del incumplimiento de
falso monto registrado.
En caso de continuar la relación (dado que no es requisito extinguirla para que
proceda esta indemnización, art. 14, LNE), la fecha de vencimiento del plazo
de treinta días otorgado conforme la intimación del art. 11 de la ley citada.
El monto de $ 50 es el importe pagado por encima del neto registrado en
recibos de remuneración.
Referencia jurisprudencial. “Si la demandada no registró adecuadamente las
remuneraciones percibidas por el trabajador y la irregularidad se extendió a la
totalidad de las remuneraciones correspondientes al año entero de trabajo y,
por tanto, a los aguinaldos entregados, corresponde incluir la proporción del
sueldo anual complementario en la indemnización del art. 10 de la ley 24.013”
(CNTrab, Sala IV, 23/4/07, “Ruggiero Rodríguez, Adriana C. c/Excelsitas SA y
otros”, LLonline, AR/JUR/1705/2007).

4) Duplicación indemnizatoria
Antigüedad al despido: 6 años.
Fecha de egreso: 14/4/10.
Remuneración mensual devengada sin SAC: $ 3.057,72. Remuneración
mensual devengada c/SAC: $ 3.312,53.
Cálculo de la indemnización:
Liquidación de duplicación indemnizatoria (art. 15, LNE)
1. Integración mes despido c/SAC ($ 3.312,53 /30 x 16)
2. Ind. sustit. del preaviso c/SAC
3. Ind. por antigüedad s/SAC ($ 3.057,72 x 6)
4. Ind. art. 15, LNE (rubros 1 + 2+3)
$ 1.766,68 $ 3.312,53 $ 18.346,32 $ 23.425,53
Referencia normativa. Art. 15, LNE. Dos son los requisitos previstos por la
norma citada, para la procedencia de esta indemnización y que varían según
se trate de despido directo o indirecto, a saber la intimación del art. 11 de la
LNE; el despido directo e incausado, dentro de los dos años de haberse
cursado dicha intimación, despido indirecto, cuando la justa "causa invocada
no tuviera vinculación con las previstas en los arts. 8o, 9° y 10 de la LNE”. La
indemnización está dada por la consecuencia prevista por el legislador para
este supuesto y consiste en "el doble de las indemnizaciones que le hubieren
correspondido (al trabajador afectado) como consecuencia del despido”. En el
régimen general, estas indemnizaciones están dadas por la integración del
mes del despido (art. 233), la indemnización sustitutiva del preaviso (art. 232)
y la indemnización común por despido (art. 245). Con relación a la primera, se
ha resuelto: “La duplicación a que alude el art. 15 de la ley 24.013, incluye la
suma prevista en el art. 233, párr. 2o, de la LCT” (CNTrab, en pleno, n° 302,
19/10/01, "Palloni, Mariela H. c/Depormed SA s/despido”, TSS, 2001-1063).
Para la procedencia de esta indemnización, no es requisito el envío de copia a
la AFIP de la intimación del art. 11 de la LNE, conforme a la siguiente
referencia jurisprudencial. ‘‘La exigencia establecida en el art. 11 de la LNE
-según la modificación introducida por la ley 25.345-, consistente en notificar
fehacientemente a la AFIP respecto de la intimación de regularización
formulada por el trabajador al empleador, sólo alcanza a las multas de los
arts. 8o, 9o y 10 de la citada ley, por lo cual su falta de realización en modo
alguno obsta la duplicación a la cual alude el art. 15, siempre y cuando se
hubiere cursado la intimación dirigida al empleador, de manera plenamente
justificada (del dictamen del Procurador Fiscal que la Corte hace suyo)” (CSJN,
31/5/ 05, “Di Mauro, José S. c/Ferrocarriles Metropolitanos SAEL y otro”, LL,
2005-E-809).
Por irregularidades de registro genéricas (ley 25.323, art. 1°)
Antigüedad al despido: 4 años.
Fecha de egreso: 17/11/10.
Remuneración mensual devengada sin SAC: $ 3.057,72. Remuneración
mensual devengada c/SAC: $ 3.312,53.
Cálculo de la indemnización:
Liquidación del recargo indemnizatorio (art. 1o, ley 25.323)
1. Integración mes despido c/SAC ($ 3.312,53/30x13)
2. Ind. suslit. del preaviso c/SAC
3. Ind. por antigüedad s/SAC ($ 3.057,72 x 4)
4. Ind. art. Io, ley 25.323 (rubro 3 x 1)
$ 1.435,43 $ 3.312,53 $ 12.230,88 $ 12.230,88
Referencia normativa. Art. Io, ley 25.323. "Las indemnizaciones previstas por
la ley 20.744, art. 245 y 25.013, art. 7o o las que en el futuro las reemplacen,
serán incrementadas al doble cuando se trate de una relación laboral que al
momento del despido no esté registrada o lo esté de modo deficiente”. En su
parte final agrega: "El agravamiento indemnizatorio establecido en el presente
artículo, no será acumulable a las indemnizaciones previstas por los arts. 8o,
9°, 10 y 15 de la LNE”. Al igual que los casos anteriores, este rubro constituye
una indemnización especial y no una multa, dado que esta última categoría es
ajena al régimen del contrato individual del trabajo y propia del esquema
sancionatorio de la ley 25.212. La norma tiende a compensar el mayor daño
sufrido por el trabajador afectado por los incumplimientos de registro del
empleador, cuando no hubiera logrado acceder a la reparación especial de los
arts. 8o a 10 de la LNE (muchas veces, como consecuencia de la desigualdad
propia del contrato, cuando el empleador advertido precipita su resolución
antes que el trabajador impulse el mecanismo del art. 11 de la LNE o,
también, por defecto al envío de copia del requerimiento a la AFIP). Aquí, la
sustancia del rubro está dado por aquello que se duplica, que es de carácter
indemnizatorio y no por el accidente, que es la mera duplicación. De allí su
carácter indemnizatorio y no punitivo. El monto de esta indemnización especial
estará dado por uno igual al de la indemnización común por despido del art.
245 de la LCT y son dos los requisitos previstos para su procedencia, a saber,
omisión o deficiencia en el registro y que no procedieran las indemnizaciones
previstas por la LNE.
Con relación al primero, su redacción amplía los supuestos de irregularidades
de registro expresamente contemplados por los arts. 8° a 10 de la LNE, dado
que considera comprendidos como tales los casos de registro "deficiente” que,
según el Diccionario de la Real Academia, es algo "falto o incompleto”. Esta
redacción ampliada fue admitida para la procedencia del rubro en casos de
registro deficiente referido a la jornada de trabajo cumplida [trabajador
registrado a “tiempo parcial” o "media jornada” cuando cubría jornada
completa], como se advierte en la siguiente referencia jurisprudencial. “Es
procedente la multa prevista en el art. 1o de la ley 25.323 dada la maniobra de
registrar a la trabajadora como moza de media jornada cuando trabajaba
jornada completa, pues, ello constituye un fraude que no puede validarse
considerando a dicha registración como correcta a los fines de eximirse de la
mencionada indemnización” (CNTrab, Sala VI, 15/9/09, “Fuertes, Silvia A.
c/Alepidote y Caputo SRL”, DT, 2009-118; id., id., Sala IX, 17/6/06, "Clerici,
María C. c/Full Comunicaciones y otro”, LLonline, AR/JUR/3171/2006).
No fue admitida en los casos de registro de falsa categoría profesional: “La
circunstancia de que el trabajador se encontrase registrado con una categoría
profesional distinta a la que le correspondía no resulta suficiente para aplicar el
incremento del art. 1° de la ley 25.323, puesto que el pago insuficiente de la
remuneración no lleva a concluir que la relación se encontrara deficientemente
registrado, pues, en esos casos, el empleador registró los salarios que
efectivamente abonaba al empleado y respecto de las cuales hizo los aportes
correspondientes a los organismos de seguridad social” (CNTrab, Sala III,
16/10/09, "Rodríguez, Cristian I. c/Coto CICSA”, LLonline,
AR/JUR/43646/2009; id., Sala II, 11/9/09, “Ramos, María V. c/Marta Harff SA
y otros”, LLonline, AR/JUR/7227/2009).
c) Por obligar al trabajador a litigar (ley 25.323, art. 2°)
Antigüedad al despido: 6 años y fracción mayor a 3 meses. Fecha de egreso:
14/12/10.
Remuneración mensual devengada sin SAC: $ 3.057,72. Remuneración
mensual devengada c/SAC: $ 3.312,53.
Cálculo de la indemnización:
Liquidación de duplicación indemnizatoria (art. 2° ley 25.323)
1. Integración mes despido c/SAC ($ 3.312,53/31 x 17)
2. Ind. sustit. del preaviso c/SAC ($ 3.312,53 x 2)
3. Ind. por antigüedad s/SAC ($ 3.057,72 x 7)
4. Ind. art. 2°, ley 25.323 (rubros 1 +2+3/2)
$ 1.816,55 $ 6.625,06 $ 21.404,04
$ 29.845,65 $ 14.922,82
Referencia normativa. Art. 2o, ley 25.323. "Cuando el empleador,
fehacientemente intimado por el trabajador, no le abonare las indemnizaciones
previstas en los arts. 232, 233 y 245 de la ley 20.744... y, consecuentemente,
lo obligare a iniciar acciones judiciales o cualquier instancia previa de carácter
obligatorio para percibirlas, éstas serán incrementadas en un 50%”. También
aquí estamos en presencia de una indemnización especial y no de una multa,
resultando de aplicación lo explicado en los casos anteriores. Lo que se repara
es el mayor daño sufrido por el trabajador, que como consecuencia del
incumplimiento de pago del empleador (total o parcial, sea en despido directo
o indirecto), se ve obligado a impulsar el mecanismo administrativo o judicial
necesario para la concreción de su derecho a ser indemnizado por el despido.
Según el texto legal se exige: a) intimación fehaciente por parte del
trabajador; b) resultado negativo, y c) la posterior reclamación administrativa
o judicial. Acerca del contenido y tiempo propio de la intimación, nos
remitimos a lo explicado en el capítulo anterior (§ 20, a). La indemnización
consiste en un monto equivalente al 50% del que corresponda por las
indemnizaciones previstas por los artículos indicados en la norma. Se trata de
la indemnización por integración del mes del despido, sustitutiva del preaviso y
por antigüedad. Ello confirma la referencia jurisprudencial. “A fin de calcular la
indemnización prevista en el art. 2° de la ley 25.323 debe liquidarse el 50% de
los montos correspondientes a indemnización por despido, indemnización
sustitutiva de preaviso e integración del mes de despido, y estas dos últimas
con incidencia del sueldo anual complementario” (CNTrab, Sala III, 29/3/06,
"Presti, María C. c/SPM Sistema de Protección Médica SA y otro", LLonline, AR/
JUR/702/2006).
La norma no distingue los casos de incumplimiento por pago parcial, tampoco
prevé reducción indemnizatoria proporcional para esos casos. Dado que se
exige la previa intimación y la persistencia del incumplimiento del empleador
que obliga al trabajador a litigar, entendemos que el incumplimiento por pago
parcial también debe generar este recargo indemnizatorio. En este sentido, se
ha resuelto que “resulta procedente la indemnización prevista en el art. 2° de
la ley 25.323, pues, si bien es cierto que la demandada realizó un pago al
momento del distracto, dicho pago resultó parcial, ya que no ha abonado las
indemnizaciones debidas conforme a derecho, siendo de aplicación el art. 742
del Cód. Civil que debe complementarse con el art. 260 de la ley de contrato
de trabajo" (CNTrab, Sala VII, 13/2/07, “Jerez, Francisca I. c/Danone
Argentina SA”, LLonline, AR/JUR/481/2007) y, en sentido coincidente, se
sostuvo “A los fines del progreso de la sanción establecida en el art. 2o de la
ley 25.323, la circunstancia de que la empleadora demandada haya satisfecho
en forma parcial la obligación indemnizatoria no la exime del pago de la multa
en cuestión, en tanto la actora cuestionó en forma fehaciente la eficacia
cancelatoria del pago efectuado y la accionada con su actitud dio origen a la
promoción de una acción judicial para obtener el pago íntegro de las sumas
debidas” (CNTrab, Sala II, 17/ 11/06, “Scognamillo, María F. c/Danone
Argentina SA”, LLonline, AR/JUR/9077/2006). La Cámara ha interpretado de
un modo restrictivo el alcance de esta indemnización, declarando que en ella
no están comprendidos los trabajadores que cuentan con indemnizaciones
fijadas por los estatutos del periodista profesional, ley 12.908 (CNTrab,
5/6/07, en pleno, n° 313, "Casado, Alfredo A. c/Sistema Nacional de Medios
Públicos SE”, LL, 2007- D-212, y DT, 2007-668), y encargados de edificios de
propiedad horizontal, ley 12.981 (id., id., 10/9/98, en pleno, n° 320, “Iurleo,
Diana L. c/Cons. de Propietarios del Edificio Luis Sáenz Peña 1195”, LL, 2008-
E-587, y DT, 2008-891).

d) Por omisión de entrega de certificados del art. 80 de la LCT


Remuneración mensual devengada sin SAC: $ 3.057,72. Remuneración
mensual devengada c/SAC: $ 3.312,53.
Cálculo de la indemnización:
Liquidación de la indemnización especial (art. 80, LCT)
1. Remuneración mensual devengada c/SAC $ 3.312,53
2. Ind. art. 80, LCT (rubro 1x3) $ 9.937,59
Referencias normativas. Art. 80 (texto art. 45, ley 25.345), LCT; ley 24.576;
art. 3o, decr. 146/01; res. grab AFIP 2316; res. ANSeS 601/08. Nos hemos
ocupado de los alcances de esta obligación a cargo del empleador y los
requisitos exigidos por la norma a cargo del empleador, en el capítulo primero
(ver § 16, a, 1, y siguientes). El monto de esta indemnización especial es el
equivalente a tres veces la mejor remuneración mensual, normal y habitual
percibida por el trabajador durante el último año o durante el tiempo de
prestación de servicios. Como la remuneración también se percibe con el SAC,
aunque diferido en el tiempo, entendemos que la interpretación que mejor se
ajusta a lo dispuesto por el art. 9" de la LCT es considerar que la base de
cálculo debe incluir la incidencia del SAC, tal como se plantea en el ejemplo de
liquidación.
e) Tarifada por resolución del contrato sin relación de trabajo
Remuneración mensual devengada sin SAC: $ 3.057,72. Remuneración
mensual devengada c/SAC: $ 3.312,53.
Cálculo de la indemnización:
Liquidación de la indemnización especial (art. 24, LCT)
1. Remuneración mensual devengada c/SAC $ 3.312,53
2. Ind. art. 24, LCT, un mes (rubro lxl) $ 3.312,53
Referencia normativa. Art. 24, LCT. En caso de resolución del contrato, antes
de iniciarse la efectiva prestación de servicios, la norma referida remite al
marco regulatorio del derecho común, dado que el sistema de tarifación propio
del derecho del trabajo puede resultar impotente para resolver la situación de
los daños adicionales que tal tipo de resolución pudiera traer aparejados (el
ejemplo típico, es el del trabajador contratado para desempeñarse en un lugar
distante, al que accede con su familia, incurriendo en gastos propios derivados
del traslado y radicación, para que una vez allí se encuentre con la noticia de
la resolución de su contrato). La norma aplicable, del derecho común, es el
art. 1638 del Cód. Civil, que sigue el esquema de reparación integral del daño
así ocasionado con la resolución injustificada del contrato. Con carácter
supletorio, el art. 24 de la LCT agrega que la indemnización en estos casos,
“no podrá ser inferior al importe de un mes de la remuneración que se hubiere
convenido, o la que resulte de aplicación de la CCT correspondiente”. En el
ejemplo de liquidación, calculamos su importe en la versión tarifada y mínima,
que incluye la incidencia del SAC, dado que integra la remuneración mensual,
aun cuando su pago es diferido.

f) Por despido durante la suspensión por enfermedad inculpable


Remuneración mensual devengada sin SAC: $ 3.057,72. Remuneración
mensual devengada c/SAC: $ 3.312,53. Fecha de inicio de la suspensión:
18/11/10.
Fecha del despido: 11/1/11.
Fecha del alta médica: 18/4/11.
Antigüedad del trabajador, sin cargas de familia.
Variante 1. Cuatro años (con tres meses de suspensión máxima paga, art.
208, LCT); el límite temporal está dado por los tres meses, por ser anteriores
al alta y vencen el 17/2/11. El tiempo comprendido en la indemnización,
estará dado por los días que transcurren desde el día siguiente al despido
(12/1/11) hasta el vencimiento de los tres meses de suspensión paga (17/
2/11) y será de un mes y siete días.
Variante 2. Seis años (con seis meses de suspensión máxima paga, art. 208,
LCT): el límite temporal está dado por la fecha del alta médica (18/4/11), por
ser anterior al vencimiento de los seis meses de suspensión paga máxima
prevista para el caso. El tiempo comprendido en la indemnización, estará dado
por los días que transcurren desde el día siguiente al despido (12/1/11) hasta
el alta médica (18/4/11) y será de tres meses y seis días.
Cálculo de la indemnización:
Liquidación de la indemnización especial (art. 213, LCT)
Variante 1, un mes y siete días
1. Remuneración mensual devengada c/SAC $ 3.312,53
2. ídem por 7 días ($ 3.312,53 / 30,4 x 7) $ 762,75
Indemnización art. 213, LCT (en Variante 1) $ 4.075,28
Variante 2, tres meses y siete días
1. Remuneración mensual devengada c/SAC $ 3.312,53
2. Remuneración mensual devengada
c/SAC x 3 meses (x 3) $ 9.937,59
3. ídem por 6 días ($ 3.312,53 / 30,4 x 6) $ 653,79
Indemnización art. 213 LCT (en Variante 2) $ 10.591,38
Referencia normativa. Art. 213, LCT. El presupuesto de hecho, para la
procedencia de esta indemnización especial, está dado por el despido del
trabajador "durante el plazo de las interrupciones pagas por accidente o
enfermedad inculpable”. La norma por defecto alude a "interrupciones”
cuando, en realidad, se ocupa de la "suspensión” de ciertos efectos del
contrato, tal cual se describe en el título X de la LCT, donde se encuentra
regulada la indemnización. Los requisitos para su procedencia son dos:
suspensión pagada por accidente o enfermedad y despido. Puede ocurrir que
el despido sea provocado por el empleador (falta de pago de remuneraciones,
conforme al art. 208, LCT) y la indemnización también proceda por despido
indirecto. El primer requisito exige que el trabajador cumpla con los recaudos
previstos en el art. 209 de la LCT para que la suspensión del contrato se
encuentre justificada y sea “paga”, a saber: aviso al empleador con indicación
del lugar donde se encuentra, o acreditación inequívoca del accidente o
enfermedad. Ello surge de la siguiente referencia jurisprudencial. “El art. 213
de la ley 20.744 de contrato de trabajo, que tiene por finalidad proteger al
trabajador frente al despido arbitrario durante la enfermedad, no es aplicable
al caso donde no se ha acreditado la comunicación de la enfermedad al
empleador, ni de los certificados médicos acompañados surja alguna
prescripción médica que diera cuenta de la misma” (CNTrab, Sala VII, 4/7/06,
“Ronchi, María C. E. c/Pelacchi, María C.”, LLonline, AR/JUR/6536/2006). El
monto de esta indemnización está dado por el importe equivalente a "los
salarios correspondientes a todo el tiempo que faltare para el vencimiento” del
período pago de suspensión (son los plazos del art. 208, LCT), “o la fecha del
alta, según demostración que hiciera el trabajador". El importe de salarios
correspondientes a cierto tiempo, incluye el SAC, puesto que se devenga junto
con ellos. Dos son las variantes de liquidación que proponemos, ya que son
dos los supuestos contemplados en la norma, al referirse a las fechas de
vencimiento de los plazos de suspensión del contrato o la fecha del alta. Dado
que el contexto temporal que delimita el alcance de la norma está en los
plazos del art. 208 de la LCT, la fecha del alta debe hallarse dentro de ellos.
De manera tal que, de ambas fechas, prevalecerá la anterior en el tiempo.
Como se trata de una indemnización, no genera ni aportes ni contribuciones.
Tampoco es considerada como tiempo de servicio.

g) Despido por causa de matrimonio o maternidad Cálculo de la


indemnización:
Liquidación de la indemnización especial (art. 182, LCT)
Con SAC en la base de cálculo
1. Remuneración mensual devengada c/SAC
2. Ind. art. 182, LCT (rubro 1x12)
Sin SAC en la base de cálculo
1. Remuneración mensual devengada s/SAC
2. Ind. art. 182, LCT (rubro 1x13)
Referencia normativa. La indemnización se encuentra regulada por el art. 182
de la LCT y está dada por un monto equiva-
$ 3.312,53 $ 39.750,36
$ 3.057,72 $ 39.750,36
lente a un año de remuneraciones (doce meses + SAC). A dicha indemnización
convergen por remisión expresa los arts. 178 (despido por causa de
embarazo) y 181 (despido por causa de matrimonio) de la misma ley.

h) Compensación por la opción del art. 183, inc. c, LCT


Antigüedad al momento de la extinción del contrato: seis años y fracción
mayor a tres meses.
Remuneración mensual computable (art. 245, LCT). Variante 1: $ 3.312,53.
Variante 2: $ 9.675.
SMVM vigente: $ 1.840.
Cálculo de la indemnización:
Liquidación de la indemnización especial (art. 183, inc. b, LCT)
Con base de cálculo no afectada por el tope del SMVM
1. Remuneración mensual computable
(art. 245, LCT) $ 3.312,53
2. Compensación art. 183, inc. b, LCT
($ 3.312,53 x0,25 x7) $ 5.796,93
Con base de cálculo afectada por el tope del SMVM
1. Remuneración mensual computable
(art. 245, LCT) $ 9.675,00
2. Cálculo sin tope ($ 9.675 x 0,25 x 7) $16.931,25
3. Cálculo con tope ($ 1.840 x 7) $ 12.880,00
Compensación del art. 183, inc. b
(el calculado con tope) $ 12.880,00
Referencia normativa. Art. 183, inc. b, LCT. La trabajadora que, vigente la
relación laboral, tuviera un hijo, continuara residiendo en el país y se hallare al
vencimiento del período de licencia por maternidad, puede acceder a esta
compensación de modo expreso o tácito. La primera vía exige la comunicación
rescisoria al empleador con el detalle que nos hemos ocupado en el capítulo
anterior. La segunda, del modo previsto por el art. 186 de la LCT, no
incorporándose a su empleo luego de vencida su licencia. El importe de esta
compensación es equivalente al 25% de la indemnización común por despido
(art. 245, LCT), con el tope máximo particular aquí previsto, que condiciona el
haber base de cálculo al máximo de un SMVM. El cálculo debe ser realizado
sobre la base del 25% de la remuneración computable a los fines del art. 245
de la LCT y que no debe exceder del importe equivalente a una vez el SMVM.
Este tope máximo, ha quedado restablecido luego de la derogación del art.
141 de la LNE (que impedía utilizar al SMVM como parámetro de cálculos) por
el art. 1° de la ley 26.598. Haremos dos variantes liquidatorias con salarios
diferentes, en un caso no afectado por el importe del SMVM y en otro sí, sobre
la base de un valor de SMVM de $ 1.840.

i) Por infracción al régimen de tutela sindical

1) Despido del candidato


Fecha de notificación al empleador (postulación u oficialización de
candidatura): 15/10/15.
Vencimiento del plazo de estabilidad de seis meses: 14/4/16. Fecha del
despido: 20/1/16.
Tiempo faltante hasta el vencimiento del período de estabilidad (del 21/1/16 al
14/4/16): dos meses y veintiséis días. Remuneración mensual devengada sin
SAC: $ 3.057,72. Remuneración mensual devengada c/SAC: $ 3.312,53.
Cálculo de la indemnización:
Liquidación de indemnización especial (art. 52, LAS, candidato)
Con SAC incluido en la base de cálculo
1. Remuneración mensual devengada c/SAC $ 3.312,53
2. Remuneración c/SAC por dos meses (rubro 1x2) $ 6.625,06
3. Remuneración c/SAC por 26 días
($ 3.312,53/30,4x 26) $ 2.833,08
4. Remuneración c/SAC por el año posterior
(rubro 1 x 12) $ 39.750,36
Indemnización especial (art. 52, LAS) $ 49.208,50
Con SAC discriminado
1. Remuneración mensual devengada s/SAC $ 3.057,72
2. Remuneración s/SAC por dos meses (rubro 1 x 2) $ 6.115,44
3. Remuneración s/SAC por 26 días
($ 3.057,72/30,4x 26) $ 2.615,16
4. Remuneración s/SAC por el año posterior
(rubro 1x12) $ 36.692,64
Subtotal s/SAC $ 45.423,24
5. SAC s/el rubro $ 3.785,27
Indemnización especial (art. 52, LAS) $ 49.208,50
Referencia normativa. Arts. 47 a 52, ley 23.551; arts. 29 y 30, decr. 467/88;
convenios OIT 87, 98 y 135.

Referencias normativas. Arts. 50 y 52, LAS. La indemnización especial, que se


acumula a las comunes, será de una suma equivalente al importe de "las
remuneraciones imputables al período de estabilidad aún no agotado” con más
“el importe de un año de remuneraciones". Como la base de cálculo está dada
por el importe de las remuneraciones que hubiera percibido en ese tiempo, el
cálculo se realiza agregando la incidencia del SAC a la remuneración
devengada. A falta de regulación específica, en caso de remuneraciones
variables, debiera estarse al criterio de la “normalidad próxima”, esto es la
remuneración que debiera percibir normalmente en ese lapso.

2) Despido del representante con mandato vigente


Vencimiento del mandato: 12/5/17.
Fecha del despido: 20/1/16.
Tiempo faltante hasta el vencimiento del mandato (del 21/1/ 16 al 12/5/17):
quince meses y veintitrés días.
Remuneración mensual devengada sin SAC: $ 3.057,72.
Remuneración mensual devengada c/SAC: $ 3.312,53.
Cálculo de la indemnización:
Liquidación de indemnización especial (art. 52, LAS, representante)
Con SAC incluido en la base de cálculo
1. Remuneración mensual devengada c/SAC $ 3.312,53
2. Remuneración c/SAC por 15 meses (rubro 1x15) $ 49.687,95
3. Remuneración c/SAC por 23 días
($ 3.312,53/30,4x23) $ 2.506,19
4. Remuneración c/SAC por el año posterior
(rubro 1x12) $ 39.750,36
Indemnización especial (art. 52, LAS) $ 91.944,50
Con SAC discriminado
1. Remuneración mensual devengada s/SAC $ 3.057,72
2. Remuneración s/SAC por dos meses (rubro 1x15) $ 45.865,80
3. Remuneración s/SAC por 23 días
($ 3.057,72/30,4x23) $ 2.313,41
4. Remuneración s/SAC por el año posterior
(rubro 1x12) $ 36.692,64
Subtotal s/SAC $ 84.871,85
5. SAC s/el rubro $ 7.072,65
Indemnización especial (art. 52, LAS) $ 91.944,50
Referencia normativa. Arts. 48 y 52, LAS. La indemnización especial, que se
acumula a las comunes, será de "una suma equivalente al importe de las
remuneraciones que le hubieren correspondido (al trabajador despedido)
durante el tiempo faltante del mandato y el año de estabilidad posterior”.
Como la base de cálculo está dada por el importe de las remuneraciones que le
hubiera correspondido percibir en ese tiempo, el cálculo se realiza agregando
la incidencia del SAC a la remuneración devengada. A falta de regulación
específica, en caso de remuneraciones variables debiera estarse al criterio de
la “normalidad próxima”.

3) Despido del representante con mandato vencido, dentro del período de


estabilidad posterior
Vencimiento del mandato: 12/5/15.
Fecha del despido: 20/1/16.
Tiempo faltante hasta el vencimiento del año de estabilidad posterior (del
21/1/16 al 12/5/16): tres meses y veintitrés días.
Remuneración mensual devengada sin SAC: $ 3.057,72.
Remuneración mensual devengada c/SAC: $ 3.312,53.
Cálculo de la indemnización:
Liquidación de indemnización especial (art. 52, LAS, mandato vencido)
Con SAC incluido en la base de cálculo
1. Remuneración mensual devengada c/SAC $ 3.312,53
2. Remuneración c/SAC por 3 meses (rubro 1x3) $ 9.937,59
3. Remuneración c/SAC por 23 días
($ 3.312,53 /30,4 x23)$ 2.506,19
Indemnización especial (art. 52, LAS) $ 12.443,78
Con SAC discriminado
1. Remuneración mensual devengada s/SAC $ 3.057,72
2. Remuneración s/SAC por 3 meses (rubro 1 x 3) $ 9.173,16
3. Remuneración s/SAC por 23 días
($ 3.057,72 /30,4 x23)$ 2.313,41
Subtotal s/SAC $ 11.486,57
4. SAC s/el rubro $ 957,21
Indemnización especial (art. 52, LAS) $ 12.443,78
Referencia normativa. Art. 52, LAS. La indemnización especial que se acumula
a las comunes, en el presente caso de mandato ya vencido, queda limitada a
"una suma equivalente al importe de las remuneraciones que le hubieran
correspondido (al trabajador despedido) durante el tiempo (faltante del... año
de estabilidad posterior”. Por las razones indicadas anteriormente, incluiremos
la incidencia del SAC en el cálculo y también aquí corresponde aplicar, en
remuneraciones variables, el criterio de la "normalidad próxima”.

La sanción del art. 132 bis, LCT


Referencia normativa. Art. 132 bis (según ley 25.345, art. 43), LCT; art. 1o,
decr. 146/01. Acerca de los presupuestos de hecho contemplados por la norma
de referencia nos remitimos al análisis realizado en oportunidad de tratar la
intimación cursada por el trabajador. Como el nombre empleado por la norma
lo indica, se trata de una sanción y del tipo conminatoria, similar a la regulada
por el art. 666 bis del Cód. Civil y tiene por objeto compulsar al deudor a la
regularización de una obligación de hacer, consistente en depositar las sumas
que ha retenido de la remuneración de su empleado. El importe de esta
sanción, que es mensual, está dado por un importe "equivalente a la
remuneración que se devengaba mensual mente a favor" del trabajador. La
reglamentación aclara que el importe es el de "la última remuneración
mensual devengada" y no se ha previsto ajuste o incremento sobre dicho
importe en los períodos futuros. Los parámetros temporales que fija la norma,
están dados por la fecha de inicio que es "el momento de producirse la
extinción del contrato”, y la fecha de terminación es el momento en que “el
empleador acreditare de modo fehaciente haber hecho efectivo el ingreso de
los fondos retenidos” (art. 132 bis, LCT); "con más los intereses y multas que
pudieren corresponder” (art. 1o, decr. 146/01).
El texto de la norma es claro y no admite otro modo de conclusión al límite
temporal (como sería el acogimiento del empleador a moratorias), sólo
contrariando el texto de la norma puede admitirse dicha situación como
eximente de la sanción. Esto último, salvo que el plan traiga alguna disposición
expresa en contrario. A falta de excepción expresa al art. 132 bis prevalecen
las consecuencias fijadas por esta norma, aun cuando el empleador incluya los
aportes retenidos y no depositados en algún plan de facilidades. Así, se ha
resuelto que “la excepción que prevé el art. 9° de la ley 26.283, que regula el
régimen de regularización de deudas, está referida a las ‘sanciones
establecidas en las leyes 11.683, 17.250, 22.161 y 24.769 y sus
modificaciones, por lo que no debe considerarse incluida también la sanción
incorporada por la ley 25.345 al art. 132 bis de la LCT’” (CNTrab, Sala II,
5/2/09, ‘‘Laya, Roxana V. c/Fundación Favaloro para la Docencia e
Investigación Médica”, LLonline, AR/JUR/1591/2009).
Y, mediando excepción expresa (tal el caso del art. 12 de la ley 26.283, que
señala: "El acogimiento al presente régimen por parte de los empleadores
implicará para sus trabajadores el reconocimiento de sus aportes
previsionales”), donde la norma especial (plan de facilidades), modifica el
alcance del art. 132 bis de la ley de contrato de trabajo, se ha resuelto:
“Corresponde disponer el cese de la aplicación de la multa impuesta al
empleador en los términos del art. 132 bis de la ley de contrato de trabajo a
partir de la fecha en la que aquél se acogió a una moratoria mediante un plan
de facilidades de pago, en tanto éste aún se encuentra vigente”. Cabe acotar
que esta sentencia interlocutoria fue dictada con posterioridad al cumplimiento
parcial de la sanción que había hecho el empleador, puesto que ¡a sentencia
de 1a instancia había aplicado el criterio de condena a futuro, ratificado luego
por la Cámara en la sentencia definitiva que había dictado antes en esa misma
causa (CNTrab, Sala III, 27/9/07, "Niwa, Beatriz G. c/Obra Social Bancaria
Argentina”, LLonline, AR/JUR/6309/2007).
La sanción termina a partir del momento que el empleador acredita el
cumplimiento total de la obligación (depósito de las sumas retenidas con más
sus intereses) y ello del modo como la propia LCT lo indica en su art. 277, esto
es, en el expediente judicial donde tramita el reclamo. A partir de esta
acreditación cesará la sanción con efectos a partir de la acreditación (ex nunc)
y sin alcance retroactivo (ex tune), dado que esto último no podría ocurrir sin
lesión al derecho de propiedad del trabajador, afectado por la falta de depósito
y reparado el tiempo durante el cual persistió el incumplimiento del
empleador, con la sanción correspondiente a esos períodos ya vencidos. Con
relación al salario computable, dado que se trata de un importe equivalente al
devengado mensualmente, como de dicha condición participa el SAC, en
nuestro ejemplo utilizaremos el salario mensual devengado con dicha
incidencia. En la práctica, los casos más frecuentes son por la imposibilidad
fáctica de practicar la liquidación completa del rubro en el mismo reclamo
laboral (administrativo o judicial), dado que persiste a esa fecha el
incumplimiento del empleador. La solución a la que habitualmente se recurre,
está por la liquidación parcial del rubro, conteniendo los períodos devengados
hasta un momento cercano al inicio de la acción y el requerimiento de condena
al empleador por monto indeterminado, atendiendo a la circunstancia que se
trata de un crédito “cuya estimación” depende “de elementos aún no
definitivamente fijados” (CPCCN y CPCCBA, art. 330, inc. 6).
Dicho monto indeterminado, proviene de los períodos devengados con
posterioridad al último liquidado y hasta el momento en que el empleador
acredite los extremos requeridos por el art. 132 bis y su reglamentación para
el cese de la sanción. Pueden darse luego, en el curso del proceso, distintas
alternativas para la fijación del monto, dado que el empleador puede acreditar
el depósito total en la etapa de conocimiento o con posterioridad, ya en la
ejecución del rubro. En caso de persistir el incumplimiento al momento de la
sentencia, ¿hasta cuándo corresponde la condena? ¿es viable la sentencia que
condene al pago de la sanción por los períodos futuros hasta el momento en
que el empleador acredite el cumplimiento de la obligación?
La solución jurídica parece compleja y se deriva de la ausencia de una norma
procesal expresa, que acompañe el texto de la norma sustancial que
comentamos y permita la condena a futuro, tal como lo hacen el art. 688 del
CPCCN y el art. 677 del CPCCBA para el proceso especial de desalojo, a
beneficio del propietario. Podrían darse, por lo menos, dos interpretaciones,
según se entienda que prevalece la ley procesal sobre la sustantiva o a la
inversa. De ese modo, la primera variante sería considerar que la sentencia no
puede condenar a futuro sin una norma procesal expresa y, por lo tanto, fijar
como límite temporal de la sanción el momento del decisorio definitivo (o del
informe pericial contable). La segunda se fundamentaría en la decisión
prevista en la norma sustantiva y condenar al empleador al pago de los
períodos pasados y futuros, hasta el cumplimiento de la condición extintiva
prevista por la ley de fondo. Esta solución es la que consideramos adecuada,
ya que debe prevalecer la norma sustantiva sobre la adjetiva (arts. 28 y 31,
Const, nacional). Sobre la base de la primera variante, se ha resuelto: "La
sanción prevista en el art. 132 bis de la LCT (t.o. 1976-238) no puede ser
calculada más allá del mes anterior a la sentencia, sin perjuicio del derecho del
trabajador a reclamar su ampliación en un pleito futuro, ya que en nuestro
ordenamiento procesal no es admisible la ‘condena de futuro’ en supuestos
como el analizado” (CNTrab, Sala II, 4/6/08, “Zapata, Griselda E. c/Obra
Social Bancaria Argentina”, LLonline, AR/JUR/4033/2008).
Y se ha aplicado también la segunda, al resolver “modificar en este punto la
sentencia apelada y establecer que la demandada deberá abonar
mensualmente al actor una suma equivalente a la remuneración de $ 1.322,
desde la fecha del despido y hasta que se acredite de modo fehaciente en
autos el efectivo ingreso de los fondos oportunamente retenidos con destino al
sistema de seguridad social, según liquidación que se llevará a cabo en el
momento previsto por el art. 132, LO” (CNTrab, Sala VI, 15/12/09, “Mezza,
José c/Blasco Hnos. y Cía. SA y otro”, LLonline, AR/JUR/ 62848/2009).
Última remuneración mensual devengada c/SAC: $ 3.312,53. Fecha de
egreso: 20/1/10.
Fecha de estimación del rubro (inicio del reclamo): 5/4/10.
Tiempo entre la fecha de egreso y la estimación del reclamo: dos meses y
quince días.
Fecha de acreditación del cumplimiento de la obligación por parte del
empleador: 20/11/10.
Tiempo entre la fecha de egreso y la acreditación del cumplimiento: diez
meses.
Cálculo de la sanción:
Liquidación de la sanción del art. 132 bis, LCT
Cálculo provisional al momento del reclamo
1. Remuneración mensual devengada c/SAC $ 3.312,53
2. Remuneración c/SAC por 2 meses (rubro lx 2) $ 6.625,06
3. Remuneración c/SAC por 15 días
($ 3.312,53 /30,4 x15)$ 1.634,47
Sanción (provisional) del art. 132 bis, LCT al tiempo del reclamo $
8.259,53
Cálculo definitivo luego de acreditado el cumplimiento
1. Remuneración mensual devengada s/SAC $ 3.312,53
2. Remuneración s/SAC por 10 meses (rubro 1x10) $ 33.125,30
Sanción (definitiva) del art. 132 bis, LCT $ 33.125,30
Liquidaciones de rubros derivados de la extinción DEL CONTRATO POR
DESPIDO INJUSTIFICADO, EN ESTATUTOS ESPECIALES

a) Servicio doméstico
Fecha del despido: 21/1/11.
Antigüedad al despido: cinco años y fracción mayor a tres meses.
Categoría profesional: empleada doméstica con retiro, tercera categoría, decr.
ley 326/56 y decr. 7979/56.
Horario de trabajo: de lunes a sábados de 07.30 a 16.30 hs.
Última remuneración percibida: $ 2.200 (diciembre de 2010).
Mejor remuneración devengada: $ 2.200 sin SAC o bien $ 2.383,33 con SAC
(en diciembre de 2010).
Cálculo de la liquidación final e indemnizaciones por despido incausado:
Liquidación final e indemnizaciones por despido sin causa (servicio doméstico)
1. Remuneración del mes de despido
($ 2.200 /31 x21) $ 1.490,32
2. SAC prop. Io 2010 ($ 1.490,32/ 12) $ 124,19
3. Ind. vacaciones no gozadas año ant.
(15 días hábiles: $ 2.200 / 25 x 15) $ 1.320,00
4. SAC s/vacaciones año ant. ($ 1.320 / 12) $ 110,00
5. Vacaciones prop, no gozadas 2010
(21 x 15/365 = 0,86 días hábiles) $ 75,68
6. SAC s/vacaciones prop, no gozadas
($ 75,68 /12) $ 6,31
7. Ind. sustit. del preaviso (10 días)
($2.200/30,4x 10) $ 723,68
8. SAC s/preaviso($ 723,68/ 12) $ 60,31
9. Ind. por antigüedad ($ 2.383,33 / 2 x 6) $ 7.149,99
Total $ 11.060,48
Donde:
Se ha incluido el SAC y su parte proporcional, según lo dispuesto por el art. 10
del decr. ley 326/56.
Las vacaciones no gozadas, por quince días hábiles (dada la antigüedad mayor
a cinco años), son calculadas conforme decr. ley 326/56, art. 4o, inc. 2°.
La indemnización sustitutiva del preaviso se ha calculado en función de los diez
días corridos de anticipación, dada la antigüedad en el caso mayor a dos años
y en función de la remuneración correspondiente al último mes (art. 12, decr.
7979/56) con el SAC proporcional (art. 10, decr. ley 326/56).
La indemnización por antigüedad se ha calculado en función de medio mes de
sueldo en dinero convenido (más la proporción del SAC que lo integra), por
cada año de antigüedad (art. 9°, decr. ley 326/56).
Referencia normativa. Decr. ley 326/56, reglamentado por decr. 7979/56.
Aplicable a trabajadores que presten servicios "dentro de la vida doméstica y
que no importen para el empleador lucro o beneficio económico”. El estatuto
especial es del tipo "cerrado” por tratarse de trabajadores que están excluidos
del régimen general de la LCT (art. 2o, inc. b). Por lo tanto, no resultan de
aplicación las disposiciones del régimen general que regulan indemnizaciones
comunes, especiales o la sanción del art. 132 bis de la LCT que hemos
liquidado más arriba. Tampoco son extensivas a estos trabajadores las
disposiciones de la LNE, dado que el art. 1o de la reglamentación (decr.
2725/91) limita el alcance de sus disposiciones a los trabajadores
comprendidos en la LCT.
En el ejemplo, la remuneración mensual es fija. En caso de remuneraciones
variables, el art. 12 del decr. 7979/56 indica que debe fijarse "conforme al
promedio del sueldo en dinero de los dos últimos años, o del percibido durante
el período de servicios, cuando fuere menor”.

b) Trabajo agrario
Fecha del despido: 15/6/10.
Antigüedad al despido: 6 años y fracción mayor a 3 meses.
Última remuneración percibida: $ 3.200 (mayo de 2010).
Mejor remuneración devengada: $ 3.200 sin SAC o bien $ 3.466,67 con SAC
(en mayo de 2010).
Cálculo de la liquidación final e indemnizaciones por despido incausado:
Liquidación final e indemnizaciones por despido sin causa (trabajo agrario)
1. Remuneración del mes de despido
($ 3.200/30x 15) $ 1.600,00
2. SAC prop. 1o 2010 [($3.200x 5) + 1.600]/ 12 $ 1.466,67
3. Ind. vacaciones no gozadas
(166 x 15/365 = 6,82 días) (3.466,67/25 x 6,82) $ 945,71
4. Ind. por antigüedad ($ 3.200 x 7) $ 22.400,00
5. Incremento del art. 76, inc. b, ley 22.248
(20% s/rubro 4) $ 4.480,00
Total $ 30.892,38
Donde:
El SAC y su proporción resultan de lo dispuesto por los arts. 40 a 42 del
Registro Nacional de Trabajo Agrario (RNTA) y el art. 13 de su reglamentación
(decr. 563/81).
El salario vacacional es liquidado conforme lo determina el art. 22 del RNTA
(dividiendo la remuneración mensual por veinticinco, con inclusión del SAC que
la integra), sobre la base de los quince días corridos que le hubieran
correspondido por antigüedad (art. 19, inc. b, RNTA) habiendo trabajado todo
el año.
La indemnización por antigüedad se ha calculado conforme lo determina el art.
76 del RNTA.
Referencia normativa: Ley 26.727, esta nueva ley reemplaza el anterior
sistema de la ley 22.248, resultando del nuevo régimen que para el caso de
extinción del contrato de trabajo se rige por lo dispuesto en el título XII de la
ley 20.744 (t.o. 1976) y sus modificatorias (art. 16, ley 26.727). Asimismo, el
contrato de trabajo agrario no podrá ser celebrado a prueba por período
alguno (art. 16). Por el art. 22, la indemnización por antigüedad o despido
mínima no podrá ser inferior a dos meses de sueldo, tomando como base la
mejor remuneración mensual, normal y habitual devengada durante el último
año o durante el tiempo de prestación de servicios si éste fuera menor.

c) Pequeña empresa
Referencias normativas. Arts. 83 a 106, ley 24.467. El trabajador dependiente
de la pequeña empresa, habilitada como tal por la autoridad de aplicación, no
es un trabajador excluido de la LCT. Únicamente las disposiciones especiales
contenidas en la norma referida sustituyen su puntual regulación (ausencia de
integración del mes del despido y preaviso de un mes para todos los casos,
art. 95, en lo que aquí interesa) las disposiciones del régimen general.
Fecha del despido: 18/8/16.
Antigüedad al despido: seis años y fracción mayor a tres meses.
Última remuneración percibida: $ 3.200 (mayo de 2010). Mejor remuneración
devengada: $ 3.200, sin SAC, o bien $ 3.466,67, con SAC (en mayo de 2010).
Cálculo de la liquidación final e indemnizaciones por despido incausado:
Liquidación final e indemnizaciones por despido sin causa (pequeña empresa)
1. Remuneración del mes de despido
($ 3.200/31 x 18) $ 1.858,06
2. SAC prop. 2° 2010 [($ 3.200 x 1) + 1.858,06] /12 $ 421,51
3. Vacac. prop. 2010
(230 x 21 / 365 = 13,23 días) (3.466,67 / 25 x 13,23) $ 1.834,56
4. Ind. por antigüedad ($ 3.200 x 7) $ 22.400,00
5. Ind. sustit. del preaviso ($ 3.466,67 x 1)$ 3.466,67
Total
$ 29.980,80

d) Obreros de la industria de la construcción


El estatuto especial para los obreros de la industria de la construcción lo regula
la ley 22.250 y su reglamentación por decr. 1342/81. Es un estatuto del tipo
“abierto”, ya que no se trata de trabajadores excluidos de la LCT y el mismo
régimen especial considera aplicables las disposiciones de esta última ley (art.
35, ley 22.250). Sólo no resultan aplicable el régimen general, en lo referido a
la extinción del contrato de trabajo, expresamente regulada por el régimen
especial.
Por lo tanto y en lo que interesa a este capítulo, mantienen vigencia para estos
trabajadores las disposiciones de la LCT que hemos tratado (arts. 80 y 132
bis). La duplicación indemnizatoria del art. 15 de la LNE también resulta
aplicable y estará dada "por una suma igual a la que le correspondiere al
trabajador en concepto de fondo de desempleo” (decr. 2725/91, art. 15).
Referencia jurisprudencial. "Las disposiciones del art. 8° de la ley 24.013
resultan compatibles con el régimen especial previsto para el personal de la
construcción, por cuanto este último no prevé multa para el empleador ni
indemnización alguna para el trabajador por obligaciones no registradas”
(CNTrab, Sala V, 8/6/07, “Acosta, Recalde A. c/Galuzzi, Alberto E.”, LLon line,
AR/JUR/2669/2007). "El régimen de la ley 24.013 resulta de aplicación a los
trabajadores comprendidos en la ley 22.250” (id., Sala III, 28/2/95,
"Crescendo, Jesús c/Sheed, Federico”, DT, 1995- B-1392).
Fecha de ingreso: 2/9/14.
Fecha de egreso: 22/1/16.
Categoría profesional: Ayudante (construcción) CCT 76/75 y ley 22.250.
Horario de trabajo: lunes a viernes de 06.00 a 19.30 hs. Jornada semanal:
67,5 horas. Horas extraordinarias: 22,5 hs. semanales al 50%, en un mes
promedio de 30,4 días representan 98 horas mensuales al 50% (22,5 / 7 x
30,4).
Última remuneración percibida: $ 900, "a cuenta” (1a quincena enero 2015).
Mejor remuneración mensual devengada: $ 4.132,56, sin SAC o bien $
4.476,94 con SAC (en diciembre de 2015).
Cálculo de la liquidación final por rubros adeudados e indemnizaciones del
régimen especial:
1. Horas extras $ 1.157,87
2. Diferencias salariales $ 666,94
3. SAC prop. Ia cuota año 2010 $ 244,40
4. Remuneración días trabajados en enero de 2010 $ 2.932,78
5. Vacaciones prop, no gozadas 2010 $ 179,08
6. Fondo de desempleo art. 15, ley 22.250 $ 5.620,28
7. Indemnización art. 18, ley 22.250 $ 13.430,82
8. Indemnización art. 19, ley 22.250
(rubros 1 + 2 + 3 + 4) $ 5.001,99
9. Ind. art. 80, LCT (3 meses c/SAC) $ 13.430,82
10. Sanción art. 132 bis, LCT,
primeros dos meses c/SAC $ 8.953,88
Total $51.618,87
Donde:
Los rubros horas extras y diferencias salariales fueron incluidos para posibilitar
el agregado del recargo indemnizatorio del art. 19 del Registro Nacional de la
Industria de la Construcción (RNIC) y su aplicación. El detalle de cómo son
calculados dichos rubros, los aguinaldos, remuneraciones y vacaciones, ha sido
explicado en la parte pertinente de este capítulo y allí nos remitimos.
El rubro “6”, fondo de desempleo, se ha calculado en función de lo regulado
por el art. 15 del RNIC, teniendo en cuenta un supuesto donde éste no fue
ingresado por el empleador. El rubro es una suma líquida acumulada en
función del tiempo de servicios del trabajador y durante su transcurso, que se
devenga mes a mes. Su exigibilidad, entrega, percepción y goce se difiere al
momento de la extinción del contrato de trabajo, cualquiera que sea su causa.
Los aportes obligatorios al fondo de desempleo que debe depositar el
empleador se calculan y liquidan con relación a la remuneración devengada del
trabajador. Dicho aporte debe ser de un mínimo del 12% en caso de una
antigüedad superior a un año, como lo es en nuestro ejemplo; a partir de los
años subsiguientes el del 8%. El sistema establecido por el art. 15 del RNIC
reemplaza al régimen de preaviso e indemnización por antigüedad de la LCT.
El rubro "7” corresponde a la indemnización especial del art. 18 del RNIC, cuyo
presupuesto de hecho consiste en el incumplimiento del empleador, a la
entrega de la libreta de aportes dentro de las cuarenta y ocho horas de
finalizada la relación laboral, seguido por la intimación fehaciente del
trabajador a regularizar dicha situación dentro del plazo de dos días hábiles. La
indemnización especial está dada por un importe que no podrá ser inferior a
treinta días de remuneración mensual ni exceder los noventa días. Fue
liquidado en el máximo previsto, de noventa días con inclusión de SAC ($
4.757,59 x 3 = $ 13.430,82).
El rubro “8” se ocupa de liquidar la indemnización especial del art. 19 del
RNIC, cuyo presupuesto de hecho consiste en el incumplimiento del empleador
al pago de salarios en las cantidades fijadas por escalas complementarias al
CCT aplicable. Se requiere, por parte del trabajador, “intimación fehaciente
formulada dentro de los diez días hábiles contados a partir del momento en
que legalmente deba efectuársele el pago de las remuneraciones
correspondiente al período a que se refiera la reclamación y a condición de que
el empleador no regularice el pago en los tres días hábiles subsiguientes al
requerimiento”. En la liquidación, fue incluido un importe igual a la sumatoria
de los rubros salariales impagos (1 a 4).
El rubro “9”, contempla la indemnización especial del art. 80 de la LCT, fue
calculado desde la base de los parámetros explicados anteriormente, lo propio
con la sanción del art. 132 bis de la LCT. La procedencia de estos rubros es
admisible en el régimen especial, dado que no existe regulación específica de
dichos supuestos en el RNIC que excluya su aplicación.
Referencia jurisprudencial. “Si bien el art. 35 de la ley 22.250 establece que
las disposiciones del estatuto de la construcción excluyen las normas
contenidas en la ley de contrato de trabajo en cuanto se refieran a aspectos de
la relación laboral específicamente contempladas en la ley especial,
corresponde condenar a la demandada al pago de la indemnización prevista en
el art. 45 de la ley 25.345, toda vez que no se observa la existencia en dicho
régimen de obligaciones similares a la previstas en el art. 80 de la ley de
contrato de trabajo" (CNTrab, Sala III, 14/10/09, “Romero, Terencio c/Conitec
SÁ”, LLonline, AR/JUR/43653; id., Sala V, 12/ 8/09, "Urzagasti, Oscar A.
c/Giacomino, Mirla F.”, LLonline, AR/ JUR/29504/2009, entre otros). “Toda vez
que la sanción conminatoria del art. 132 bis de la ley de contrato de trabajo
tiene como finalidad sancionar al empleador que no cumplió con la obligación
de ingresar los montos descontados de los sueldos de sus trabajadores con
destino a los organismos de seguridad social, corresponde aplicarla al
demandado aun cuando la relación laboral se desarrolló en el ámbito de la
industria de la construcción, pues se refiere a aspectos de la relación laboral
no contemplados en el régimen especial” (CTrab Tucumán, Sala VI, "Sánchez,
Seferino F. c/JAPA SRL”, LLNOA, 2006-209).
e) Periodistas profesionales
El régimen lo regula la ley 12.908. Como no se trata de un trabajador excluido
de la LCT, estas disposiciones resultan de aplicación, salvo en los casos
especialmente regulados por esa ley. Estos últimos están dados, en lo
sustancial, por el cálculo de las indemnizaciones derivadas del despido
injustificado, preaviso y vacaciones.
Fecha del despido: 15/8/16.
Antigüedad al despido: 3 años y fracción mayor a 3 meses.
Última remuneración percibida: $ 4.600 sin SAC o bien $ 4.983,33 con SAC
(mayo de 2014).
Remuneración mensual promedio de los últimos seis meses: $ 4.720 sin SAC o
bien $ 5.113,33 con SAC.
Cálculo de la liquidación final e indemnizaciones por despido incausado:
Liquidación final e indemnizaciones especiales (periodista profesional)
1. Remuneración del mes de despido ($ 4.600/31 x 15)
2. SAC prop. 2a cuota año 2014 ($ 4.600 + $ 2.225,81 / 12)
3. Integración mes despido c/SAC ($ 4.983,33/31 x 16)
4. Vacaciones prop, no gozadas 2014 (227 x 15 / 365 = 9,33 días hábiles)
5. Ind. sustit. preaviso, art. 43, inc. b ($ 5.113,33 x 4)
6. Ind. por antigüedad, art. 43, inc. c ($ 5.113,33 x 4)
7. Indemnización especial, art. 43, inc. d ($ 5.113,33 x6)
8. Ind. art. 80, LCT (3 meses c/SAC)
($ 4.983,33 x 3)
9. Sanción art. 132 bis, LCT, primeros 2 meses
c/SAC ($ 4.983,33 x 2)$ 9.966,66
Total $ 104.194,66
Donde:
Los rubros "1” a “4”, "8” y “9” fueron calculados en función de la última
remuneración habitual percibida + SAC.
El rubro “3” fue incluido por entender que, en el régimen especial, no existe
regulación específica acerca del momento en que debe comenzar a correr el
plazo del preaviso que resulte incompatible con la aplicación del sistema
general previsto en el art. 233 de la LCT. Es más, el art. 43 de la ley 12.908
en cierto modo lo compatibiliza, al disponer: “El plazo del preaviso comenzará
a computarse a partir del primer día hábil del mes siguiente al de su
notificación”, con lo cual cabe suponer que corresponde el salario por la
integración del mes del despido, cuando es omitido el preaviso. Sin embargo,
debemos aclarar
$ 2.225,81
$ 568,82
$ 2.572,04
$ 2.324,72
$ 20.453,32
que existe jurisprudencia contradictoria sobre su viabilidad en estos casos.
Referencias jurisprudenciales. A favor, CNTrab, Sala V, 13/7/ 10, “Gervasoni,
Julieta A. R. c/Sistema Nacional de Medios Públicos SE", DT, 2010-3308, y
LLonline, AR/JUR/42809/2010; id., Sala X, 31/8/10, "Hanglin, Rolando V.
c/LS4 Radio Continental SA”, DT, 2010-2898, y LLonline, AR/JUR/51547/2010.
En contra, CNTrab, Sala II, 20/9/06, expte. 16.504/05, SD 94.462, "Esteban,
Alicia B. c/Cámara Argentina de la Propiedad Horizontal y Actividades
Inmobiliarias s/ despido”.
El rubro “4” fue calculado sobre la base de quince días hábiles de vacaciones
anuales, dado que la antigüedad en el ejemplo, no excede los diez años (art.
35, inc. a, ley 12.908).
Los rubros "5” a "7” fueron calculados sobre la base de la remuneración
promedio de los últimos seis meses, tal como lo indica el art. 43, inc. e, de la
ley 12.908 (según ley 16.792). Referencia jurisprudencial. “Resulta
improcedente la aplicación del plenario 'Tulosai' a los fines de determinar la
base remuneratoria derivada del despido, dado que el mismo se refiere al
cálculo de la indemnización prevista el art. 245 de la ley de contrato de trabajo
y a la relación habida entre las partes se aplicó la ley 12.908 como norma
estatutaria específica" (CNTrab, Sala X, 31/8/10, "Hanglin, Rolando V. c/LS4
Radio Continental SA”, DT, 2010-2898, y LLonline, AR/JUR/51547/2010).
El rubro "5” equivale a cuatro meses de indemnización susti- tutiva del
preaviso, dada la antigüedad mayor a tres años del ejemplo (art. 43, inc. b,
ley 12.908, según ley 16.792). Esta indemnización procede sólo en caso de
omisión de preaviso y no cuando éste es cumplido por el empleador.
Referencia jurisprudencial. “El periodista debidamente preavisado de su
cesantía sin causa, no tiene derecho a percibir la indemnización especial
equivalente a seis meses a que se refiere la disposición transitoria del art. 83
de la ley 12.908, convertida en definitiva por la ley 13.503” (CNTrab, en pleno,
n° 55, 30/7/59, “Zapata, Ramón M., y otros c/Promotores Asociados de
Teleradiodifusión”, LL, 96-119, y DT, 1959-722).
El rubro “7” está dado por el importe equivalente a seis meses de sueldo (art.
43, inc. d, ley 12.908, según ley 16.792).
Los rubros "8” y “9” fueron incluidos, por tratarse de un trabajador no excluido
de la LCT y que dichos aspectos de la extinción del contrato no cuentan con
regulación específica que los desplace.
Acerca de la procedencia del recargo previsto por el art. 2° de la ley 25.323,
en el ámbito de la Ciudad de Buenos Aires la jurisprudencia plenaria se ha
expedido por la negativa (CNTrab, en pleno, n° 313, 5/6/07, “Casado, Alfredo
A. c/Sistema Nacional de Medios Públicos SE”, LL, 2007-D-212, y DT, 2007-
668).

f) Empleados administrativos de empresas periodísticas


El régimen especial está dado por el decr. ley 13.839/46, con las
modificaciones introducidas por las leyes 13.502 y 15.535. Como no se trata
de un trabajador excluido de la LCT, estas disposiciones resultan de aplicación,
salvo en los casos especialmente regulados por el régimen particular. Estos
últimos están dados, en lo sustancial, por una indemnización por omisión del
preaviso agravada.
Fecha del despido: 13/9/16.
Antigüedad al despido: 2 años y fracción menor a 3 meses.
Última remuneración mensual devengada: $ 4.600 sin SAC o bien $ 4.983,33
con SAC (mayo de 2014).
Cálculo de la liquidación final e indemnizaciones por despido incausado:
Liquidación final e indemnizaciones especiales (empl. adm. empresas
periodísticas)
1. Remuneración del mes de despido
($4.600/30x 13) $ 1.993,33
2. SAC prop. 2a cuota año 2014
($4.600+ $4.600 + $ 1.993,33/14) $ 932,78
3. Integración mes despido c/SAC
($4.983,33/30 x17) $ 2.823,89
4. Vacaciones prop, no gozadas 2014
(256 x14 /365 = 9,82 días) $ 1.957,45
5. Ind. sustit. preaviso, art. 33 ($ 4.983,33 x 6) $ 29.899,98
6. Ind. por antigüedad, art. 33 ($ 4.983,33 x 2) $ 9.966,66
7. Ind. (art. 80, LCT, 3 meses c/SAC)
($ 4.983,33 x3) $14.949,99
8. Sanción (art. 132 bis, LCT), primeros 2 meses
c/SAC ($ 4.983,33 x 2)$ 9.966,66
Total $ 72.490,74
Donde:
Los rubros "1” a “4”, “8” y "9” fueron calculados en función de la última
remuneración habitual devengada + SAC.
El rubro "3” fue incluido por entender que, en el régimen especial, no existe
regulación específica acerca del momento en que debe comenzar a correr el
plazo del preaviso que resulte incompatible con la aplicación del sistema
general previsto en el art. 233 de la LCT.
El rubro "4” fue calculado sobre la base de catorce días corridos de vacaciones
anuales, dado que el régimen especial no contiene regulación distinta del
instituto y rige el sistema general de la LCT (art. 150 y siguientes).
El rubro “5" fue calculado en función de la remuneración devengada con la
incidencia del SAC, puesto que el art. 33 del régimen especial indica que el
monto de la indemnización por omisión del preaviso es equivalente a seis
meses de sueldo y dicho período devenga el SAC. Se trata de una
indemnización que sólo procede en caso de omisión de preaviso y no cuando el
mismo es cumplido por el empleador.
El rubro “6” fue calculado en base a la remuneración devengada y con
inclusión del SAC. Referencia jurisprudencial. “La liquidación de la
indemnización por antigüedad a que se refiere el art. 33 del decr. 13.839/46
debe practicarse teniendo en cuenta el último sueldo devengado por el
dependiente” (CNTrab, en pleno, n° 75, 13/7/61, "López, Emilio c/Empresa
Periodística Argentina SA”, LL, 103-545, y DT, 1962-428).
Como no se trata de trabajador excluido de la LCT, se adicionaron los rubros
"7” y “8” al sólo título ejemplificativo.

g) Trabajo a domicilio
El trabajo por cuenta ajena llevado a cabo en la vivienda del trabajador o en el
local de un tallerista, cuenta con regulación especial por ley 12.713,
reglamentada por decr. 118.755/42. No se trata de trabajadores excluidos de
la LCT y, por lo tanto, resultan aplicables las disposiciones generales de esta
última. Referencia jurisprudencial. “Para regir las relaciones entre las partes en
el llamado trabajo a domicilio, no es de aplicación la ley 11.729, en cambio,
probada la relación de dependencia, son de aplicación las disposiciones del
decr. 33.302/45 (cap. CXV, ley 12.921). El tallerista y el intermediario no
pueden invocar los beneficios de las referidas leyes” (CNTrab, en pleno, n° 34,
24/ 7/56, "García de Vila, Palmira c/Alegría y Cía. SRL”, DT, 1956- 581). "La
ley 12.713 no constituye un estatuto que regula de modo integral la modalidad
del trabajo a domicilio, pues se trata de una norma de Policía del Trabajo por
lo cual, y sobre la base de lo dispuesto por los arts. 2° y 9° de la LCT, a los
trabajadores a domicilio les resulta aplicable el régimen laboral común. (...)
Corresponde confirmar la sentencia que concluyó que el vinculo entre las
partes fue de naturaleza laboral, ya que la circunstancia que la actora no
tuviera horario fijo no excluye la existencia de subordinación jurídica porque
para cumplir con la entrega de los materiales en las fechas fijadas debía
cumplir con una jornada mínima de trabajo, y el hecho que se abonase una
remuneración por producto entregado revela que se trataba de una forma o
especie de remuneración que es el pago a destajo, medido por unidad de obra,
previsto expresamente en el art. 112 de la LCT” (CNTrab, Sala III, 19/9/05,
"Cardozo Franco, Bárbara V. c/Nostarco SA”, LLonline, AR/JUR/4393/2005).
Fecha del despido: 24/8/16.
Antigüedad al despido: 6 años y fracción mayor a 3 meses.
Remuneración habitual percibida: $ 3.200 (julio de 2012).
Mejor remuneración devengada: $ 3.200 sin SAC o bien $ 3.466,67 con SAC
(en mayo de 2012).
Cálculo de la liquidación final e indemnizaciones por despido incausado:
Liquidación final e indemnizaciones especiales (trabajo a domicilio)
1. Remuneración del mes de despido
($ 3.200/31 x24) $ 2.477,42
2. SAC prop. 2a cuota año 2012
[($ 3.200+ $ 3.200+ $ 2.477,42)/12] $ 739,78
3. Vacac. prop. 2012 (236 x 21 / 365 = 13,58 días)
(3.466,67/ 25 x 13,58)$ 1.883,10
4. Integración mes despido c/SAC
($ 3.466,67 /31 x 7) $ 782,80
5. Ind. sustit. preaviso (2 meses c/SAC) $ 6.933,34
6. Ind. por antigüedad ($ 3.200 x 7) $ 22.400,00
7. Ind. (art. 80, LCT, 3 meses c/SAC)
($ 3.466,67 x3) $10.400,01
8. Sanción (art. 132 bis, LCT), primeros 2 meses
c/SAC ($ 3.466,67 x 2)$ 6.933,34
Total $ 52.549,79
Donde:
Cada uno de los rubros ha sido liquidado según los parámetros que hemos
visto en cada caso, en el régimen general del que participa el trabajo a
domicilio dependiente.

h) Viajante de comercio
El viajante de comercio (trabajador cuya actividad preponderante es la
concertación de ventas por cuenta ajena, fuera del establecimiento patronal)
es un trabajador cuya relación se encuentra comprendida en las disposiciones
de la LCT y complementada con las que surgen de la ley 14.546.
Fecha del despido: 20/11/16.
Antigüedad al despido: 3 años y fracción mayor a 3 meses.
Remuneración habitual percibida (entre haber básico garantizado, adicionales
y comisiones): $ 5.200, sin SAC, o bien $ 5.633,33 con SAC (octubre de
2010).
Mejor remuneración devengada: $ 6.800 (en mayo de 2014).
Cálculo de la liquidación final e indemnizaciones por despido incausado:
Liquidación final e indemnizaciones especiales (viajante de comercio)
1. Comisiones adeudadas s/planilla juramentada
(art. 11, ley 14.546) $ 1.385,00
2. Remuneración del mes de despido
($ 5.200/30x20) $ 3.466,67
3. SAC prop. 2a cuota año 2014
[($ 5.200x4)+$ 3.466,67]/12 $ 2.022,22
4. Vacac. prop. 2010 (324 x 14 / 365 = 12,43 días)
(5.633,33 /25 x 12,43)$ 2.800,89
5. Integración mes despido c/SAC
($ 5.633,33/30x10) $ 1.877,78
6. Ind. sustit. preaviso (1 mes c/SAC) $ 5.633,33
7. Ind. por antigüedad ($ 6.800 x 4) $ 27.200,00
8. Ind. por pérdida clientela (art. 14, ley 14.546)
(rubro 7 x 0,25) $ 6.800,00
Total $51.185,89
Donde:
Los rubros "2” a “7” están liquidados de acuerdo al régimen general y a lo que
hemos tratado en cada caso particular.
El rubro “1” queda condicionado a la declaración jurada presentada por el
trabajador y la presunción que de ella pueda derivarse frente a la prueba en
contrario que debe producir el empleador (art. 11, ley 14.546).
El rubro “8” constituye la indemnización especial por pérdida de clientela del
art. 14 de la ley 14.546 y el importe liquidado, resulta del 25% del calculado
en concepto de indemnización por antigüedad o despido injustificado.

i) Encargados de edificios de propiedad horizontal


El estatuto particular de los encargados de edificios de propiedad horizontal lo
dispuso la ley 12.981 y el decr. 11.296/49. Dichas normas conviven con el
general de la LCT, dado que no se trata de un trabajador excluido de esta
última.
Fecha del despido: 20/9/16.
Antigüedad al despido: 7 años y fracción de un mes.
Remuneración habitual percibida y mejor remuneración mensual devengada: $
6.400 sin SAC o bien $ 6.933,33 con SAC (agosto de 2010).
Cálculo de la liquidación final e indemnizaciones por despido incausado:
Liquidación final e indemnizaciones especiales (encargado de edificio)
1. Remuneración del mes de despido
($6.400/30x20) $ 4.266,67
2. SAC prop. 2a cuota año 2010
[($ 5.200 x2)+$4.266,67]/12 $ 1.422,22
3. Vacac. prop. 2010 (263 x 20/ 365 = 14,41 días)
(6.933,33/20 x 14,41) $ 4.995,46
4. Integración mes despido c/SAC
($ 6.933,33 /30x 10) $ 2.311,11
5. Ind. sustit. preaviso (3 meses c/SAC) $ 20.799,99
6. Ind. por antigüedad ($ 6.933,33 x 8) $ 55.466,64
Total $ 89.262,09
Donde:
Los rubros "1”, “2” y “4” están liquidados según el régimen general y a lo que
hemos tratado en cada caso particular.
El rubro "3” se ha calculado en función de los veinte días hábiles de licencia
previstos para la antigüedad en el empleo utilizada, en nuestro ejemplo, por el
art. 3o, inc. d, de la ley 12.981.
El rubro "5" fue calculado en base a los tres meses de preaviso, fijados por el
art. 6o de la ley 12.981, para el despido posterior al período de prueba
(primeros sesenta días).
Referencia jurisprudencial. “Dado que la LCT -art. 92 bis- establece un período
de prueba mayor al sistema de protección contra el despido arbitrario previsto
en la ley 12.981, este régimen resulta más beneficioso para el encargado de
casa de renta, y por ello, prevalece en virtud del principio de la norma más
favorable” (CNTrab, Sala II, 29/2/08, “Barreto, Evangelista c/Con- sorcio de
Propietarios del Edificio de la calle Independencia 2179”, LLonline,
AR/JUR/589/2008). “El encargado de casa de renta despedido antes de tener
una antigüedad de sesenta días, no tiene derecho a la indemnización
sustitutiva del preaviso” (CNTrab, en pleno, n° 233, 18/5/82, "Romero, Ramón
R. c/Cons. de Propietarios Edif. Agüero 1761", LL, 1982-C-219; DT, 1982-712,
y LLonline, AR/JUR/2819/1982).
El rubro "6" fue calculado en base a lo dispuesto por el art. 6o de la ley 12.981.
La fracción mayor al año (cualquiera que sea) es considerada como un año
más. No hay topes previstos para el cálculo indemnizatorio, dado que esta
norma especial desplaza en su aplicación, al régimen general del art. 245 de la
LCT. Por tal motivo también, el mes de remuneración fue calculado con la
incidencia del SAC, dado que todo período de remuneración lo devenga.
Referencia jurisprudencial. “Teniendo en cuenta que resulta aplicable el art. 6o
de la ley 12.981 el derecho a cada base mensual del resarcimiento a fin de
realizar el cálculo de la indemnización por antigüedad se adquiere por el
transcurso de cualquier fracción de año por poco significativa que sea, no
siendo necesario para ello que transcurran más de tres meses, como lo exige
el art. 245 de la LCT” (CNTrab, Sala IX, 31/5/04, "Borda, Daniel c/Perlui SA",
DT, 2004-1536, y LLonline, AR/JUR/27042004).
Aunque se ha decidido su aplicación para determinar la mejor remuneración
como base de cálculo, en la siguiente referencia jurisprudencial: “La base
salarial a tomar en cuenta para el cálculo de la indemnización por antigüedad
del encargado de edificio es la mejor remuneración mensual, normal y habitual
percibida durante el año anterior al despido, ya que ante la falta de precisión
del art. 6° de la ley 12.981 en torno a las pautas que deben considerarse para
establecer la base salarial, por analogía resulta aplicable el criterio que deriva
del art. 245, LCT” (CNTrab, Sala II, 5/7/10, "Borda, Hugo O. c/Consorcio de
Propietarios del Edificio Esmeralda 486”, LLonline, AR/JUR/42250/2010).
En jurisdicción de la Ciudad de Buenos Aires, la jurisprudencia plenaria ha
interpretado que para estos trabajadores no resulta aplicable el recargo
indemnizatorio del art. 2o de la ley 25.323 (CNTrab, en pleno, n° 320,
10/9/08, “Iurleo, Diana L. c/Cons. de Propietarios del Edificio Luis Sáenz Peña
1195”, LL, 2008-E-587, y DT, 2008-891).

S-ar putea să vă placă și